Prévia do material em texto
1 APOSTILA 01 DE FÍSICO-QUÍMICA – PROF. PEDRO MADEIRA (2022) APOSTILA 01 – FÍSICO-QUÍMICA SUMÁRIO FOLHA DE DADOS 02 01. O ESTUDO DOS GASES 03 Exercícios 19 Gabarito 51 02. TERMODINÂMICA QUÍMICA 57 Exercícios 89 Gabarito 122 2 APOSTILA 01 DE FÍSICO-QUÍMICA – PROF. PEDRO MADEIRA (2022) CONSTANTES Constante de Avogadro = 6,02 x 1023 mol–1 Constante de Faraday (F) = 9,65 x 104 C mol–1 = 9,65 x 104 A s mol–1 = 9,65 x 104 J V mol–1 Volume molar de gás ideal = 22,4 L (CNTP) Carga elementar = 1,602 x 10–19 C Constante dos gases (R) = 8,21 x 10–2 atmLK–1mol–1 =8,31 J K–1mol–1 = 62,4 mmHgLK–1mol–1 = 1,98 calK–1mol–1 Constante gravitacional (g) = 9,81 m.s–2 Constante de Rydberg (R∞hc) = 2,18 x 10–18 J = 13,6 eV Constante de Planck(h) = 6,63 x 10–34 J.s DEFINIÇÕES Pressão de 1 atm = 760 mmHg = 101325 N.m–2 = 760 Torr 1 J = 1 N.m = 1 kg.m2.s – 2 ; 1 pm = 1 x 10–12 m; 1 eV = 1,602 x 10–19 J Condições normais de temperatura e pressão (CNTP): 0°C e 760 mmHg. Condições ambientes: 25°C e 1 atm. Condições-padrão: 25°C, 1 atm, concentração das soluções: 1 mol L−1 (rigorosamente: atividade unitária das espécies), sólido com estrutura cristalina mais estável nas condições de pressão e temperatura em questão. (s) ou (c) = sólido cristalino; (l) ou (l) = líquido; (g) = gás; (aq) = aquoso; (graf) = grafite; (CM) = circuito metálico; (conc) = concentrado; (ua) = unidades arbitrárias; [A] = concentração da espécie química A em mol L−1. MASSAS MOLARES Elemento Químico Número Atômico Massa Molar (g.mol –1) Elemento Químico Número Atômico Massa Molar (g.mol –1) H 1 1,01 Mn 25 54,94 He 2 4,00 Fe 26 55,85 Li 3 6,94 Ni 28 58,71 Be 4 9,01 Cu 29 63,54 B 5 10,81 Zn 30 65,37 C 6 12,01 Ge 32 72,63 N 7 14,01 As 33 74,92 O 8 16,00 Se 34 78,96 F 9 19,00 Br 35 79,90 Ne 10 20,18 Ag 47 107,87 Na 11 22,99 Sn 50 118,69 Mg 12 24,31 Te 52 127,60 Al 13 26,98 I 53 126,90 Si 14 28,09 Xe 54 131,29 P 15 30,97 Ba 56 137,33 S 16 32,06 Pt 78 195,08 Cl 17 35,45 Au 79 196,97 Ar 18 39,95 Hg 80 200,59 K 19 39,10 Pb 82 207,19 Ca 20 40,08 Bi 83 208,98 Ti 22 47,87 U 92 238,00 Cr 24 52,00 3 APOSTILA 01 DE FÍSICO-QUÍMICA – PROF. PEDRO MADEIRA (2022) CAPÍTULO 01 – O ESTUDO DOS GASES CONTEÚDO TÓPICO 01: GASES IDEAIS TÓPICO 02: TEORIA CINÉTICA DOS GASES IDEAIS TÓPICO 03: GASES REAIS TÓPICO 01 – GASES IDEAIS O gás enche qualquer recipiente que o contenha. A) Equações de Estado Forma Geral ! = #(%, ', () Gás Perfeito ! = (*' % van der Waals ! = (*' % − (, − -. ( %/ ! Berthelot ! = *' %" − , − - '%"! Dieterici ! = *'0#(% &'(!⁄ ) %" − , Virial ! = *' %" 12 + 4 %" + 5 %"! +⋯7 OBSERVAÇÕES PROF. PEDRO MADEIRA B) Variáveis de Estado B.1) Pressão SÍMBOLO VALOR 1 Pa 1 N.m–2, 1 kg.m–1.s–2 1 bar 105 Pa 1 atm 101325 Pa 1 Torr (101325/760) Pa 1 mmHg (101325/760) Pa 1 psi 6,894757... kPa Experiência de Torricelli (1643) OBSERVAÇÕES 4 APOSTILA 01 DE FÍSICO-QUÍMICA – PROF. PEDRO MADEIRA (2022) Manômetros de mercúrio OBSERVAÇÕES OBSERVAÇÕES Sistema de coleta de gases sobre líquidos 5 APOSTILA 01 DE FÍSICO-QUÍMICA – PROF. PEDRO MADEIRA (2022) B.2) Temperatura: Lei Zero da Termodinâmica Atenção: Escala Celsius x Escala Absoluta (Kelvin) q (oC) + 273,15 = T(K) Medição de temperaturas 1) Termoscópio de Galileu (1593): termômetro de água/gás 2) Termômetro de Galileu (1666): inventado por um grupo de pesquisadores que incluía Torricelli 3) Fahrenheit (1709 e 1714): termômetros de álcool e Hg 4) Medida da resistência elétrica de um material conhecido. Ex: sensor de platina metálica (acima) e sensor de RuO2 (abaixo). 5) Medição da pressão de um gás em condições ideias 6) Medição da pressão de vapor em equilíbrio com o liquido. Ex: He(l) em equilíbrio com seu vapor. 6 APOSTILA 01 DE FÍSICO-QUÍMICA – PROF. PEDRO MADEIRA (2022) 7) Termômetro de infravermelho 8) Termopar 9) Carnot 10) Estatística Crédito da imagem: LMU/MPQ Munich RESOLVA AGORA 01 (ITA 2015) Contribuiram de forma direta para o desenvolvimento do conceito de pressão atmosférica A ( ) Friedrich August Kekulé e John Dalton. B ( ) Michael Faraday e Fritz Haber. C ( ) Galileu Galilei e Evangelista Torricelli. D ( ) Jöns Jacob Berzelius e Eduard Büchner. E ( ) Robert Bunsen e Henry Louis Le Chatelier. B.3) Volume SÍMBOLO VALOR 1 m3 1000 L 1 dm3 1 L 1 cm3 1 mL PROF. PEDRO MADEIRA C) Leis Empíricas dos Gases Ideais Para que um gás se comporte como ideal, a pressão do gás precisa estar próxima de zero (gás rarefeito). O raciocínio atual para o entendimento das leis empíricas e confecção dos gráficos, deve se basear na equação PV = nRT. Boyle (1661) Charles – Gay-Lussac OBSERVAÇÕES Pr es sã o Volume Pr es sã o 1/Volume Aumento da temperatura Aumento da temperatura Diminuição da pressão Vo lu m e Temperatura (oC) -273 0 Temperatura (K) Diminuição do volume Pr es sã o 0 0 7 APOSTILA 01 DE FÍSICO-QUÍMICA – PROF. PEDRO MADEIRA (2022) Experimento de Charles V = constante x (q + 273,15oC) V = constante x T; p = constante x T “Para uma massa fixa de gás sob pressão constante, o aumento relativo de volume por grau de aumento de temperatura é o mesmo para todos os gases” OBS: coeficiente de expansão térmica a 0oC = ao. Escala gasosa de temperatura: è Se t é em oC, T = 273,15oC + t RESOLVA AGORA 02 A lei de Charles também se escreve como V = V0 (1 + aq), onde q é a temperatura Celsius, a é uma constante e V0 o volume da amostra do gás a 0oC. Para o nitrogênio a 0oC, obtiveram-se os seguintes valores de a: p/Torr 749,7 599,6 333,1 98,6 103 a/(oC)–1 3,6717 3,6697 3,6665 3,6643 Escolha a melhor medição para estimar o valor do zero absoluto de temperatura na escala Celsius. Efetue este cálculo a partir deste ponto. o o p 1 Vα = V t ¶æ ö ç ÷¶è ø ( )o o o o o op V 1V = V + t V = V 1+ α t V = V α + t t α æ ö¶æ ö Þ Þ ç ÷ç ÷¶è ø è ø o 1T = + t α æ ö ç ÷ è ø 8 APOSTILA 01 DE FÍSICO-QUÍMICA – PROF. PEDRO MADEIRA (2022) RESOLVA AGORA 03 (IME 2010) As alternativas abaixo representam processos hipotéticos envolvendo 2 mols de um gás ideal, contidos em um conjunto cilindro-pistão. Assinale a alternativa que apresenta mais de três estados (V, T) nos quais a pressão é máxima: (A) (B) (C) (D) (E) PROF. PEDRO MADEIRA D) Equação de Estado do gás ideal (Clapeyron): !!"! #!$! = !""" #"$" = & R = constante universal dos gases ideais. R 8,31447 J.K–1.mol–1 8,20574 x 10 – 2 L.atm.K–1.mol–1 62,364 L.Torr.K–1.mol–1 1,98721 cal.K–1.mol–1 OBSERVAÇÕES V(Litros) 15 12,5 10 300 390 480 T(K) V(Litros) 12 9 300 330 450 480 T(K) V(Litros) 15 10 320 480 T(K) V(Litros) 12 9 300 330 450 480 T(K) V(Litros)15 10 300 450 T(K) 9 APOSTILA 01 DE FÍSICO-QUÍMICA – PROF. PEDRO MADEIRA (2022) RESOLVA AGORA 04 (IME 2008) Uma amostra de 0,512 g de uma liga metálica Al-Zn reage com HCl, recolhendo-se o gás formado. Após a total dissolução da amostra, o gás recolhido é seco, resfriado e submetido a um processo de compressão representado pela reta AB no diagrama P-V. Sabendo que a temperatura máxima ao longo do processo de compressão é 298K, determine o teor de alumínio nesta amostra. Considere que o gás se comporta idealmente. PROF. PEDRO MADEIRA E) Volumes molares Condições Vm (L/mol) CNTP (1atm; 273K) 22,4 CNTP (1bar; 273K) 22,71 CNATP (1atm; 298K) 24,4 (1atm; 300K) 24,6 PROF. PEDRO MADEIRA F) Densidades Gasosas F.1) Absoluta Observe a dedução: !% = (*' ⇒ ! *' = ( %⟹ ! *' = : ;%⟹ !; *' = : % < = =; *' F.2) Relativa Observe a dedução: <+ <, = =+;+ *'+ =,;, *', ⇒ <+ <, = =+;+', =,;,'+ <+ <, = ;+ ;, PROF. PEDRO MADEIRA G) Misturas Gasosas G.1) Fração Molar >+ = (+ (&-& ; >, = (, (&-& ; A>. = 2 . G.2) Fração Volumétrica Princípio de Avogadro: nas mesmas condições de pressão e temperatura, o volume dos gases é diretamente proporcional ao número de moléculas (mol). Assim, a fração molar de um gás em uma mistura é igual à sua fração volumétrica. Ex: Ar atmosférico seco: 21% em volume de O2 = fração molar de gás oxigênio na mistura. G.3) Massa Molar aparente Se 1 mol da mistura gasosa tiver sua massa medida, por exemplo, através do seu volume molar, a massa acusada será a média ponderada das massas molares das moléculas da mistura. Observe: ;%/ =A>.;. . Ex: Ar atmosférico seco: Map = 0,21 x 32 + 0,79 x 28 = 28,8 g/mol Prof. Pedro Madeira P(atm) 0,25 0,40 V (L) 0,90 0,60 B A 10 APOSTILA 01 DE FÍSICO-QUÍMICA – PROF. PEDRO MADEIRA (2022) G.4) Fração em massa A fração em massa de cada gás deve ser calculada através da massa molar aparente. Observe: #+ = :+ :010%2 = >+;+ ;%/ Ex: fração em massa do O2 no ar seco: #-" = B, C2 ∙ EC CF, F ≈ B, CE G.5) Pressões Parciais – Lei de Dalton Se um determinado gás da mistura ocupasse sozinho o volume total da mistura na mesma temperatura, a pressão deste gás seria menor do que a pressão total. Denomina-se de pressão parcial tal pressão. Como a pressão é diretamente proporcional ao número de moléculas, a Lei de Dalton estipula que a pressão parcial de um gás é dada pelo produto da pressão total pela sua fração molar: =+ = =&-& ∙ >+; A=. = =&-& . G.6) Volumes Parciais – Lei de Amagat De forma análoga, tem-se a lei dos volumes parciais: %+ = %&-& ∙ >+; A%. = %&-& . G.7) Equação de estado para misturas. Observe a dedução: =&-&%&-& = (&-&*'⟹ =&-& ∙ >+ ∙ %&-& = >+(&-&*' Assim, =+%&-& = (+*' ou =&-&%+ = (+*' Nunca utilize as duas grandezas parciais em misturas: PAVA = nART ERRADO RESOLVA AGORA 05 (ITA 2003) Determine a massa específica do ar úmido, a 25oC e pressão de 1 atm, quando a umidade relativa do ar for igual a 60%. Nessa temperatura, a pressão de vapor saturante da água é igual a 23,8 mmHg. Assuma que o ar seco é constituído por N2(g) e O2(g) e que as concentrações dessas espécies no ar seco são iguais a 79 e 21% (v/v), respectivamente. G.8) Balanceamento de combustões com ar. Ex1: CH4 (completa) com ar estequiométrico (1 O2 : 4 N2) Ex2: TNT (completa) com ar estequiométrico (21 O2: 79 N2) RESOLVA AGORA 06 (IME 2010 – Q32) Em um recipiente fechado queima-se propano com 80% da quantidade estequiométrica de ar. Admitindo que não haja hidrocarbonetos após a combustão, que todos os produtos da reação estejam na fase gasosa e que a composição volumétrica do ar seja de uma parte de O2 para quatro partes de N2, calcule a porcentagem molar de CO2 no recipiente após a combustão (considere comportamento ideal para os gases). (A) 4,35% (B) 4,76% (C) 5,26% (D) 8,70% (E) 14,28% RESOLVA AGORA 07 (ITA 2020 – Q68) Considerando que o ar é composto aproximadamente de 21% de O2 e 79% de N2 em volume, tem-se que a razão molar ar/combustível da combustão completa de um determinado alcano é igual a 59,5. A partir desse dado, assinale a alternativa que corresponde à soma dos coeficientes estequiométricos de todas as substâncias presentes nessa reação. A ( ) 30,5 B ( ) 55,5 C ( ) 82,0 D ( ) 112,0 E ( ) 124,5 11 APOSTILA 01 DE FÍSICO-QUÍMICA – PROF. PEDRO MADEIRA (2022) RESOLVA AGORA 08 – BASEADA NO IME 2022 Forneça o balanceamento da combustão de C8H18 com ar (21% O2 e 79% N2) nas seguintes condições: a) considerando 20% de combustão incompleta dando CO e 80% de combustão completa. b) um motor de 5,7 L contendo 0,3g de combustível e ar a 100ºC e 1 atm, considerando 20% de combustão incompleta dando CO e 80% de combustão completa. Calcule a porcentagem molar final de cada componente. TÓPICO 02 – TEORIA CINÉTICA DOS GASES IDEAIS Postulados: • O gás ideal é uma coleção de partículas de massa “m” e volume desprezível que se deslocam caoticamente e por todo o volume do recipiente que o contém. • Não existem interações intermoleculares. • A única forma de interação entre as partículas é através das colisões perfeitamente elásticas entre elas e entre elas e a parede do recipiente. A) Relação entre a velocidade quadrática média e a temperatura. ' = ()*+, OBSERVAÇÕES PROF. PEDRO MADEIRA 12 APOSTILA 01 DE FÍSICO-QUÍMICA – PROF. PEDRO MADEIRA (2022) B) Relação entre a energia cinética média e a temperatura. !! = # $%& PROF. PEDRO MADEIRA C) Distribuição das velocidades gasosas (Maxwell-Boltzmann) A distribuição das energias cinéticas de uma amostra gasosa segue a distribuição de Maxwell, ilustrada pela equação a seguir: #(H) = IJ1 ; CJ*'7 3 !⁄ H!0#45" !'&⁄ f(v) = fração de partículas com velocidade “v”. O gráfico a seguir é obtido a partir da equação da distribuição de Maxwell: Para uma mesma temperatura, o gás mais leve possui mais velocidade (curva V). Já o gás mais pesado possui menos velocidade média (curva I). Outra abordagem para o gráfico é de que um mesmo gás em temperaturas mais altas é mais veloz em média (curva V) do que em temperaturas mais baixas (curva I). PROF. PEDRO MADEIRA D) Velocidades Notáveis no gráfico da distribuição de Maxwell. vp = velocidade mais provável; vma = velocidade média aritmética; v = velocidade quadrática média (velocidade média). OBSERVAÇÕES PROF. PEDRO MADEIRA 13 APOSTILA 01 DE FÍSICO-QUÍMICA – PROF. PEDRO MADEIRA (2022) E) Velocidades relativas de dois gases (Lei de Graham). OBSERVAÇÕES RESOLVA AGORA 09 (ITA 2011) Considere dois cilindros idênticos (C1 e C2), de paredes rígidas e indeformáveis, inicialmente evacuados. Os cilindros C1 e C2 são preenchidos, respectivamente, com O2(g) e Ne(g) até atingirem a pressão de 0,5 atm e temperatura de 50ºC. Supondo comportamento ideal dos gases, são feitas as seguintes afirmações: I. O cilindro C1 contém maior quantidade de matéria que o cilindro C2. II. A velocidade média das moléculas no cilindro C1 é maior que no cilindro C2. III. A densidade do gás no cilindro C1 é maior que a densidade do gás no cilindro C2 . IV. A distribuição de velocidades das moléculascontidas no cilindro C1 é maior que a das contidas no cilindro C2. Assinale a opção que apresenta a(s) afirmação(ões) CORRETA(S). A ( ) Apenas I e III. B ( ) Apenas I e IV. C ( ) Apenas II. D ( ) Apenas II e IV. E ( ) Apenas III. PROF. PEDRO MADEIRA F) Frequência de colisões e livre caminho médio. ; ; Onde Z = frequência de colisões e l = livre caminho médio OBS: o livre caminho médio é independente da T se o gás estiver confinado em um recipiente de volume constante, pois T/p é constante nesta situação. Já a freqüência de colisões aumenta pois a velocidade relativa é dependente da raiz de T. OBS2: o livre caminho médio só depende do volume. Esta dependência é linear. OBSERVAÇÕES A B A B A B v M T = v M T relv p Z = k T s × × × k T = 2 p l s × × × reduzidamassa mm mm ondekTv BA BA rel = + == µ pµ 8 14 APOSTILA 01 DE FÍSICO-QUÍMICA – PROF. PEDRO MADEIRA (2022) Atenção ao quadro conclusivo: Condição Transformação H! H6 K! K6 L! L6 Isocórica Isotérmica Isobárica Qualquer RESOLVA AGORA 10 (ITA 1989) Consideremos um gás formado de moléculas todas iguais e que corresponda ao que se considera um gás ideal. Este gás é mantido num recipiente de volume constante. Dentre as afirmações abaixo, todas referentes ao efeito do aumento de temperatura, assinale a CORRETA, em relação ao caminho livre médio das moléculas e à freqüência das colisões entre as mesmas: Caminho livre médio Freqüência de colisões A ( ) Inalterado Aumenta B ( ) Diminui Inalterada C ( ) Aumenta Aumenta D ( ) Inalterado Diminui E ( ) Diminui Aumenta PROF. PEDRO MADEIRA G) Velocidades de difusão e efusão gasosas. A velocidade de difusão é dada por espaço/tempo. A velocidade de efusão é dada por uma quantidade /tempo. Nos dois casos, a lei de Graham é válida. H78 = !M1N+ √CJ;*' OBSERVAÇÕES RESOLVA AGORA 11 (ITA 1992) Um recipiente A contém, inicialmente, uma mistura gasosa, comprimida, dos isótopos 20 e 22 do Neônio. Este recipiente é envolvido completamente por outro, B, conforme a figura ilustrada abaixo. No inicio, o recipiente B estava completamente evacuado. Por um pequeno furo na parede de A, o gás escapa de A para B. Numa situação deste tipo, a concentração (em fração molar) do isótopo mais leve no gás remanescente dentro do recipiente A, em função do tempo, a partir do início do vazamento: A ( ) permanece constante. B ( ) vai diminuindo sempre. C ( ) vai aumentando sempre. D ( ) aumenta, passa por um máximo, retomando ao valor inicial. E ( ) diminui, passa por um mínimo, retomando ao valor inicial. OBSERVAÇÕES RESOLVA AGORA 12 (ITA 2019 – Q08 FÍSICA) Em um reservatório são armazenados 1 mol de gás hélio e 1 mol de gás oxigênio em equilíbrio térmico. Por meio de um orifício de dimensões muito menores que o comprimento livre médio das espécies gasosas, inicia- se um vazamento de gás para o exterior. Sobre essa situação são feitas as seguintes afirmações: I. No interior do reservatório, os átomos de hélio têm, em média, energia cinética menor em comparação à das moléculas de oxigênio. II. No interior do reservatório, os átomos de hélio têm, em média, velocidade de translação maior em comparação à das moléculas de oxigênio. III. A porção do gás que vaza e a que permanece no interior do reservatório têm a mesma fração molar de hélio. 2 2 1 1 T P=4 =4 T P Þ 2 2 1 1 P V 1=4 = P V 4 Þ 2 2 1 1 T V=4 =4 T V Þ FURO A B 15 APOSTILA 01 DE FÍSICO-QUÍMICA – PROF. PEDRO MADEIRA (2022) A ( ) Apenas a afirmação I é falsa. B ( ) Apenas a afirmação II é falsa. C ( ) Apenas a afirmação III é falsa. D ( ) Há mais de uma afirmação falsa. E ( ) Todas as afirmações são verdadeiras. TÓPICO 03 – GASES REAIS A) Interações Intermoleculares As interações intermoleculares podem ser avaliadas pelo fator de compressibilidade, Z: Se Z = 1 è o comportamento é ideal e as interações intermoleculares são nulas. Se Z > 1 è as interações de repulsão são predominantes de modo que o volume observado é maior do que o estimado pela equação dos gases ideais. Se Z < 1 è as interações de atração são predominantes. Graficamente, tem-se: OBSERVAÇÕES PROF. PEDRO MADEIRA real ideal V Z = V 16 APOSTILA 01 DE FÍSICO-QUÍMICA – PROF. PEDRO MADEIRA (2022) B) Isotermas Reais Observe o esquema das isotermas reais: PROF. PEDRO MADEIRA OBSERVAÇÕES C) A Equação de van der Waals Correções feitas por van der Waals à equação dos gases ideais: (V – nb) è volume de movimentação das moléculas gasosas. [a(n/V)2] è fator de diminuição da pressão devido às interações de atração. OBS.: A pressão do gás depende da freqüência de colisões e da força das colisões Assim, a equação de van der Waals é dada por: RESOLVA AGORA 13 (IME 2021) O modelo dos gases ideais, ou perfeitos, descreve bem o comportamento para a maioria dos casos, no entanto, foi necessário desenvolver modelos mais precisos dentre os quais se destaca a equação de Van der Waals. Deduza a equação de Van der Waals, assumindo que o volume da partícula/molécula não seja desprezível e existam interações entre as partículas moléculas. Considere o seguinte: • V é o volume do recipiente do gás; • B é o volume total ocupado pelas moléculas do gás; • As forças de atração são praticamente nulas no seio da mistura do gás; e • Próximo às paredes do recipiente, as moléculas são atraídas ao centro com uma força proporcional ao quadrado da concentração do gás, o que reduz a intensidade dos impactos nas paredes do recipiente. ( ) ( ) [ ] 2 2 nRT n n p = - a ou p+a V - bn = nRT V-nb V V é ù ê ú ë û 17 APOSTILA 01 DE FÍSICO-QUÍMICA – PROF. PEDRO MADEIRA (2022) RESOLVA AGORA 14 a) De acordo com a equação de van der Waals, o volume crítico é VC = 3b e a pressão crítica é PC = (a/27b2). Calcule a temperatura crítica e o fator de compressibilidade previsto no ponto crítico. b) Encontre a expressão do princípio dos estados correspondentes para um gás de van der Waals. Esta expressão relaciona a pressão reduzida (PR) com as outras variáveis reduzidas (TR e VR). Dados: PR = P/PC; TR = T/TC; VR = V/Vc; RESOLVA AGORA 15 (ITA 1998) A figura a seguir mostra de forma esquemática três isotermas, pressão versus volume, para o caso de um gás ideal. Trace isotermas análogas para o caso de um gás real que, por compressão, acaba totalmente liquefeito. No seu gráfico deve ficar claro, para cada isoterma, quais são os pontos que correspondem ao início e ao fim da liquefação em função da redução do volume. PROF. PEDRO MADEIRA D) Isotermas de van der Waals e a correção de Maxwell A reta paralela correspondente à pressão de vapor do líquido deve ser traçada de tal forma que as áreas A1 e A2 sejam iguais (critério de Maxwell). PROF. PEDRO MADEIRA 18 APOSTILA 01 DE FÍSICO-QUÍMICA – PROF. PEDRO MADEIRA (2022) E) Tabelas Tabela 1: Constantes Críticas de gases Pc / atm Vc / (cm3mol-1) TC / K ZC TB / K He 2,26 57,76 5,21 0,305 22,64 Ne 26,86 41,74 44,44 0,307 122,1 Ar 48,00 75,25 150,72 0,292 411,5 Kr 54,27 92,24 209,39 0,291 575,0 Xe 58,0 118,8 289,75 0,290 768,0 F2 55 144 Cl2 76,1 124 417,2 0,276 Br2 102 135 5840,287 H2 12,8 64,99 33,23 0,305 110,0 N2 33,54 90,10 126,3 0,292 327,2 O2 50,14 78,0 154,8 0,308 405,9 CH4 45,6 98,7 190,6 0,288 510 C2H4 50,50 124 283,1 0,270 C2H6 48,20 148 305,4 0,285 C6H6 48,6 260 562,7 0,274 CO2 72,85 94,0 304,2 0,274 714,8 H2O 218,3 55,3 647,4 0,227 NH3 111,3 72,5 405,5 0,242 HCl 81,5 81,0 324,7 0,248 HBr 84,0 363,0 HI 80,8 423,2 PROF. PEDRO MADEIRA Tabela 2: Constantes de Van der Waals a (L2.atm/mol) b (L/mol) He 0,03412 0,02370 Ne 0,21070 0,01709 H2 0,24440 0,02661 O2 1,36000 0,03183 N2 1,39000 0,03913 Cl2 6,49300 0,05622 CO 1,48500 0,03985 NO 1,34000 0,02789 CO2 3,59200 0,04267 H2O 5,46400 0,03049 NH3 4,17000 0,03707 CH4 2,25300 0,04278 C2H2 4,39000 0,05136 C2H4 4,47100 0,05714 C2H6 5,48900 0,06380 CH3OH 9,52300 0,06702 19 APOSTILA 01 DE FÍSICO-QUÍMICA – PROF. PEDRO MADEIRA (2022) CAPÍTULO 01 – O ESTUDO DOS GASES EXERCÍCIOS TÓPICO 01 – GASES IDEAIS SEÇÃO VESTIBULARES 1. (UFC 1998) Observe a reação da nitroglicerina Quando a nitroglicerina, C3H5N3O9, explode, todos os produtos são gases. Utilizando a equação da reação dada abaixo e os dados a seguir apresentados, calcule o volume total de gases, em litros, produzido nas condições normais de temperatura e pressão, quando 454g de nitroglicerina explodem. Reação: 4C3H5N3O9(!) ® 12CO2(g)+6N2(g)+10H2O(g)+O2(g) Dados: Massa molar de C3H5N3O9 = 227g Volume molar nas CNTP = 22,4L Marque a opção que apresenta o cálculo correto do volume solicitado. A) 22,4L B) 44,8L C) 156,8L D) 324,8L E) 649,6L 2. (UFC 1996) Observe o funcionamento do air bag A indústria automobilística tem utilizado um novo dispositivo de segurança, instalado nos carros, que é constituído por um balão inflável (air bag), o qual, após impacto do veículo, infla em quarenta milisegundos pela injeção de nitrogênio gasoso. Este gás é originado do composto NaN3(s), armazenado no balão, o qual se decompõe através da reação: 2NaN3(s) ® 2Na(s) + 3N2(g) Calcule a quantidade de NaN3(s) necessária para gerar o volume de 50L à temperatura de 27 ºC e pressão de 2 atm. 3. (Fac. Albert Einstein) A reforma do biogás O biogás, majoritariamente constituído por metano é uma mistura gasosa obtida a partir da degradação da matéria orgânica. Essa mistura, quando descartada na atmosfera, causa inúmeros danos ao meio ambiente. O processo denominado reforma a vapor do biogás, representado na equação, produz gás hidrogênio, uma fonte alternativa aos combustíveis fósseis. CH4(g) + H2O(g) → CO(g) + 3 H2(g) DH = 206 kJ/mol Considerando R = 0,08 atm.L/mol.K, a quantidade mínima de energia que deve se fornecida na reforma a vapor com quantidade suficiente de metano para produção de 100 L de gás hidrogênio, armazenados a 300 K e 3,6 atm, é igual a a) 512 kJ. b) 1536 kJ. c) 1030 kJ. d) 206 kJ. e) 2060 kJ. 4. (UFC 1998) Observe o poluente O monóxido de carbono é um dos poluentes do ar presente, especialmente, em zona urbana. A pressão do monóxido de carbono de 0,004 atm, no ar, resulta em morte, em pouco tempo. Um carro ligado, porém parado, pode produzir, entre outros gases, 0,60 mol de monóxido de carbono por minuto. Se uma garagem a 27oC tem volume de 4,1 x 104 L, em quanto tempo, na garagem fechada, atinge-se a concentração letal de CO? Obs: Considere que a pressão, na garagem, permanece constante e que não há monóxido de carbono presente, inicialmente. (Constante dos gases = 0,082 L.atm/mol.k) 5. (UCS 2020) Combustível e comburente A dimetil-hidrazina e o tetróxido de dinitrogênio formam uma mistura autoignitora para propulsores de foguetes espaciais. Essa combinação de combustíveis foi utilizada em alguns motores dos módulos espaciais que pousaram na Lua durante as missões Apollo na década de 1970. A reação química que ocorre entre esses dois compostos pode ser representada, simplificadamente, por meio da equação descrita abaixo: C2H8N2(liq) + N2O4(liq) → N2(g) + H2O(g) + CO2(g) Admitindo que o tetróxido de dinitrogênio esteja em excesso e que o rendimento da reação seja 100%, pode- se concluir que o volume total de gases liberado nas CNTP, a partir de 240 g de dimetil-hidrazina, é de a) 572,8 L. b) 658,6 L. c) 724,2 L. d) 806,4 L. e) 930,0 L. 6. (UFC 2001) Ao desejar identificar o conteúdo de um cilindro contendo um gás monoatômico puro, um estudante de Química coletou uma amostra desse gás e determinou sua densidade, d = 5,38 g/L, nas seguintes condições de temperatura e pressão: 15oC e 0,97 atm. Com base nestas informações, e assumindo o modelo do gás ideal: A) Calcular a massa molar do gás. B) Identificar o gás. 7. (UECE – 2003) O gás metano, conhecido há mais de 2000 anos pelos chineses que o utilizavam em processos de mumificação, é o composto mais simples da química orgânica. Imagine que 6,4 kg de gás metano e 25,6 kg de um gás X ocupam volumes iguais na mesma pressão e temperatura. O mol do gás X é: A) 8g B) 32g C) 48g D) 64g 20 APOSTILA 01 DE FÍSICO-QUÍMICA – PROF. PEDRO MADEIRA (2022) 8. (UECE 2006) Tem algo errado neste enunciado Através da eletrólise de uma solução de sais, obteve-se no cátodo uma mistura de gases com a seguinte composição em massa: 67% de cloro; 28% de bromo e 5% de oxigênio. A porcentagem volumétrica do cloro na mistura é, aproximadamente: A) 67% B) 72% C) 74% D) 76% 9. (UECE 2007) Recipientes conectados Um frasco de 250 mL contém neônio a uma pressão de 0,65 atm. Um outro frasco de 450 mL contém argônio a uma pressão de 1,25 atm. Os gases são misturados a partir da abertura de uma válvula na conexão que liga os dois recipientes. Considerando o volume da conexão desprezível e, ainda, o sistema mantido a uma temperatura constante, a pressão final da mistura de gases é, aproximadamente, A) 1,03 atm B) 1,90 atm C) 2,06 atm D) 2,80 atm 10. (FAMERP 2021) Recipientes conectados A figura ilustra uma montagem experimental composta por três recipientes contendo gases puros à mesma temperatura e separados por válvulas. Em determinado instante as válvulas são abertas, permitindo que as moléculas gasosas possam se difundir pelos recipientes até que seja atingido o equilíbrio. A temperatura permanece constante durante todo o processo. a) Classifique o sistema quanto ao número de fases após a abertura das válvulas. Considerando que a velocidade de difusão é inversamente proporcional à raiz quadrada da densidade dos gases, qual dos gases deve se difundir com a maior velocidade? b) Calcule a pressão parcial do gás carbônico na mistura após o equilíbrio. Organize os gases em ordem crescente de número de moléculas existentes no sistema. 11. (CEFET/CE 2003) Num reator isotérmico de volume constante V, são adicionados n mols de NOBr(g). Se a pressão inicial é p0 e a conversão (a) do NOBr é 100%, de acordo com a equação química não balanceada abaixo, podemos afirmar que a relação entre a pressão final (p) e a pressão inicial (p0) é igual a: (g = gás) NOBr(g) à NO(g) + Br2(g) A) 1 B) 1,5 C) 2 D) 2,5 E) 3 Prof. Pedro Madeira 12. (CEFET/CE 2003) Um balão meteorológico esférico tem um raio de 1m, quando está no nível do mar a 20ºC, e se expande para um raio de 3m, quando elevado a sua altitude máxima na temperatura de –20ºC. Admitindo comportamento ideal dos gases no balão, podemos afirmar que a pressão, medida em atm, dentro do balão, na sua altitude máxima, é igual a: A) 1,2 x 10–2 B) 2,2 x 10–2 C) 3,2 x10–2 D) 4,2 x 10–2 E) 5,2 x10–2 13. (CEFET/CE 2004) Considere dois recipientes de volumes iguais, A e B, contendo gases ideais onde a razão entre os números de moléculas nA/nB é igual a 3, e a razão entre as temperaturasabsolutas TB/TA é igual a 2. Nestas condições, podemos afirmar corretamente que a razão entre as pressões pA / pB é igual a: A) 1 B) 2 C) 7/2 D) 3 E) 3/2 14. (CEFET/CE 2004) Um recipiente de 11L contém 20g de neônio e certa massa de hidrogênio. A densidade da mistura (Ne + H2) é igual a 0,002 g/cm3 a 0ºC. A pressão exercida, em atm, pela mistura é igual a: A) 1,1 B) 2,1 C) 4,1 D) 6,1 E) 8,1 PROF. PEDRO MADEIRA 15. (CEFET/CE 2005) Um gás natural tem a seguinte composição (em percentagem molar): CH4 (metano) 80% C2H6 (etano) 20% Pede-se: a) A composição percentual em volume. b) A composição percentual em massa. c) O número de metros cúbicos que serão ocupados por 13,9 kg da mistura nas CNTP. 16. (CEFET/CE 2006) Sobre as leis das transformações gasosas, pede-se: a) Esboce a Lei de Boyle num diagrama p versus 1/V e determine o coeficiente angular da reta obtida. b) Esboce a lei das isóbaras (Charles-Gay Lussac) num diagrama V versus T e determine o coeficiente angular da reta obtida . 17. (UNESP 2021) metal + ácido = sal + H2 Um professor de química fez uma demonstração em que produziu hidrogênio gasoso pela reação, a quente, de ácido clorídrico com 6,75g de alumínio sólido sob forma de folhas amassadas. A equação que representa essa reação é: Al(s) + 3HCl(aq) → AlCl3(aq) + 3/2 H2(g) Adote para a constante universal do gases o valor R = 8 J/(mol.K). Supondo que todo o gás hidrogênio produzido 21 APOSTILA 01 DE FÍSICO-QUÍMICA – PROF. PEDRO MADEIRA (2022) nessa reação seja armazenado a uma temperatura constante de 27ºC em um recipiente rígido de volume 10 L, a quantidade de hidrogênio produzida nessas condições ficaria submetida a uma pressão de a) 6x104 N/m2. b) 8x104 N/m2. c) 5x104 N/m2. d) 9x104 N/m2. e) 4x104 N/m2. 18. (UNICAMP 2021) Adsorção de gases Um dos grandes desafios para a consolidação de uso do hidrogênio como combustível é seu armazenamento seguro e em grande quantidade. O hidrogênio pode ser armazenado puro, como gás ou líquido. Atualmente, parece mais adequado armazenar o hidrogênio na forma de hidretos metálicos ou adsorvido em materiais porosos nanoestruturados. Para que o armazenamento seja considerado eficiente, o material deve apresentar capacidade de armazenamento máxima em pressão constante e boa reversibilidade; ou seja, o armazenamento (adsorção) e a liberação (dessorção) devem ocorrer em condições similares. Essas características do armazenamento podem ser observadas em um gráfico denominado “isoterma de adsorção”, que é uma curva de composição de hidrogênio no material (C, kg de H2 / kg de material) em função da pressão. a) A figura mostra a isoterma de três materiais que poderiam ser empregados para armazenar H2. Qual curva (A, B ou C) representa o melhor material para se armazenar o hidrogênio? Justifique sua escolha. b) Um carro com motor a combustão interna consome 24 kg de gasolina (d=700 kg/m3) ou 8 kg de hidrogênio para percorrer uma distância de 400 km, adsorvido em um material intermetálico do tipo Mg2Ni. Considerando que a massa e o volume de um carro médio são aproximadamente de 6m3 e 1000 kg, respectivamente, uma possível desvantagem desta tecnologia alternativa estaria relacionada à massa ou ao volume relativamente ocupado pelo Mg2Ni? Justifique. Dados do Mg2Ni: capacidade de armazenamento de H2 = 3,6 kg de H2 por 100kg de Mg2Ni; densidade = 3400 kg/m3. 19. (UNICAMP 1996) O gás carbônico, CO2, é pouco solúvel em água. Esse processo de dissolução pode ser representado pela equação: CO2(g) + H2O(l) = HCO3 –(aq) + H+(aq) Essa dissolução é muito aumentada quando se adiciona NaOH na água. Para se determinar a quantidade de CO2 em uma mistura desse gás com o gás nobre neônio, foi realizado um experimento. O esquema abaixo mostra o experimento e o resultado observado. A proveta está graduada em mililitros (mL). Sabendo que não houve variação de temperatura durante o experimento e considerando desprezíveis a solubilidade do gás neônio em água e a pressão de vapor da água nessas condições: a) como a presença de NaOH aumenta a dissolução do gás carbônico na água? b) calcule a pressão parcial do CO2 na mistura inicial, sabendo que a pressão ambiente é de 90kPa (quilopascal). 20. (UNICAMP 2020) O CO2 dissolvido em bebidas carbonatadas, como refrigerantes e cervejas, é o responsável pela formação da espuma nessas bebidas e pelo aumento da pressão interna das garrafas, tornando-a superior à pressão atmosférica. O volume de gás no “pescoço” de uma garrafa com uma bebida carbonatada a 7ºC é igual a 24 mL, e a pressão no interior da garrafa é de 2,8x105 Pa. Trate o gás do “pescoço” da garrafa como um gás perfeito. Considere que a constante universal dos gases é de aproximadamente 8 J/mol.K. Calcule o número de moles de gás no “pescoço” da garrafa é igual a a) 1,2x105 b) 3,0x103 c) 1,2x10–1 d) 3,0x10–3 21. (UNICAMP 1997) Observe o air bag Com a intenção de proteger o motorista e o passageiro de lesões corporais mais graves, em muitos países já é obrigatório, em automóveis, o dispositivo chamado de “air bag”. Em caso de acidente um microprocessador desencadeia uma série de reações químicas que 22 APOSTILA 01 DE FÍSICO-QUÍMICA – PROF. PEDRO MADEIRA (2022) liberam uma certa quantidade de nitrogênio, N2(g), que infla rapidamente um balão plástico situado à frente dos ocupantes do automóvel. As reações químicas que ocorrem nesse processo estão representadas pelas seguintes equações: (Equação 1) 2 NaN3(s) = 2 Na(s) + 3 N2(g) (Equação 2) 10 Na(s) + 2 KNO3(s) = 5 Na2O(s) + K2O(s) + N2(g) (Equação 3) K2O(s) + Na2O(s) + SiO2(s) = silicato alcalino (vidro) No caso de acionamento do sistema de segurança descrito, supondo que o volume do saco plástico, quando totalmente inflado, seja de 70 litros e que, inicialmente, houvesse 2,0 moles de NaN3 e 2,0 moles de KNO3: a) Qual será a pressão do gás (em kPa), dentro do balão, quando este estiver totalmente inflado? Considere a temperatura como sendo 27oC. b) Supondo-se que o processo envolvesse apenas as reações representadas pelas equações 1 e 2, qual seria a massa total de substâncias sólidas restantes no sistema? 22. (UNICAMP 1997) Recipientes conectados O esquema abaixo representa um dispositivo para se estudar o comportamento de um gás ideal. Inicialmente, no frasco 1, é colocado um gás à pressão de 1 atmosfera, ficando sob vácuo os frascos 2 e 3 . Abre-se, em seguida, a torneira entre os frascos 1 e 2 até que se estabeleça o equilíbrio. Fecha-se, então, esta torneira e abre-se a torneira entre os frascos 1 e 3. O volume do frasco 1 é 9 vezes maior do que o do frasco 2 e o do 3 é 9 vezes maior que o do 1. a) Feito o procedimento acima descrito, em que frasco haverá menor quantidade de moléculas do gás? Justifique. b) Sendo p2 a pressão final no frasco 2 e p3 a pressão final no frasco 3 qual será o valor da relação p2/p3, ao final do experimento? Justifique. Observação: Desprezar o volume dos tubos das conexões. 23. (UNICAMP 1998) Os polímeros são formados pela união de um grande número de unidades básicas, denominadas monômeros. Um dos polímeros mais utilizados é o polietileno, que é produzido a partir da reação de polimerização do gás etileno, que se pode indicar como: j CH2=CH2(g) à (-CH2-CH2-)j(s) sendo j um valor médio. Para a fabricação de um balde, foram utilizados 280 g de polietileno com j = 10.000. a) Calcule o volume de etileno, a 25oC e 1 bar, necessário para produzir o referido balde. Considere que o gás seja ideal. b) Se um balde de mesma massa e praticamente de mesmo tamanho fosse produzido a partir de polietileno com j = 20.000, o volume de etileno utilizado seria maior? Justifique sua resposta. 24. (UNICAMP 2000) Hidróxido absorveCO2 A Apolo 13, uma nave tripulada, não pôde concluir sua missão de pousar no solo lunar devido a uma explosão num tanque de oxigênio líquido. Esse fato desencadeou uma série de problemas que necessitaram de soluções rápidas e criativas. Um desses problemas foi o acúmulo de gás carbônico no módulo espacial. Para reduzir o teor desse gás na cabine da nave, foi improvisado um filtro com hidróxido de lítio que, por reação química, removia o gás carbônico formado. a) Escreva a equação química que justifica o uso do hidróxido de lítio como absorvedor desse gás. b) Qual seria a massa de hidróxido de lítio necessária para remover totalmente o gás carbônico presente, considerando-o a uma pressão parcial igual a 2% da pressão ambiente total de 1,0 atm, estando a cabine à temperatura de 20ºC e supondo-se um volume interno de 60 m3? Prof. Pedro Madeira 25. (UNICAMP 2002) Observando o local do incêndio, nossos heróis perceberam que aquele não era o lugar ideal para guardar nem medicamentos, nem reagentes destinados ao laboratório de análises da empresa. Apesar disso, o local era considerado o mais seguro e, como também era refrigerado, fora o escolhido. Destruição geral! Como a explosão fora seguida de incêndio e de outras explosões, o teto e as janelas foram destruídos, e a chuva, apesar de ajudar a extinguir o fogo, também causou estragos. Examinando com cuidado o local, Rango observou várias garrafas e garrafões quebrados além de uma estante metálica caída e uma geladeira destruída... Preso aos cacos de um garrafão de 5 litros, pôde ler num rótulo: “Éter etílico”. O volume do almoxarifado foi estimado em 82 metros cúbicos. – E se o éter de 5 garrafões, contendo 4 kg de éter, cada um, houvesse se evaporado naquela sala?... – perguntou- se Rango. a) Considerando o conteúdo de cinco garrafões, qual a pressão parcial aproximada do éter (C4H10O) que evaporou no almoxarifado, supondo que ele tivesse se distribuído uniformemente e considerando as propriedades de gás ideal? b) Se apenas 10 % do vapor de éter tivesse se queimado, qual a energia liberada em joules? Dado: DH = – 2530 kJ.mol–1 (combustão do éter). Temperatura = 27oC. 26. (UNICAMP 2005) NO é vasodilatador O óxido nítrico (NO) é um gás que, produzido por uma célula, regula o funcionamento de outras células, configurando-se como um princípio sinalizador em sistemas biológicos. Essa descoberta não só conferiu o 23 APOSTILA 01 DE FÍSICO-QUÍMICA – PROF. PEDRO MADEIRA (2022) Prêmio Nobel de Medicina em 1998 para Ignaro, Furchgott e Murad, como também abriu as portas para muitos progressos científicos nesta área, inclusive no desenvolvimento do ViagraÒ. Como fármaco, a produção do NO começa com a reação entre SO2, ácido nítrico e água, originando, além desse gás, o ácido sulfúrico. Como produto final, o NO é comercializado em cilindros de 16 litros, diluído em N2. A concentração máxima é de 0,08 % em massa. Este cilindro chega a fornecer cerca de 2400 litros de gás a 25ºC e 1 atmosfera. a) Escreva a equação química da reação de produção do NO. b) Qual é a massa aproximada de NO contida no cilindro a que se refere o texto da questão? 27. (UNICAMP 2006) Algumas misturas gasosas podem ser importantes em ambientes hospitalares, assim como na prática de esportes, como mergulho autônomo a grandes profundidades. Uma dessas misturas, denominada Trimix, contém 16% de oxigênio, 24% de hélio e 60% de nitrogênio (porcentagem em volume). Suponha um cilindro de Trimix mantido à temperatura ambiente e a uma pressão de 9000 kPa. a) Escreva as fórmulas dos gases da mistura. b) Qual é a pressão parcial do hélio no cilindro? Mostre os cálculos. c) Qual é a massa molar média da mistura? Mostre os cálculos. PROF. PEDRO MADEIRA 28. (UNICAMP 2000) Uma questão de lógica Recentemente a Prefeitura de São Paulo ameaçava fechar as portas de um centro comercial por causa do excesso de gás metano em seu subsolo. O empreendimento foi construído nos anos 1980 sobre um lixão e, segundo a CETESB, o gás metano poderia subir à superfície e, eventualmente, causar explosões. a) Uma propriedade que garante a ascensão do metano na atmosfera é a sua densidade. Considerando que os gases se comportam como ideais, e que a massa molar média do ar atmosférico é de 28,8 g mol–1, justifique esse comportamento do metano em relação ao ar atmosférico. b) Na época do acontecimento, veiculou-se na imprensa que, “numa mistura com o ar, se o metano se encontra dentro de um determinado percentual (5% a 15% em volume quando em ar ambiente com 21% de oxigênio) e existe uma faísca ou iniciador, a explosão irá ocorrer”. Partindo-se do ar atmosférico e de metano gasoso, seria possível obter a mistura com a composição acima mencionada, pela simples mistura desses gases? Justifique. 29. (UNICAMP 2013) Decomposição do H2O2 é exo Na década de 1960, desenvolveu-se um foguete individual denominado “Bell Rocket Belt", que fez grande sucesso na abertura das Olimpíadas de 1984. Simplificadamente, esse foguete funciona à base da decomposição de peróxido de hidrogênio contido no compartimento 2, onde ele é estável. Abrindo-se a válvula 3, o peróxido de hidrogênio passa para o compartimento 4, onde há um catalisador. Nesse compartimento, o peróxido se decompõe muito rapidamente, de acordo com a equação abaixo: H2O2(l) à H2O(g) + ½ O2(g) DH = – 54 kJ.mol–1 Com base nessas informações, responda: a) No funcionamento do dispositivo há liberação ou absorção de energia? Justifique. b) Considerando a decomposição total de 68 quilogramas de peróxido de hidrogênio contidos no dispositivo, quantos metros cúbicos de gases são produzidos? Leve em conta que nas condições de uso do dispositivo o volume molar gasoso é de 0,075 m3 mol–1. 30. (FUVEST 2000) Observe o Heliox Os humanos estão acostumados a respirar ar com pressão parcial de O2 próxima de 2,1x104 Pa, que corresponde, no ar, a uma porcentagem (em volume) desse gás igual a 21%. No entanto, podem se adaptar a uma pressão parcial de O2 na faixa de (1 a 6)x104 Pa, mas não conseguem sobreviver se forçados a respirar O2 fora desses limites. a) Um piloto de uma aeronave, em uma cabine não pressurizada, voando a uma altitude de 12 km, onde a pressão atmosférica é de 2,2 x 104 Pa, poderá sobreviver se a cabine for alimentada por O2 puro? Explique. b) Um mergulhador no mar, a uma profundidade de 40 m, está sujeito a uma pressão cinco vezes maior do que na superfície. Para que possa sobreviver, ele deve respirar uma mistura de gás He com O2, em proporção adequada. Qual deve ser a porcentagem de O2, nessa mistura, para que o mergulhador respire um "ar" com a mesma pressão parcial de O2 existente no ar da superfície, ou seja, 2,1 x 104 Pa? Justifique. Obs.: O He substitui com vantagem o N2. 31. (FUVEST 2004) Para demonstrar a combustão de substâncias em oxigênio puro, este gás pode ser gerado a partir de água sanitária e água oxigenada, que contêm, respectivamente, hipoclorito de sódio e peróxido de 24 APOSTILA 01 DE FÍSICO-QUÍMICA – PROF. PEDRO MADEIRA (2022) hidrogênio. A reação que ocorre pode ser representada por NaClO + H2O2 à NaCl + H2O + O2(g) É assim que, num frasco, coloca-se certo volume de água oxigenada e acrescenta-se, aos poucos, certo volume de água sanitária. Observa-se forte efervescência. Ao final da adição, tampa-se o frasco com um pedaço de papelão. Em seguida, palha de aço, presa a um fio de cobre, é aquecida em uma chama até ficar em brasa. O frasco com oxigênio é destampado e, rapidamente, a palha de aço rubra é nele inserida. Então, observa-se luminosidade branca intensa, com partículas de ferro incandescentes espalhando-se pelo frasco. a) Calcule o volume de água sanitária quando se usa, no experimento, um frasco de volume adequado, sabendo-se que deve ser gerado, nas condições ambiente, um volume de 500mL de oxigênio, volume este suficiente para expulsar o ar e preencher o frasco. b) Explique por que, ao ar atmosférico, o ferro fica apenas vermelho rubro, mas queima rapidamente, quando exposto a oxigênio puro. Dados: volume molar do oxigênio nas condições ambiente........25,0 L/mol densidade da água sanitária..............................1,0 g/mL composição da água sanitária: 2,13 g de Cl, na forma de hipoclorito, em 100 g de solução aquosa. Prof. Pedro Madeira 32. (FUVEST 2008) Foram misturados 2,00 L de um alcano de m átomos de carbono por molécula e 2,00 L de outro alcano de n átomos de carbono por molécula, ambos gasosos. Esses alcanos podem ser quaisquer dois dentre os seguintes: metano, etano, propano ou butano. Na combustão completa dessa mistura gasosa, foram consumidos 23,00 L de oxigênio. Todos os volumes foram medidos nas mesmas condições de pressão e temperatura. a) Escreva a equação da combustão completa de um alcano de n átomos de carbono por molécula. Para identificar os dois alcanos que foram misturados, conforme indicado acima, é preciso considerar a lei de Avogadro, que relaciona o volume de um gás com seu número de moléculas. b) Escreva o enunciado dessa lei. c) Identifique os dois alcanos. Explique como chegou a essa conclusão. 33. (FUVEST 2012) A um recipiente, contendo solução aquosa de ácido sulfúrico, foi adicionada uma massa m de carbonato de sódio. Imediatamente após a adição desse sal, foi adaptado, à boca do recipiente, um cilindro de raio r, no interior do qual um êmbolo, de massa desprezível, pode se deslocar sem atrito. Após algum tempo, o carbonato de sódio foi totalmente consumido, e o gás liberado moveu o êmbolo para cima. Nessa transformação, o ácido sulfúrico era o reagente em excesso. a) Escreva a equação química balanceada que representa a transformação que ocorreu dentro do recipiente. b) O experimento descrito foi repetido utilizando-se carbonato de potássio em lugar de carbonato de sódio. A massa de carbonato de potássio utilizada nesse segundo experimento também foi m. A altura atingida pelo êmbolo foi a mesma nos dois experimentos? Explique. (Considere desprezível a variação de temperatura no sistema). c) Escreva a expressão matemática que relaciona a altura x, atingida pelo êmbolo, com a massa m de carbonato de sódio. Para isso, considere que - a solubilidade do gás, na solução, é desprezível, e não há perda de gás para a atmosfera; - nas condições do experimento, o gás formado se comporta como um gás ideal, cujo volume é dado por V = nRT/P, em que: P = pressão do gás n = quantidade de matéria do gás (em mol) R = constante universal dos gases T = temperatura do gás (em K) Observação: Use a abreviatura MM para representar a massa molar do carbonato de sódio. 34. (FUVEST 2019) Como o milho vira pipoca Um grão de milho de pipoca, visto a olho nu, apresenta duas regiões distintas, representadas por A e B na figura. Em A, ocorre o tecido acumulador de amido, usado, pela planta, para nutrir o embrião. Em B, os tecidos vegetais possuem maior teor de água. Ao ser aquecida, parte da água transforma-se em vapor, aumentando a pressão interna do grão. Quando a temperatura atinge 177ºC, a pressão se torna suficiente para romper o grão, que vira uma pipoca. 25 APOSTILA 01 DE FÍSICO-QUÍMICA – PROF. PEDRO MADEIRA (2022) Um estudo feito por um grupo de pesquisadores determinou que o interior do grão tem 4,5 mg de água da qual, no momento imediatamente anterior ao seu rompimento, apenas 9% está na fase vapor, atuando como um gás ideal e ocupando 0,1 mL. Dessa forma, foi possível calcular a pressão Pfinal no momento imediatamente anterior ao rompimento do grão. A associação correta entre região do milho e Pfinal é dada por: a) A = endosperma e Pfinal = 8,3 atm. b) B = endosperma e Pfinal = 5,9 atm. c) A = xilema e Pfinal = 22,1 atm. d) B = xilema e Pfinal = 5,9 atm. e) B = endosperma e Pfinal = 92,0 atm. 35. (UNICAMP 2017) Bebidas gaseificadas apresentam o inconveniente de perderem a graça depois de abertas. A pressão do CO2 no interior de uma garrafa de refrigerante, antes de ser aberta, gira em torno de 3,5 atm, e é sabido que, depois de aberta, ele não apresenta as mesmas características iniciais. Considere uma garrafa de refrigerante de litros, sendo aberta e fechada a cada horas, retirando- se de seu interior 250 mL de refrigerante de cada vez. Nessas condições, pode-se afirmar corretamente que, dos gráficos a seguir, o que mais se aproxima do comportamento da pressão dentro da garrafa, em função do tempo é o A) B) C) D) 36. (UECE 2017) Coleta de gás sobre água No laboratório de química, onde é comum recolher-se um gás pelo deslocamento de água, foram coletados 400 mL de gás oxigênio a 25ºC e 1atm de pressão. Sabendo-se que a pressão de vapor da água na mesma temperatura é 0,03 atm, é correto afirmar que o volume de oxigênio seco obtido nas mesmas condições de temperatura e pressão é a) 328,0 mL b) 388,0 mL c) 368,0 mL d) 354,0 mL 37. (UECE 2016) Catalisador do processo Haber Usado como catalisador no processo Haber, como agente de contraste em ressonância magnética e em camada protetora de aço contra ferrugem, o óxido ferroso-férrico é obtido pela reação entre o ferro metálico e o vapor d’água que produz também hidrogênio molecular. Ao fazer reagir 840g de ferro metálico, obtém- se um volume de hidrogênio medido a 127ºC e 5 atm de pressão correspondente a A) 87,46 L. B) 131,20 L. C) 57,06 L. D) 43,73 L. 2 4 26 APOSTILA 01 DE FÍSICO-QUÍMICA – PROF. PEDRO MADEIRA (2022) 38. (UFJF 2015) A lei dos gases ideais pode ser utilizada para determinar a massa molar de uma substância. Sabendo-se que a densidade (d) do enxofre na forma gasosa, na temperatura de 500ºC e pressão de 0,888 atm, é 3,710 g/L, é CORRETO dizer que a fórmula da molécula de enxofre nessas condições é: A) S2 B) S4 C) S6 D) S8 E) S9 39. (UEM 2015) Uma garrafa metálica aberta, de volume interno de (um) litro, é colocada em um sistema onde sua temperatura pode ser alterada (aquecida ou resfriada), sem que seu volume interno se altere. Assinale a(s) afirmação(ões) correta(s) sobre esse sistema, inicialmente colocado na temperatura de 27ºC, nos experimentos descritos abaixo, realizados ao nível do mar, onde a pressão atmosférica é 1,0 atm, ou na cidade de La Paz, na Bolívia, onde a pressão atmosférica é de 0,75 atm. 01) Tanto ao nível do mar como em La Paz, constatou- se que a quantidade de gás dentro da garrafa diminui com o seu aquecimento. 02) Ao se aquecer a garrafa ao nível do mar até 127ºC, a quantidade de ar dentro da garrafa será aproximadamente igual à quantidade de ar dentro da garrafa colocada em La Paz na temperatura de 27ºC. 04) Tanto ao nível do mar como em La Paz, ao se aquecer a garrafa até a temperatura de 250ºC, tampá-la e resfriá-la à temperatura ambiente, a pressão do gás no interior da garrafa será menor do que a pressão atmosférica. 08) Na temperatura de 0oC, o número de moléculas de ar no interior da garrafa colocada ao nível do mar ou colocada na cidade de La Paz é idêntico. 16) O número de moléculas de ar dentro da garrafa a 27ºC dividido pelo número de moléculas de ar dentro da garrafa a 227ºC não será o mesmo para experimentos feitos ao nível do mar ou em La Paz. Prof. Pedro Madeira 40. (UFG 2014) Gases sobre água Em um experimento, 90cm3 de um gás são injetados em uma proveta submersa, de modo que o nível do gás em seu interior tenha a mesma altura que a água da cuba, conforme esquema apresentado a seguir. O experimento ocorre a 29ºC. A massa do gás injetado é de 203 mg. Dados: Pressão de vapor da água a 29ºC = 30 mmHg Considerando o exposto, determine a massa molar do gás em questão e escreva a fórmulaestrutural plana de um dos isômeros do gás. 41. (UFG 2012) Considere o esquema apresentado a seguir, em que um experimento é executado do seguinte modo: um ovo cozido e sem casca, colocado sobre o bocal de uma garrafa à temperatura ambiente, não passa para seu interior em virtude de seu diâmetro ser levemente maior que o do bocal, conforme desenho A. Em seguida o ovo é retirado e a garrafa é aquecida à conforme desenho B. Com a garrafa ainda aquecida, o ovo é recolocado sobre o bocal da garrafa e, durante o processo de resfriamento da garrafa, ele passa para seu interior conforme desenho C. Explique o fenômeno que ocorre no experimento descrito e justifique por que o ovo, após o resfriamento, passa pelo bocal da garrafa. 42. (UESPI 2012) Observe o Heliox Uma criança com severa infecção nos brônquios apresenta problemas respiratórios, e o médico administra “heliox”, uma mistura de oxigênio e hélio com 90,0% em massa de O2. Se a pressão atmosférica é igual a 1 atm, calcule a pressão parcial de oxigênio que foi administrada à criança. a) 0,53 atm b) 0,60 atm c) 0,69 atm d) 0,75 atm e) 0,82 atm 43. (UEM 2012) Para as situações (I) e (II) expressas abaixo, na mesma altitude, e o dado fornecido a seguir, considerando uma bexiga de borracha deformável e de massa desprezível, hermeticamente fechada, contendo 2,0 g de gás hélio (supondo que seja um gás ideal), inicialmente a 25°C, que pode explodir quando atingido o dobro de sua capacidade volumιtrica inicial, assinale o que for correto. Situações: 1 60 °C, 27 APOSTILA 01 DE FÍSICO-QUÍMICA – PROF. PEDRO MADEIRA (2022) I. A bexiga permanece em repouso sobre um piso plano e horizontal, cuja área de contato entre a bexiga e o piso é 1,0 cm2 e a pressão no interior da bexiga é de 2,0 atm. II. Com a situação descrita em (I), é colocado sobre a bexiga um corpo de massa M. A área de contato entre a bexiga e o piso se torna igual a 10 cm2 e é exatamente igual à área de contato entre o corpo e a bexiga. Considere que a face do corpo de massa M que toca a bexiga é plana e possui área sempre maior do que a área de contato entre o corpo e a bexiga. 01) Na situação II, seria possível calcular a massa M do corpo, se soubéssemos também a pressão interna na bexiga e a pressão atmosférica (ambiente). 02) Ao aumentar-se a temperatura do sistema na situação I para 51ºC, a bexiga irá explodir. 04) Ao colocar-se o corpo de massa M sobre a bexiga, mantendo-se o sistema a 25ºC, sua pressão interior deverá aumentar em virtude do aumento do volume do gás. 08) O volume ocupado pelo gás hélio na situação I é, aproximadamente, de 6,1 litros. 16) Na situação II, a pressão exercida pelo sistema corpo+bexiga sobre o piso é dependente da pressão atmosférica no local do experimento. 44. (UEM 2011) O manômetro é um aparelho que serve para medir a pressão de um gás. Ele consiste em um tubo em forma de U, aberto nas duas extremidades, contendo em seu interior Hg, conforme ilustrado na figura abaixo. Uma das extremidades está conectada à válvula de saída de gás do botijão. Com relação ao funcionamento desse manômetro, analise as alternativas abaixo e assinale o que for correto. 01) Se a pressão atmosférica no local da medida é 710 mmHg, tem-se que a pressão do gás é 2.610 mmHg. 02) Se essa medida fosse realizada ao nível do mar e no topo do monte Everest e, em ambos os casos, a uma temperatura de -5 °C, os valores encontrados para a pressão do gás seriam diferentes. 04) Se, nas condições normais de temperatura e pressão (CNTP), substituíssemos o Hg por água, os valores da altura da coluna no tubo em U seriam diferentes, pois a água possui densidade menor que a do Hg. 08) A elevação ou diminuição da temperatura de todo o sistema (botijão e manômetro) não alterará a diferença entre os níveis do Hg no tubo. 16) Supondo que a massa de 13 kg de gás butano contido no botijão fosse substituída por 13 kg de gás hidrogênio na mesma temperatura, não haveria alteração entre os níveis de Hg no tubo. 45. (UNICAMP 2019) Episódios recentes de erupções vulcânicas têm trazido consequências trágicas para a sociedade e para o meio ambiente. Ativo desde 1983, o Vulcão Kilauea apresentou, em 2018, a sua maior erupção já registrada. Quase ao mesmo tempo, foi a vez do Vulcão Fuego da Guatemala mostrar sua força. No Kilauea não houve explosões, ao contrário do que ocorreu no Fuego. Os especialistas afirmam que a ocorrência de uma erupção explosiva depende da concentração e do tipo de gases dissolvidos no magma, como SO2, HF e HCl, além de vapor de água e CO2 aprisionados. A figura a seguir dá informações sobre a relação entre quantidades (em mol) de SO2, HF e HCl no magma de três vulcões distintos. De acordo com a figura, em relação às quantidades de gases dissolvidos no magma, é correto afirmar que as concentrações de SO2 são maiores que as de HF e de HCl. A) nos três vulcões e, neles, HF e HCl são aproximadamente iguais. B) em apenas dois vulcões e, neles, HF e HCl são aproximadamente iguais. C) nos três vulcões, mas em apenas dois deles HF e HCl são aproximadamente iguais. D) em apenas dois vulcões, mas nos três vulcões HF e HCl são aproximadamente iguais. 28 APOSTILA 01 DE FÍSICO-QUÍMICA – PROF. PEDRO MADEIRA (2022) 46. (UFRGS 2018) Decomposição do HNO3 A decomposição térmica do ácido nítrico na presença de luz libera NO2 de acordo com a seguinte reação (não balanceada). HNO3(aq) → H2O(ℓ) + NO2(g) + O2(g) Assinale a alternativa que apresenta o volume de gás liberado, nas CNTP, quando 6,3g de HNO3 são decompostos termicamente. A) 2,24 L B) 2,80 L C) 4,48 L D) 6,30 L E) 22,4 L 47. (UFSC 2018) O verão é a estação na qual, ao menos em países de clima tropical e subtropical, faz-se uso significativo de condicionadores de ar para ampliar o “conforto térmico” em ambientes fechados. Você sabe como funciona um condicionador de ar? O sistema é baseado em ciclos de compressão e expansão de um gás refrigerante, tipicamente formado por substâncias como CHClF2 e CHF3, que flui por um sistema fechado. A representação esquemática abaixo ilustra simplificadamente o processo. Disponível em: <https://cen.acs.org/articles/95/i33/Periodic-graphics- chemistry-air-conditioning.html>. [Adaptado]. Acesso em: 19 ago. 2017. Com base no exposto acima, é correto afirmar que: 01) no compressor, representado na etapa 1, o aumento da pressão sobre o gás faz com que a temperatura diminua. 02) no processo de expansão, representado na etapa 3, o gás refrigerante tem sua temperatura reduzida. 04) no condensador, representado na etapa 2, o gás refrigerante no estado gasoso é convertido em um sólido. 08) a variação de temperatura que ocorre durante a expansão (etapa 3) independe do volume do dispositivo no qual a expansão é induzida. 16) os processos de expansão e compressão dependem do vapor de água no sistema, já que o gás refrigerante é um composto iônico gasoso e, portanto, não está sujeito a variações de volume. 32) as variações de pressão que ocorrem nos processos de expansão e compressão dependem da quantidade de gás refrigerante no sistema. 48. (UNICAMP 2017) Fermento em pó químico Um teste caseiro para saber se um fermento químico ainda se apresenta em condições de bom uso consiste em introduzir uma amostra sólida desse fermento em um pouco de água e observar o que acontece. Se o fermento estiver bom, ocorre uma boa efervescência; caso contrário, ele está ruim. Considere uma mistura sólida que contém os íons dihidrogenofosfato, H2PO4 – e hidrogenocarbonato, HCO3 –. a) Considerando que o teste descrito anteriormente indica que a mistura sólida pode ser de um fermento que está bom, escreva a equação química que justifica esse resultado. b) Tendo em vista quea embalagem do produto informa que 18 g desse fermento químico devem liberar, no mínimo, 1,45x10–3 m3 de gases a 298K e 93.000 Pa determine a mínima massa de hidrogenocarbonato de sódio que o fabricante deve colocar em 18 gramas do produto. 49. (UCS 2021) Os Boranos são muito loucos A figura abaixo ilustra uma série de boranos, ou seja, compostos químicos formados exclusivamente por boro e hidrogênio. Suponha que uma amostra de 1,0g de um desses compostos ocupe um volume de 820 cm3, a 357ºC e 1 atm. Com base nessas informações, é possível concluir que o composto em questão é o a) B4H10 b) B5H9 c) B4H11 d) B6H10 e) B10H14 29 APOSTILA 01 DE FÍSICO-QUÍMICA – PROF. PEDRO MADEIRA (2022) SEÇÃO ITA / IME 50. (ITA 1980 – Q13) Dois balões esféricos de mesmo volume são unidos por um tubo de volume desprezível, provido de torneira. Inicialmente o balão A contém 1,00 mol de um gás ideal e em B há vácuo. Os dois balões são mantidos às temperaturas indicados no desenho acima. A torneira é aberta durante certo tempo. Voltando a fechá-la, verifica-se que a pressão em B é 0,81 do valor da pressão em A. Quanto do gás deve ter sobrado no balão A? A ( ) 0,20 mol B ( ) 0,40 mol C ( ) 0,50 mol D ( ) 0,60 mol E ( ) 0,80 mol 51. (ITA 1981 – Q02) Considere a reação representada pela equação: Na(c) + 1/2 Cl2(g) à NaCl(c) executada em recipiente de 22,4 litros, empregando 1,0 mol de cada um dos reagentes. Desejam-se representar: I. as variações dos números de mols dos reagentes e do produto, do início ao fim da reação; e II. a variação da pressão gasosa dentro do recipiente, a 0oC, do início ao fim da reação. Dispõe-se dos seguintes gráficos onde a escala de tempo é arbitrária: A melhor combinação de variações a representar e gráficos representativos é: A ( ) I-a e II-c B ( ) I-a e II-d C ( ) I-a e II-e D ( ) I-b e II-d E ( ) I-b e II-e 52. (ITA 1981 – Q09) Um cilindro de gás de 50 litros contém N2 e O2 na proporção molar de 4 para 1; a pressão no interior do cilindro é de 2,0 atm e a temperatura é mantida a 31oC. Nessa temperatura injeta-se monóxido de carbono no cilindro até que a pressão suba para 2,5 atm. Sabe-se que não há reação entre os gases e que o volume do cilindro não se altera. É CORRETO afirmar-se que, após a injeção do monóxido de carbono: A ( ) o número de mols de O2 é 1,0 B ( ) os números de mols de O2 e de CO são iguais C ( ) a concentração de O2 na mistura final é de 20% em volume D ( ) a concentração de CO na mistura final é de 25% em volume E ( ) a concentração de N2 na mistura final é de 64% em volume 53. (ITA 1983 – Q11) Num equipamento adequado para permitir adição de soluções, assim como coleta e medida de volume de gases, fez-se a seguinte experiência: Após colocar neste equipamento 100 cm3 de uma solução aquosa contendo 1,06 g de carbonato de sódio por litro de solução, adiciona-se um excesso de solução de ácido clorídrico. Admitindo que, nesta experiência, todo o gás que pudesse ser produzido pela reação entre as duas soluções foi de fato coletado, qual o volume medido, em cm3, sabendo-se que a experiência foi realizada na temperatura de 27oC e pressão de 750 mmHg? A ( ) 0,10 x 22,4 B ( ) 0,10 x 24,9 C ( ) 100 x 0,0821 D ( ) 0,20 x 62,3 E ( ) 0,40 x 62,3 54. (ITA 1984 – Q04) Num saco plástico de paredes flexíveis, constituído com filme de polietileno, são introduzidos 8,0 litros de hélio e 2,0 litros de metano, medidos nas condições normais de temperatura e pressão. Este saco está num laboratório onde a temperatura é 25oC e a pressão é 700 mmHg. Com relação a esta mistura, nas condições do laboratório, assinale a única afirmação FALSA: A ( ) A fração molar de metano na mistura é 0,20. B ( ) A concentração de metano na mistura é 20% (V). C ( ) A pressão parcial de metano na mistura é 140 mmHg. D ( ) A concentração de metano na mistura é 50% (massa). E ( ) A concentração de metano na mistura é 3,2 g de CH4/22,4 L mistura. Prof. Pedro Madeira 30 APOSTILA 01 DE FÍSICO-QUÍMICA – PROF. PEDRO MADEIRA (2022) 55. (ITA 1984 – Q11) No processo de Mond de purificação do níquel, o metal impuro é tratado com CO, segundo a equação: O volume de CO medido nas CNTP, necessário para purificar 0,500 g de níquel contendo 25% de impurezas inertes será A ( ) (22,4 x 0,25 x 0,500 / 58,7) litros. B ( ) (22,4 x 4 x 0,500 x 58,7) litros. C ( ) (22,4 x 4 x 0,500 / 0,75 x 58,7) litros. D ( ) (22,4 x 0,75 x 0,500 / 58,7) litros. E ( ) (22,4 x 4 x 1,5 x 58,7) litros. 56. (ITA 1985 – Q05) Todas as afirmações desta questão referem-se a 1,00 cm3 de hidrogênio líquido, cuja densidade vale 0,070 g.cm–3 a 13,4 K. Qual das afirmações abaixo é FALSA? A ( ) O líquido contém 0,035 mol de moléculas H2. B ( ) O líquido contém 2,1 x 1022 moléculas de H2. C ( ) O gás obtido por vaporização desse líquido ocupa o volume de 0,57 L a 25oC e pressão de 1,5 atmosferas. D ( ) O gás obtido por vaporização desse líquido apresenta densidade de 6,2 x 10–2 g.L–1 a 25oC e pressão de 1,5 atmosfera. E ( ) O gás obtido por vaporização desse líquido exercerá uma pressão de 8,6 atmosferas à temperatura de 25oC e volume de 0,10 L. PERGUNTA Calcule a densidade do hidrogênio gasoso a 25oC e pressão de 1,5 atmosferas. PROF. PEDRO MADEIRA 57. (ITA 1987 – Q18) Uma amostra de óxido de crômio (III) contaminada com impureza inerte é reduzida com hidrogênio de acordo com a seguinte equação: Cr2O3(c) + 3 H2(g) à 2 Cr(c) + 3 H2O(g) O volume de H2 medido nas CNTP, necessário para purificar 5,0 g de óxido de crômio (III) contendo 15% de impurezas inertes será: A ( ) ((0,15 x 5,0 x 3 x 22,4) / 152) litros. B ( ) ((0,85 x 5,0 x 3 x 22,4) / 152) litros. C ( ) ((0,15 x 5,0 x 3 x 22,4) / 104) litros. D ( ) ((0,85 x 5,0 x 3 x 22,4) / 104) litros. E ( ) ((0,15 x 5,0 x 3 x 22,4) / 104) litros. 58. (ITA 1988 – Q27) Num saco de plástico flexível e não permeável a gases, inicialmente vazio, são introduzidos sucessivamente, 50,0 cm3 de N2, 20,0 cm3 de O2 e 30,0 cm3 de CO2, todos medidos nas CNTP. Considere as afirmações seguintes, relativas às concentrações nesta solução gasosa mantida nas CNTP. I. A solução contém 50,0% de N2, 20,0% de O2 e 30,0% de CO2, todas estas porcentagens em volume. II. A solução contém 50,0% de N2, 20,0% de O2 e 30,0% de CO2, todas estas porcentagens em massa. III. As frações molares de N2, O2 e CO2 são, respectivamente, 0,500; 0,200 e 0,300. IV. A solução é 0,500/22,4 molar em N2; 0,200/22,4 molar em O2 e 0,300/22,4 molar em CO2. Destas afirmações estão corretas apenas: A ( ) I e III B ( ) I e IV C ( ) II e IV D ( ) I, III e IV E ( ) II, III e IV. 59. (ITA 1989 – Q30) Num garrafão de 3,5 L de capacidade, contendo 1,5 L de solução 1,0 molar de ácido sulfúrico, introduzem-se 32,7 g de aparas de zinco; fecha-se rapidamente com rolha de borracha. Supondo que a temperatura do ambiente onde esta perigosa experiência está sendo feita seja de 20oC, o incremento máximo de pressão interna (DP) do frasco será de A ( ) 0,41 atm B ( ) 3,4 atm C ( ) 5,6 atm D ( ) 6,0 atm E ( ) 12,0 atm. 60. (ITA 1990 – Q19) Considere os dois recipientes cilíndricos, 1 e 2, providos de manômetro e interligados por um tubo com torneira, de volume desprezível, conforme figura abaixo. O primeiro índice, nas grandezas abordadas, se refere ao recipiente 1 ou 2. O segundo índice, i ou f, refere-se respectivamente, ao que ocorre inicialmente, antes de abrir a torneira e ao que ocorre no estado final, depois de a torneira permanecer aberta muito tempo. Em face destas informações podemos afirmar que: A ( ) P1,f = . 2 3 /P2,f B ( ) n1,f = n2,f C ( ) n1,f = . 2 3 /n2,f D ( ) n1,f = . 6 3 / n2,i E ( ) P1,f= P2,f = . 2 3 /P2,i 61. (ITA 1991 – Q04) Um recipiente de aço de volume V1, contém ar comprimido na pressão P1. Um segundo recipiente de aço de volume V2, contém ar menos comprimido na pressão P2. Ambos os cilindros estão na pressão ambiente. Caso sejam interligados por uma tubulação de volume desprezível, a pressão final em ambos os cilindros será igual a: A ( ) (V1P1+V2P2)/(V1 + V2) B ( ) (V1P2+V2P1)/(V1 + V2) C ( ) (V1P1+V2P2)/(P1 + P2) D ( ) (V1P2+V2P1)/(P1 + P2) E ( ) (P1/V1 + P2/V2).V1V2 Ni CO Ni(CO)4 31 APOSTILA 01 DE FÍSICO-QUÍMICA – PROF. PEDRO MADEIRA (2022) 62. (ITA 1991 – Q11) No início de uma experiência a bureta do lado esquerdo da figura estava completamente cheia de água. Pela torneira da bureta foi admitido nitrogênio gasoso puro. Após a admissão do nitrogênio o nível de água na bureta ficou na mesma altura que no recipiente do lado direito da figura. A pressão ambiente externa era 760mmHg e a temperatura era 25oC. Nesta temperatura a pressão de vapor da água é 24 mmHg. Assinale a afirmação falsa a respeito do volume gasoso dentro da bureta após o estabelecimento do equilíbrio: A ( ) 736 / 760 das moléculas correspondem a N2. B ( ) A pressão total é 784 mm Hg. C ( ) A pressão parcial do vapor de água é 24 mm Hg. D ( ) A pressão parcial do nitrogênio é 736 mm Hg. E ( ) A fração molar das moléculas de água é 24 / 760. Prof. Pedro Madeira 63. (ITA 1991 – Q25) Considere o volume de 5,0 litros de uma mistura gasosa contendo 20% (V/V) do isótopo 40 do Argônio e 80% (V/V) do isótopo 20 de Neônio. Na temperatura de 273ºC a mistura exerce a pressão de 20 atm. A quantidade (em mol) de Argônio nesta mistura é: A ( ) B ( ) C ( ) D ( ) E ( ) 64. (ITA 1993 – Q03) Dois balões de vidro, A e B, de mesmo volume contêm ar úmido. Em ambos os balões a pressão e a temperatura são as mesmas, a única diferença sendo que no balão A, a umidade relativa do ar é de 70% enquanto que no balão B ela é de apenas 10%. Em relação ao conteúdo destes dois balões é ERRADO afirmar que: A ( ) Os dois balões contêm o mesmo número de moléculas. B ( ) Os dois balões contêm a mesma quantidade de gás, expressa em mol. C ( ) No balão B há maior massa de nitrogênio. D ( ) No balão A há maior massa total de gás. E ( ) A quantidade (mol) e a massa (grama) de vapor de água são maiores no balão A. 65. (ITA 1993 – Q15) 1,31 g de uma mistura de limalhas de cobre e zinco reagiu com excesso de solução de ácido clorídrico, numa aparelhagem adequada, produzindo gás hidrogênio. Este gás, depois de seco, ocupou um volume de 269 m! sob pressão de 0,90 atm e a 300 K (que corresponde a 1,10 x 273 K). A fração de massa do zinco nesta mistura é: A ( ) 0,13 B ( ) 0,25 C ( ) 0,50 D ( ) 0,75 E ( ) 1,00 66. (ITA 1994 – Q19) Por ocasião do jogo Brasil versus Bolívia, disputado em La Paz, um comentarista esportivo afirmou que: “Um dos maiores problemas que os jogadores da seleção brasileira de futebol terão de enfrentar é o fato de o teor de oxigênio no ar, em La Paz, ser cerca de 40% menor do que aquele ao nível do mar.” Lembrando que a concentração do oxigênio no ar, ao nível do mar, é aproximadamente 20% (v/v) e supondo que no dia em que o comentarista fez esta afirmação a pressão atmosférica, em La Paz, fosse igual a, aproximadamente, 450 mm Hg, qual das opções a seguir contém a afirmação que mais se aproxima daquilo que o comentarista poderia ter dito? A ( ) A concentração do oxigênio no ar é cerca de 12% (v/v). B ( ) A fração molar do oxigênio no ar é cerca de 0,12. C ( ) A pressão parcial do oxigênio no ar é aproximadamente expressa por (0,20 x 760 mmHg x 0,60). D ( ) A pressão parcial do oxigênio no ar é cerca de 152 mmHg. E ( ) A pressão parcial do oxigênio no ar é aproximadamente expressa por (0,20 x 760 mm Hg x 0,40). 67. (ITA 1994 – Q21) Um composto de Grignard é obtido a partir de 13,7 g de um brometo de alquila. Esse composto por hidrólise forma um hidrocarboneto que ocupa 2,5 L, medido a 32°C e pressão de 1,0 atmosfera. O hidrocarboneto pode ser: A ( ) Propano B ( ) Isobutano C ( ) 2-Metilbutano D ( ) Isoctano E ( ) 2,5-dimetil-hexano 68. (ITA 1995 – Q02) A concentração de O2 na atmosfera ao nível do mar é 20,9% em volume. Assinale a opção que contém a afirmação falsa. A ( ) Um litro de ar contém 0,209 ℓ de O2. B ( ) Um mol de ar contém 0,209 mol de O2. C ( ) Um volume molar de ar à CNTP contém 6,7 g de O2. D ( ) A concentração de O2 no ar é 20,9% em massa. E ( ) A concentração de O2 expressa como uma relação de volume ou uma relação de mol não se altera, se a temperatura ou a pressão são modificadas. 69. (ITA 1995 – Q14) Considere a queima completa de vapores das quatro seguintes substâncias: metano, etano, metanol, e etanol. Os volumes de ar necessários para queima de l litro de cada um destes vapores, todos à mesma pressão e temperatura, são respectivamente, V1, V2, V3 e V4. Assinale a alternativa que apresenta a comparação correta entre os volumes de ar utilizados na combustão. A ( ) V2 > V4 > V1 > V3 B ( ) V2 > V1 > V4 > V3 C ( ) V4 > V2 > V3 > V1 D ( ) V4 > V3 > V2 > V1 E ( ) V4 = V2 > V3 = V1 22,4 2,0 22,4 10 22,4 20 22,4 50 22,4 100 760 mmHg 32 APOSTILA 01 DE FÍSICO-QUÍMICA – PROF. PEDRO MADEIRA (2022) 70. (ITA 1996 – Q24) Três recipientes iguais de 4 litros de capacidade, chamados de 1, 2 e 3, mantidos na mesma temperatura, contêm 180 ml de água. A cada um destes recipientes se junta, respectivamente, 0,10 mol de cada uma das seguintes substâncias: óxido de cálcio, cálcio metálico e hidreto de cálcio. Após a introdução do respectivo sólido, cada frasco é bem fechado. Atingido o equilíbrio e descartada a hipótese de ocorrência de explosão, a pressão final dentro de cada recipiente pode ser colocada na seguinte ordem: A ( ) p1 = p2 = p3 B ( ) p1 < p2 < p3 C ( ) p1 < p2 ≈ p3 D ( ) p1 ≈ p2 < p3 E ( ) p1 > p2 ≈ p3 71. (ITA 1996 – Q25) Considere um recipiente de paredes reforçadas (volume fixo) provido de torneira, manômetro e de um dispositivo para produção de faíscas análogo à “vela de ignição” em motores de automóveis. No fundo do recipiente também é colocado um dissecante granulado (p. ex. sílica gel). Neste recipiente, previamente evacuado, se introduz uma mistura de hidrogênio e nitrogênio gasosos até que a pressão dentro dele atinja o valor de 0,70 atm, a temperatura sendo mantida em 20°C. O problema é descobrir a proporção de H2 e N2 nesta mistura inicial. Para isso se junta excesso de O2 à mistura, já no recipiente, até que a pressão passe ao valor de 1,00 atm. Em seguida se faz saltar uma faísca através da mistura. Assim, a temperatura e a pressão sobem transitoriamente. Deixando a mistura voltar à temperatura de 20°C, notamos que o manômetro acusa uma pressão de 0,85 atm. (lembrar que a água formada é absorvida pelo dissecante, não exercendo pressão parcial significativa). Das informações podemos concluir que a fração molar do hidrogênio na mistura inicial de H2 e N2 era igual a: A ( ) 0,07 B ( ) 0,11 C ( ) 0,14 D ( ) 0,70 E ( ) 1,00 72. (ITA 1996 – P08) Uma garrafa de refrigerante, com capacidade de 2,0 litros, contém 1,0 litro de uma solução aquosa 0,30 molar de HCl e é mantida na temperatura de 25°C. Introduzindo um pedaço de zinco metálico nesta garrafa e fechando a tampa, a pressão no interior da garrafa irá aumentar gradualmente. A questão é calcular a massa (em gramas) de zinco a ser introduzida para que a pressão aumente de 1,0 para 2,0 atm, a temperatura sendo mantida em 25°C. Escreva a equação química balanceada da reação envolvida e indique os cálculos realizados. Para os cálculos, despreze tanto a pressão de vapor da solução quanto a solubilidade do gás formado.73. (ITA 1997 – Q10) Três recipientes fechados, providos de êmbolos móveis, contêm a mesma quantidade (mol) do único gás especificado: N2 no recipiente 1; CO no recipiente 2 e CO2 no recipiente 3. Considerando a temperatura medida em kelvin e a pressão em atm, são feitas as afirmações: I. Se a pressão e a temperatura forem as mesmas, as massas específicas dos gases nos recipientes 1 e 2 serão praticamente iguais. II. Se a pressão e a temperatura forem as mesmas, as massas específicas dos gases nos recipientes 2 e 3 serão praticamente iguais. III. Se a temperatura for a mesma, mas a pressão no interior do recipiente 1 for o duplo da pressão no recipiente 2, a massa específica do gás no recipiente 1 será praticamente o duplo da massa específica do gás no recipiente 2. IV. Se a temperatura for a mesma, mas a pressão no interior do recipiente 3 for o duplo da pressão no recipiente 2, a massa específica do gás no recipiente 3 será maior do que o duplo da massa específica do gás no recipiente 2. V. Se a pressão for a mesma, mas a temperatura do recipiente 1 for o duplo da temperatura no recipiente 2, a massa específica do gás no recipiente 1 será praticamente o duplo da massa específica do gás no recipiente 2. Estão CORRETAS apenas: A ( ) I, III e IV B ( ) I e II C ( ) I e V D ( ) II e V E ( ) III e IV 74. (ITA 1998 – P02) Numa experiência de eletrólise da água formam-se 3,00g de H2(g). Calcule o volume ocupado por esta massa de hidrogênio, suposta isenta de umidade, na temperatura de 300K e sob a pressão de 684 mmHg (0,90 x 760 mmHg). 75. (ITA 2000 – Q07) Considere as afirmações abaixo relativas ao aquecimento de um mol de gás N2 contido em um cilindro provido de um pistão móvel sem atrito: I. A massa específica do gás permanece constante. II. A energia cinética média das moléculas aumenta. III. A massa do gás permanece a mesma. IV. O produto pressão x volume permanece constante. Das afirmações feitas, estão CORRETAS A ( ) apenas I, II e III B ( ) apenas I e IV C ( ) apenas II e III D ( ) apenas II, III e IV E ( ) todas. Prof. Pedro Madeira 76. (ITA 2001 – Q10) Um cilindro provido de um pistão móvel, sem atrito, contém um gás ideal. Qual dos gráficos abaixo representa, qualitativamente, o comportamento CORRETO do sistema quando a pressão (P) e/ou o volume (V) são modificados, sendo mantida constante a temperatura (T)? A ( ) B ( ) C ( ) 33 APOSTILA 01 DE FÍSICO-QUÍMICA – PROF. PEDRO MADEIRA (2022) D ( ) E ( ) 77. (ITA 2003 – Q16) Dois compartimentos, 1 e 2, têm volumes iguais e estão separados por uma membrana de paládio, permeável apenas à passagem de hidrogênio. Inicialmente, o compartimento 1 contém hidrogênio puro (gasoso) na pressão PH2, puro = 1 atm, enquanto que o compartimento 2 contém uma mistura de hidrogênio e nitrogênio, ambos no estado gasoso, com pressão total Pmist = (PH2 + PN2) = 1 atm. Após o equilíbrio termodinâmico entre os dois compartimentos ter sido atingido, é CORRETO afirmar que: A ( ) PH2, puro = 0 B ( ) PH2, puro = PN2, mist C ( ) PH2, puro = Pmist D ( ) PH2, puro = PH2, mist E ( ) Pcompartimento 2 = 2 atm 78. (ITA 2003 – Q23) Determine a massa específica do ar úmido, a 25oC e pressão de 1 atm, quando a umidade relativa do ar for igual a 60%. Nessa temperatura, a pressão de vapor saturante da água é igual a 23,8 mmHg. Assuma que o ar seco é constituído por N2(g) e O2(g) e que as concentrações dessas espécies no ar seco são iguais a 79 e 21% (v/v), respectivamente. 79. (ITA 2004 – Q16) Uma massa de 180 g de zinco metálico é adicionada a um erlenmeyer contendo solução aquosa de ácido clorídrico. Ocorre reação com liberação de gás que é totalmente coletado em um Balão A, de volume igual a 2 L. Terminada a reação, restam 49 g de zinco metálico no erlenmeyer. A seguir, por meio de um tubo provido de torneira, de volumes desprezíveis, o Balão A é conectado a um Balão B, de volume igual a 4 L, que contém gás nitrogênio sob pressão de 3 atm. Considere que a temperatura é igual em ambos os balões e que esta é mantida constante durante todo o experimento. Abrindo-se a torneira do tubo de conexão entre os dois balões, ocorre a mistura dos dois gases. Após estabelecido o equilíbrio, a pressão nos dois balões pode ser expressa em função da constante dos gases (R) e da temperatura absoluta (T) por A ( ) 1 2 RT B ( ) 1 3 RT + 2 C ( ) 3 2 RT B ( ) 1 3 RT + 2 E ( ) RT + 3 80. (ITA 2005 – Q15) A 25°C, uma mistura de metano e propano ocupa um volume (V), sob uma pressão total de 0,080 atm. Quando é realizada a combustão completa desta mistura e apenas dióxido de carbono é coletado, verifica- se que a pressão desse gás é de 0,12 atm, quando este ocupa o mesmo volume (V) e está sob a mesma temperatura da mistura original. Admitindo que os gases têm comportamento ideal, assinale a opção que contém o valor CORRETO da concentração, em fração em mols, do gás metano na mistura original. A ( ) 0,01 B ( ) 0,25 C ( ) 0,50 D ( ) 0,75 E ( ) 1,00 81. (ITA 2006 – Q14) Um recipiente fechado, mantido a volume e temperatura constantes, contém a espécie química X no estado gasoso a pressão inicial P0. Esta espécie decompõe-se em Y e Z de acordo com a seguinte equação química: X(g) à 2 Y(g) + 1/2 Z(g). Admita que X, Y e Z tenham comportamento de gases ideais. Assinale a opção que apresenta a expressão CORRETA da pressão (P) no interior do recipiente em função do andamento da reação, em termos da fração a de moléculas de X que reagiram. A ( ) P = [1 + (1/2) a] P0 B ( ) P = [1 + (2/2) a] P0 C ( ) P = [1 + (3/2) a] P0 D ( ) P = [1 + (4/2) a] P0 E ( ) P = [1 + (5/2) a] P0 82. (ITA 2008 – Q24) Em um laboratório, a 20°C e utilizando um sistema adequado, H2(g) foi obtido através da reação entre uma amostra de uma liga de 0,3 g de magnésio e um litro de uma solução aquosa 0,1 mol.L–1 em HCl. Um manômetro indicou que a pressão no interior do recipiente que contém o H2(g) era de 756,7 Torr. Sabendo-se que a pressão de vapor d’água a 20°C é 17,54 Torr e o volume de H2(g) obtido foi 0,200 L, determine a pureza da amostra da liga de magnésio (massa de magnésio x 100/massa total da amostra), considerando que somente o magnésio reaja com o HCl. 83. (ITA 2008 – Q27) Uma chapa de ferro é colocada dentro de um reservatório contendo solução aquosa de ácido clorídrico. Após um certo tempo observa-se a dissolução do ferro e formação de bolhas gasosas sobre a superfície metálica. Uma bolha gasosa, de massa constante e perfeitamente esférica, é formada sobre a superfície do metal a 2,0 metros de profundidade. Calcule: a) o volume máximo dessa bolha de gás que se expandiu até atingir a superfície do líquido, admitindo-se que a temperatura é mantida constante e igual a 25oC e que a base do reservatório está posicionada ao nível do mar. b) a massa de gás contida no volume em expansão da bolha. Sabe-se que no processo corrosivo que originou a formação da bolha de gás foram consumidos 3,0 x 1015 átomos de ferro. Dado: massa específica da solução aquosa de HCl é igual a 1020 kg.m–3 na temperatura de 25oC. 34 APOSTILA 01 DE FÍSICO-QUÍMICA – PROF. PEDRO MADEIRA (2022) 84. (ITA 2010 – Q12) Um vaso de pressão com volume interno de 250 cm3 contém gás nitrogênio (N2) quimicamente puro, submetido à temperatura constante de 250°C e pressão total de 2,0 atm. Assumindo que o N2 se comporta como gás ideal, assinale a opção CORRETA que apresenta os respectivos valores numéricos do número de moléculas e da massa específica, em kg m–3, desse gás quando exposto às condições de pressão e temperatura apresentadas. A ( ) 3,7 x 1021 e 1,1 B ( ) 4,2 x 1021 e 1,4 C ( ) 5,9 x 1021 e 1,4 D ( ) 7,2 x 1021 e 1,3 E ( ) 8,7 x 1021 e 1,3 85. (ITA 2014 – Q11) Considere um mol de um gás que se comporta idealmente, contidoem um cilindro indeformável provido de pistão de massa desprezível, que se move sem atrito. Com relação a este sistema, são feitas as seguintes afirmações: I. Se o gás for resfriado contra pressão externa constante, o sistema contrai-se. II. Se pressão for exercida sobre o pistão, a velocidade média das moléculas do gás aumenta. III. Se o sistema for aquecido a volume constante, a velocidade média das moléculas aumenta, independentemente da natureza do gás. IV. A velocidade média das moléculas será maior se o gás for o xenônio e menor se for o argônio. Das afirmações acima, está(ão) ERRADA(S) apenas A ( ) I e II. B ( ) I, III e IV. C ( ) II e III. D ( ) II e IV. E ( ) IV. Prof. Pedro Madeira 86. (ITA 2014 – Q24) Escreva a reação de combustão completa de um hidrocarboneto genérico (CaHb) com ar atmosférico. Considere a presença do nitrogênio gasoso no balanço estequiométrico da reação e expresse os coeficientes estequiométricos dessa reação em função de α e β. 87. (ITA 2014 – Q27) Considere uma mistura gasosa constituída de C3H8, CO e CH4. A combustão, em excesso de oxigênio, de 50 mL dessa mistura gasosa forneceu 70 mL de CO2 (g). Determine o valor numérico do percentual de C3H8 na mistura gasosa. 88. (ITA 2015 – Q20) Contribuíram de forma direta para o desenvolvimento do conceito de pressão atmosférica A ( ) Friedrich August Kekulé e John Dalton. B ( ) Michael Faraday e Fritz Haber. C ( ) Galilei Galilei e Evangelista Torricelli. D ( ) Jons Jacob Berelius e Eduard Buchner. E ( ) Robert Bunsen e Henry Louis Le Chatelier 89. (ITA 2015 – Q21) 3,64 gramas de fosfeto de cálcio foram adicioados a uma certa quantidade de água. Após a reação completa, todo o produto gasoso formado foi recolhido em um recipiente de 8,2 mL. Calcule o valor numérico da pressão, em atm, exercida pelo produto gasoso a 27ºC. 90. (ITA 2016 – Q08) Uma amostra de 4,4 g de um gás ocupa um volume de 3,1 L a 10ºC e 566 mmHg. Assinale a alternativa que apresenta a razão entre as massas específicas dste gás e a do hidrogênio gasoso nas mesmas condições de pressão e temperatura. A ( ) 2,2 B ( ) 4,4 C ( ) 10 D ( ) 22 E ( ) 44 91. (ITA 2017 – Q03) Um frasco fechado contém dois gases cujo comportamento é considerado ideal: hidrogênio molecular e monóxido de nitrogênio. Sabendo que a pressão parcial do monóxido de nitrogênio é igual a 3/5 da pressão parcial do hidrogênio molecular, e que a massa total da mistura é de 20g, assinale a alternativa que fornece a porcentagem em massa do hidrogênio molecular na mistura gasosa. A ( ) 4% B ( ) 6% C ( ) 8% D ( ) 10% E ( ) 12% 92. (ITA 2017 – Q24) Após inalar ar na superfície, uma pessoa mergulha até uma profundidade de 200 m, em apneia, sem exalar. Desconsiderando as trocas gasosas que ocorrem nos alvéolos pulmonares, calcule a pressão parcial do nitrogênio e do oxigênio do ar contido no pulmão do mergulhador. 93. (ITA 2018 – Q05) Um recipiente de 240L de capacidade contém uma mistura dos gases ideais hidrogênio e dióxido de carbono, a 27ºC. Sabendo que a pressão parcial do dióxido de carbono é três vezes menor que a pressão parcial do hidrogênio e que a pressão total da mistura é de 0,82 atm, assinale a alternativa que apresenta, respectivamente, as massas de hidrogênio e de dióxido de carbono no recipiente. A ( ) 2g e 44g B ( ) 6g e 44g C ( ) 8g e 88g D ( ) 12g e 88g E ( ) 16g e 44g 94. (ITA 2018 – Q07) Considere um recipiente de 320 L, ao qual são adicionados gases ideais nas seguintes condições: I. Hélio: 30.000 cm3 a 760 cmHg e 27ºC II. Monóxido de carbono: 250 L a 1.140 mmHg e –23oC III. Monóxido de nitrogênio: 2m3 a 0,273 atm e 0oC Sabendo que a pressão total da mistura gasosa é 4,5 atm, assinale a opção que apresenta a pressão parcial do hélio na mistura gasosa. A ( ) 0,1 atm B ( ) 0,2 atm C ( ) 0,5 atm D ( ) 1,0 atm E ( ) 2,0 atm 35 APOSTILA 01 DE FÍSICO-QUÍMICA – PROF. PEDRO MADEIRA (2022) 95. (ITA 2019 – Q60) Uma amostra de gás contém 80% de metano, 10% de etano, 5% de propano e 5% de nitrogênio, em volume. Considerando que todos os átomos de carbono na amostra de gás são convertidos em butadieno com 100% de rendimento, assinale a opção que apresenta a massa de butadieno obtido a partir de 100 g do gás. A ( ) 50 g B ( ) 60 g C ( ) 70 g D ( ) 80 g E ( ) 90 g 96. (ITA 2020 – Q62) Um reator com 200 L de capacidade, possui uma mistura de dióxido de nitrogênio e monóxido de carbono a 400 K, cujo comportamento pode ser considerado ideal. Os gases reagem entre si para formar dióxido de carbono e monóxido de nitrogênio. A pressão total no reator é igual a 32,8 atm e, no início da reação, a pressão parcial do monóxido de carbono é três vezes maior que a do dióxido de nitrogênio. As massas iniciais de dióxido de nitrogênio e de monóxido de carbono são, respectivamente, A ( ) 1,5kg e 4,2kg. B ( ) 1,5kg e 4,5kg. C ( ) 1,5kg e 6,6kg. D ( ) 2,3kg e 4,2kg. E ( ) 2,3kg e 6,6kg. 97. (ITA 2020 – Q68) Considerando que o ar é composto aproximadamente de 21% de O2 e 79% de N2 em volume, tem-se que a razão molar ar/combustível da combustão completa de um determinado alcano é igual a 59,5. A partir desse dado, assinale a alternativa que corresponde à soma dos coeficientes estequiométricos de todas as substâncias presentes nessa reação. A ( ) 30,5 B ( ) 55,5 C ( ) 82,0 D ( ) 112,0 E ( ) 124,5 98. (IME 1993 – Q01) Uma determinada reação química gera um produto gasoso, do qual foi coletada uma amostra para análise. Verificou-se que a amostra, pesando 0,32g, ocupa 492cm3 a 27oC e 1atm de pressão, obedece à lei dos gases ideais e é formada por 75% em peso de carbono e 25% em peso de hidrogênio. Determine: a) Qual o peso molecular deste gás e b) Qual a sua fórmula molecular mínima? Prof. Pedro Madeira 99. (IME 1995 – Q02) Mistura-se um fluxo de ar seco com vapor de água para se obter ar úmido com 2,0%, em volume, de umidade. Admitindo o comportamento ideal dos gases e a massa molecular média do ar seco como 28,96 g/mol, calcule a massa específica do ar úmido a 14,25oC e 1,00 x 105 Pa. 100. (IME 1997 – Q01) Para a determinação do poder calorífico de uma amostra, devemos encher uma bomba calorimétrica de volume 4,0x10-4 m3 com oxigênio até atingirmos uma pressão manométrica de 2,0x106 Pa. Na preparação da bomba calorimétrica para a análise, utilizamos o oxigênio de um cilindro com volume de 0,01 m3, a uma pressão manométrica de 1,0x107 Pa. Admitindo que apenas 80% do conteúdo de oxigênio do cilindro seja efetivamente utilizado, e que devemos realizar 20 testes por semana, determine a duração, em semanas, do cilindro de oxigênio utilizado para encher a bomba calorimétrica, considerando que os gases tenham comportamento ideal. 101. (IME 1998 – Q05) Uma determinada quantidade de nitrogênio (N2) ocupa um recipiente com volume de 10 litros a uma temperatura de 127oC e a uma pressão de 4,92 atm. Adiciona-se ao nitrogênio 9,03 x 1023 moléculas de oxigênio (O2). Sabendo-se que a pressão final de equilíbrio do sistema é de 6,15 atm, calcule a temperatura final de equilíbrio. R = 0,082 atm.L/mol.K 102. (IME 1999 – Q04) Borbulha-se oxigênio através de uma coluna de água e, em seguida, coletam-se 100 cm3 do gás úmido a 23ºC e 1,06 atm. Sabendo que a pressão de vapor da água a 23ºC pode ser considerada igual a 0,03 atm, calcule o volume coletado de oxigênio seco nas CNTP. 103. (IME 2004 – Q05) Na figura abaixo, o cilindro A de volume VA contém um gás inicialmente a uma pressão P0 e encontra-se conectado, através de uma tubulação dotada de uma válvula (1), a um vaso menor B de volume VB, repleto do mesmo gás a uma pressão p tal que P0 > p > Patm. Abre- se a válvula 1 até que a pressão fique equalizada nos dois vasos, após o que, fecha-se esta válvula e abre-se a válvula 2 ate que apressão do vaso menor B retorne ao seu valor inicial p, completando um ciclo de operação. Sabendo-se que o sistema é mantido a uma temperatura constante T, pede-se uma expressão para a pressão do vaso A após N ciclos. 104. (IME 2005 – Q09) No equipamento esquematizado na figura abaixo, as torneiras A, B e C estão inicialmente fechadas. O compartimento 1 de volume 2,00 L contém oxigênio sob pressão de 1,80 atm. O compartimento 2 contém nitrogênio. O compartimento 3 de volume 1,00 L contém 36 APOSTILA 01 DE FÍSICO-QUÍMICA – PROF. PEDRO MADEIRA (2022) nitrogênio e uma certa quantidade de sódio metálico. Executam-se, então, isotermicamente, as três operações descritas a seguir: 1ª) mantendo a torneira A fechada, abrem-se B e C e faz-se o vácuo nos recipientes 2 e 3, sem alterar a quantidade de sódio existente em 3; 2ª) fecham-se B e C e abre-se A, constatando que, após atingir o equilíbrio, o manômetro M1 indica uma pressão de 1,20 atm; 3ª) fecha-se A e abre-se B, verificando que, atingido o equilíbrio, o manômetro M2 indica uma pressão de 0,300 atm. Finalmente, fecha-se a torneira B e eleva-se a temperatura do recipiente 3 até 77,0oC, quando então, a pressão indicada por M2 é de 0,400 atm. Calcule a massa inicial de sódio, considerando que, antes da elevação da temperatura, todo o sódio se transformara em óxido de sódio, e que os volumes das tubulações e dos sólidos (sódio e seu óxido) são desprezíveis. 105. (IME 2006 – Q03) O gás obtido pela completa decomposição térmica de uma amostra de carbonato de cálcio com 50,0% de pureza é recolhido em um recipiente de 300 mL a 27,0ºC. Sabendo-se que a pressão no recipiente é de 1,66 MPa, determine: a) a massa de gás produzido, admitindo que seu comportamento seja ideal; b) a massa da amostra utilizada. 106. (IME 2007 – Q01) Determine o volume de cloro obtido, a 27,0ºC e 738 mmHg, pela ação de excesso de ácido clorídrico concentrado sobre 30,7 g de pirolusita com 85,0% em peso de MnO2. Considere o cloro com comportamento ideal. 107. (IME 2008 – Q07) Uma amostra de 0,512 g de uma liga metálica Al-Zn reage com HCl, recolhendo-se o gás formado. Após a total dissolução da amostra, o gás recolhido é seco, resfriado e submetido a um processo de compressão representado pela reta AB no diagrama P-V. Sabendo que a temperatura máxima ao longo do processo de compressão é 298K, determine o teor de alumínio nesta amostra. Considere que o gás se comporta idealmente. 108. (IME 2010 – Q32) Em um recipiente fechado queima-se propano com 80% da quantidade estequiométrica de ar. Admitindo que não haja hidrocarbonetos após a combustão, que todos os produtos da reação estejam na fase gasosa e que a composição volumétrica do ar seja de uma parte de O2 para quatro partes de N2, calcule a porcentagem molar de CO2 no recipiente após a combustão (considere comportamento ideal para os gases). (A) 4,35% (B) 4,76% (C) 5,26% (D) 8,70% (E) 14,28% Prof. Pedro Madeira 109. (IME 2010 – Q39) – RESOLVA AGORA As alternativas abaixo representam processos hipotéticos envolvendo 2 mols de um gás ideal, contidos em um conjunto cilindro-pistão. Assinale a alternativa que apresenta mais de três estados (V, T) nos quais a pressão é máxima: (A) (B) (C) V(Litros) 15 12,5 10 300 390 480 T(K) V(Litros) 12 9 300 330 450 480 T(K) V(Litros) 15 10 320 480 T(K) P(atm) 0,25 0,40 V (L) 0,90 0,60 B A 37 APOSTILA 01 DE FÍSICO-QUÍMICA – PROF. PEDRO MADEIRA (2022) (D) (E) 110. (IME 2010 – Q40) Um sistema fechado e sem fronteiras móveis contém uma determinada massa gasosa inerte. Sabe-se que, após aquecimento, o sistema registra um aumento de 5% na pressão e de 15ºC na temperatura (considere que o gás se comporta idealmente). A respeito do valor da temperatura inicial, pode-se dizer que: (A) é igual ou inferior a 30ºC. (B) é superior a 30ºC e inferior a 300ºC. (C) é igual ou superior a 300ºC. (D) somente pode ser calculado conhecendo-se o volume e a massa de gás. (E) somente pode ser calculado conhecendo-se o volume, a massa e a pressão inicial do gás. 111. (IME 2011 – Q31) Um recipiente de paredes rígidas, contendo apenas ar, aberto para a atmosfera, é aquecido de 27ºC a 127ºC. Calcule a percentagem mássica de ar que saiu do recipiente, quando atingido o equilíbrio final. (A) 79% (B) 75% (C) 30% (D) 25% (E) 21% 112. (IME 2011 – Q40) Um gás ideal sofre uma mudança de estado ilustrada pelos gráficos I e II abaixo. Dentre as alternativas abaixo, assinale aquela que se ajusta aos gráficos acima. (A) a é o volume, b é a temperatura, d é a pressão e o processo é uma expansão a temperatura constante. (B) d é a temperatura, b é a pressão, a é o volume e o processo é uma compressão. (C) a é o volume, b é a pressão, d é a temperatura e o processo é um resfriamento isobárico. (D) a é o volume, b é a temperatura, d é a pressão e o processo é uma compressão isotérmica. (E) a é a pressão, b é o volume, d é a temperatura e o processo é um aquecimento isobárico. 113. (IME 2012 – Q05) Na figura, uma solução concentrada de HCl, contida em A, é gotejada sobre zinco sólido em B. Um dos produtos dessa reação escoa para C, onde é completamente consumido na reação com o vapor de uma substância simples, cujo elemento pertence à família 17. O produto da reação ocorrida em C é um gás incolor. A válvula V permite somente o escoamento no sentido de B para C. O recipiente C possui volume de 1,0 L, é mantido a 100°C durante todo o processo e contém inicialmente 0,05 mol da substância simples supracitada. Observações: * os volumes das conexões e tubulações devem ser desconsiderados; * a substância presente inicialmente em C é um líquido marrom-avermelhado à temperatura ambiente. Determine: a) as reações que ocorrem em B e C, identificando o estado físico de cada uma das substâncias envolvidas. b) o número máximo de mols do produto da reação em B que pode escoar para C, sem que a pressão neste exceda 2,0 atm, se a extremidade D for fechada. V(Litros) 12 9 300 330 450 480 T(K) V(Litros) 15 10 300 450 T(K) 38 APOSTILA 01 DE FÍSICO-QUÍMICA – PROF. PEDRO MADEIRA (2022) 114. (IME 2013 – Q06) Um tubo vertical graduado, dotado de um êmbolo de peso não desprezível e sem atrito e de um dispositivo elétrico para produzir centelhamento, contém uma mistura gasosa composta de amônia (NH3) e fosfina (PH3) em equilíbrio térmico. Introduz-se, então, um volume de oxigênio gasoso que contém apenas a massa necessária para a oxidação estequiométrica dos reagentes presentes. Após a estabilização à temperatura original, o deslocamento do êmbolo indica um aumento de volume de 150 cm3. Provoca-se o centelhamento elétrico e, após o término da reação de combustão e o retorno à temperatura inicial, identifica- se um volume parcial de 20,0 cm3 de nitrogênio gasoso. Considerando que os únicos produtos reacionais nitrogenado e fosforado são, respectivamente, nitrogênio gasoso e pentóxido de difósforo, determine o volume da mistura original, antes da introdução do O2. 115. (IME 2014 – Q04) O TNT (2,4,6-trinitrotolueno) é um composto químico com propriedades combustíveis e explosivas. Em condições específicas e controladas, m gramas de TNT entram em combustão completa em presença de ar estequiométrico sem detonar ou explodir. Os produtos dessa reação foram coletados e transferidospara um sistema de captura de 820 L. Ao atingirem equilíbrio térmico com o ambiente (27oC), a pressão registrada no sistema de captura foi de 1,77 atm. Assumindo que a hipótese do gás ideal é válida, que o ar é uma mistura de N2 e O2 na proporção volumétrica de 4:1, que todo o nitrogênio existente nos produtos está na forma de uma única substância simples e que não existem produtos sólidos, determine o valor de m. 116. (IME 2015 – Q01) Uma amostra de 1,264 g de Nitropenta, uma substância sólida explosiva cuja fórmula estrutural é dada abaixo, é detonada num vaso fechado resistente de 0,050 dm3 de volume interno, pressurizado com a quantidade estequiométrica de oxigênio puro, a 300 K, necessária para a combustão completa. Calcule a pressão inicial do vaso, considerando o comportamento dos gases como ideal. 117. (IME 2017 – Q02) Uma amostra de magnésio metálico reage completa e estequiometricamente com uma mistura de oxigênio e nitrogênio em proporção molar 1:3, respectivamente, produzindo óxido de magnésio (sólido) e nitreto de magnésio (sólido). Em seguida, adiciona-se água em excesso aos produtos. Determine as massas de nitreto de magnésio e de magnésio, necessárias para liberar 11,2 L de amônia nas CNTP, conforme o procedimento descrito. 118. (IME 2018 – Q32) Um sistema fechado contendo um gás ideal no estado 1 sofre as transformações α e β, conforme indicado na figura abaixo. Sabendo que a transformação α é isotérmica e β isobárica, indique o gráfico que representa os estados do sistema. Prof. Pedro Madeira 119. (IME 2022 – Q39) Uma amostra de 390 g de sulfito de cálcio com 25% de impurezas, em massa, é atacada por ácido clorídrico concentrado em um meio reacional a 2 atm e 300 K. Considere comportamento ideal de gases. O N O O O N O O O N O O O N O O + + + + 39 APOSTILA 01 DE FÍSICO-QUÍMICA – PROF. PEDRO MADEIRA (2022) Pode-se afirmar que o volume, em litros, de anidrido sulforoso obtido pelo consumo completo do sulfito é: (A) 22,4 (B) 30,0 (C) 40,0 (D) 54,6 (E) 72,8 TÓPICO 02 – TEORIA CINÉTICA DOS GASES IDEAIS SEÇÃO VESTIBULARES 120. (UNICID 2016) Comprime-se um gás, à pressão constante de 1,0 atm, empurrando um êmbolo de modo que seu volume passe de 0,20 m3 para 0,10 m3. a) Nessa compressão, a energia interna desse gás aumenta ou diminui? Justifique sua resposta. b) Sabendo que a compressão foi realizada a 27ºC, calcule a pressão que deve ser aplicada para manter o mesmo volume de gás comprimido, à temperatura de 0oC. 121. (ESPCEX 2021) Gases apresentam um fenômeno chamado de difusão de gases. Considere que, em determinadas condições de temperatura e pressão, a velocidade de difusão de 1 mol do gás hidrogênio seja de 28 km/min. Nestas mesmas condições a velocidade (em km/h) de 1 mol do gás metano é de a) 600 km/h b) 729 km/h c) 1211 km/h d) 422 km/h e) 785 km/h 122. (UEM 2016) Assinale o que for correto. 01) No que diz respeito à interpretação microscópica de uma transformação isobárica, pode-se dizer que o aumento da violência das colisões contra as paredes internas do recipiente, provocado pelo aumento de temperatura, é compensado pela diminuição da frequência com que as colisões ocorrem. 02) Um balão de festas de 2 L, mantido a 21oC em um ambiente com ar condicionado, é levado para o exterior, onde a temperatura é 32oC. Admitindo-se que as variações de pressão possam ser desprezadas, o aumento de volume do balão será inferior a 10% do volume inicial. 04) Considere a decomposição do CaCO3(s) em CaO(s) e CO2(g). Admitindo-se que 50,0g de CaCO3(s) tenham sido totalmente decompostos, a pressão do CO2(g) produzido será de aproximadamente 4,1 atm se este gás tiver sido coletado em um recipiente de 3L e estabilizado a uma temperatura de 27ºC. 08) Um gás real, sendo resfriado isobaricamente a 1atm, atingirá seu menor volume possível, no estado gasoso, a –273oC. 16) Segundo a Lei de Amagat, o volume total de uma mistura gasosa é igual a soma dos volumes parciais de seus componentes. 123. (UEM 2012) Considerando dois recipientes idênticos e hermeticamente fechados A e B, contendo as mesmas quantidades molares dos gases rarefeitos CO2 e H2, respectivamente, que possuem a mesma energia cinética média por molécula, assinale o que for correto. 01) A soma da energia cinética média de todas as partículas constitui a energia interna dos gases contidos nos recipientes A e B. 02) Quanto maior a energia cinética média das partículas, maior será a temperatura do gás. 04) Se os gases contidos em A e B estiverem sob o mesmo nível de agitação térmica, a energia interna do gás em A será maior devido à sua massa molar maior. 08) Como o CO2 possui uma massa molar maior que o H2, a pressão que ele exerce sobre as paredes do recipiente A é maior que a pressão que o H2 exerce sobre as paredes do recipiente B. 16) A pressão manométrica exercida pelos gases contidos em A e B sobre as paredes dos respectivos recipientes independe da velocidade média ou da taxa de colisão das moléculas do gás com as paredes do recipiente. 124. (UPE 2012) Dois chumaços de algodão, I e II, embebidos com soluções de ácido clorídrico, HCl, e amônia, NH3, respectivamente, são colocados nas extremidades de um tubo de vidro mantido fixo na horizontal por um suporte, conforme representação abaixo. Após um certo tempo, um anel branco, III, forma-se próximo ao chumaço de algodão I. Baseando-se nessas informações e no esquema experimental, analise as seguintes afirmações: I. O anel branco forma-se mais próximo do HCl, porque este é um ácido forte, e NH3 é uma base fraca. 40 APOSTILA 01 DE FÍSICO-QUÍMICA – PROF. PEDRO MADEIRA (2022) II. O anel branco formado é o NH4Cl sólido, resultado da reação química entre HCl e NH3 gasosos. III. O HCl é um gás mais leve que NH3, logo se movimenta mais lentamente, por isso o anel branco está mais próximo do ácido clorídrico. Está correto o que se afirma em a) II. b) III. c) I e II. d) I e III. e) II e III. 125. (UECE 2008) A partir das pesquisas de Robert Boyle (1627- 1691), foi possível estabelecer a teoria cinético-molecular dos gases. Essa teoria afirma que A) todos os choques entre as partículas de um gás ideal e as paredes do recipiente são perfeitamente elásticos. B) nas mesmas condições de temperatura e pressão, as velocidades de difusão de dois gases são diretamente proporcionais às raízes quadradas de suas densidades. C) um mol de qualquer gás, nas condições padrões de temperatura e pressão (CPTP), ocupa um volume de 22,4L. D) à temperatura constante, o volume de uma massa de gás é diretamente proporcional à sua pressão. 126. (CEFET/CE 2006) Sobre o comportamento dos gases, é falsa a opção: A) Quando dois gases estão envolvidos numa reação, há uma relação simples entre seus volumes, medidos à mesma pressão e temperatura. B) As colisões entre as moléculas de um gás são perfeitamente elásticas, logo ocorrem sem perda de energia. C) Duas ampolas, uma contendo amônia (NH3) e a outra contendo gás sulfídrico (H2S – que tem cheiro de ovo podre), foram quebradas num canto de uma sala. Sabendo que a densidade da amônia é menor que a densidade do H2S, uma pessoa que está no centro dessa sala sentirá primeiro o cheiro de ovo podre. D) Um gás é composto por moléculas que estão separadas umas das outras por distâncias muito maiores do que suas próprias dimensões. As moléculas podem ser consideradas “pontos”, isto é, têm massa, mas o seu volume é desprezível. E) Todos os gases são semelhantes em um aspecto: a dependência do volume em relação à quantidade, à temperatura e à pressão. 127. (CEFET/CE 2007) Sobre os gases, é falso afirmar que: A) Quando entram em contato com uma superfície, exercem pressão sobre ela,porque as moléculas gasosas estão em movimento constante. B) Ao contrário das moléculas na matéria condensada, as moléculas gasosas estão separadas por distâncias grandes comparadas com o seu volume. Em conseqüência, as densidades dos gases são muito baixas nas condições atmosféricas. C) As leis dos gases ajudam-nos a prever o seu comportamento, mas não explicam que acontece à escala molecular e que origina as modificações observadas no mundo macroscópico. D) Diminuindo o volume de uma dada quantidade de gás, aumenta a sua densidade numérica (número de moléculas por unidade de volume) e, conseqüentemente, diminui a freqüência de colisões, portanto a pressão de um gás é inversamente proporcional ao volume que ele ocupa. E) A teoria cinética, um método de descrever o comportamento das moléculas gasosas, baseia-se nas seguintes hipóteses: as moléculas gasosas estão separadas por distâncias muito maiores que suas próprias dimensões; possuem massa, mas têm volume desprezível, estão em movimento constante e colidem freqüentemente umas com as outras. 128. (FUVEST 2012) Uma estudante de Química realizou um experimento para investigar as velocidades de difusão dos gases HCl e NH3. Para tanto, colocou, simultaneamente, dois chumaços de algodão nas extremidades de um tubo de vidro, como mostrado na figura acima. Um dos chumaços estava embebido de solução aquosa de HCl(g), e o outro, de solução aquosa de NH3(g). Cada um desses chumaços liberou o respectivo gás. No ponto de encontro dos gases, dentro do tubo, formou-se, após 10 s, um anel de sólido branco (NH4Cl), distante 6,0 cm do chumaço que liberava HCl(g). a) Qual dos dois gases, desse experimento, tem maior velocidade de difusão? Explique. b) Quando o experimento foi repetido a uma temperatura mais alta, o anel de NH4Cl(s) se formou na mesma posição. O tempo necessário para a formação do anel, a essa nova temperatura, foi igual a, maior ou menor do que 10 s? Justifique. c) Com os dados do experimento descrito, e sabendo-se a massa molar de um dos dois gases, pode-se determinar a massa molar do outro. Para isso, utiliza- se a expressão Considere que se queira determinar a massa molar do HCl. Caso o algodão embebido de solução aquosa de NH3 (g) seja colocado no tubo um pouco antes do algodão que libera HCl(g) (e não simultaneamente), como isso afetará o valor obtido para a massa molar do HCl? Explique. 3 3 velocidade de difusão do NH (g) MM (HC ) velocidade de difusão do HC (g) MM (NH ) = ! ! 41 APOSTILA 01 DE FÍSICO-QUÍMICA – PROF. PEDRO MADEIRA (2022) SEÇÃO ITA / IME 129. (ITA 1983 – Q19) Nitrogênio gasoso, inicialmente na temperatura ambiente, é passado por um tubo mantido num forno. A vazão do gás é tão baixa que a pressão na saída (quente) é praticamente igual à da entrada (frio). Chamemos as vazões do gás (cm3/s) na entrada de v1 e na saída de v2. A densidade do gás (g/cm3) na entrada é designada por d1 e na saída por d2. Nas condições acima teremos que: A ( ) v1 < v2 ; d1 < d2 B ( ) v1 < v2 ; d1 > d2 C ( ) v1 > v2 ; d1 < d2 D ( ) v1 > v2 ; d1 > d2 E ( ) v1 = v2 ; d1 = d2 PERGUNTA Partindo da lei geral dos gases perfeitos, deduza uma expressão que fornece o valor da densidade de um gás (d) em função da pressão (p), da temperatura (T) e de sua massa molar (M). 130. (ITA 1989 – Q29) Consideremos um gás formado de moléculas todas iguais e que corresponda ao que se considera um gás ideal. Este gás é mantido num recipiente de volume constante. Dentre as afirmações abaixo, todas referentes ao efeito do aumento de temperatura, assinale a CORRETA, em relação ao caminho livre médio das moléculas e à freqüência das colisões entre as mesmas: Caminho livre médio Freqüência de colisões A ( ) Inalterado Aumenta B ( ) Diminui Inalterada C ( ) Aumenta Aumenta D ( ) Inalterado Diminui E ( ) Diminui Aumenta Prof. Pedro Madeira 131. (ITA – 1992 – Q19) Um recipiente A contém, inicialmente, uma mistura gasosa, comprimida, dos isótopos 20 e 22 do Neônio. Este recipiente é envolvido completamente por outro, B, conforme a figura ilustrada abaixo. No inicio, o recipiente B estava completamente evacuado. Por um pequeno furo na parede de A, o gás escapa de A para B. Numa situação deste tipo, a concentração (em fração molar) do isótopo mais leve no gás remanescente dentro do recipiente A, em função do tempo, a partir do início do vazamento: A ( ) permanece constante. B ( ) vai diminuindo sempre. C ( ) vai aumentando sempre. D ( ) aumenta, passa por um máximo, retomando ao valor inicial. E ( ) diminui, passa por um mínimo, retomando ao valor inicial. 132. (ITA 2006 – Q09) A figura mostra cinco curvas de distribuição de velocidade molecular para diferentes gases (I, II, III, IV e V) a uma dada temperatura. Assinale a opção que relaciona CORRETAMENTE a curva de distribuição de velocidade molecular a cada um dos gases. A ( ) I = H2, II = He, III = O2, IV = N2 e V = H2O. B ( ) I = O2, II = N2, III = H2O, IV = He e V = H2. C ( ) I = He, II = H2, III = N2, IV = O2 e V = H2O. D ( ) I = N2, II = O2, III = H2 , IV = H2O e V = He. E ( ) I = H2O, II = N2, III = O2 , IV = H2 e V = He. 133. (ITA 2007 – Q30) Dois frascos, A e B, contêm soluções aquosas concentradas em HCl e NH3, respectivamente. Os frascos são mantidos aproximadamente a um metro de distância entre si, à mesma temperatura ambiente. Abertos os frascos, observa-se a formação de um aerossol branco entre os mesmos. Descreva o fenômeno e justifique por que o aerossol branco se forma em uma posição mais próxima a um dos frascos do que ao outro. 134. (ITA 2009 – Q13) Assumindo um comportamento ideal dos gases, assinale a opção com a afirmação CORRETA. A ( ) De acordo com a Lei de Charles, o volume de um gás torna-se maior quanto menor for a sua temperatura. B ( ) Numa mistura de gases contendo somente moléculas de oxigênio e nitrogênio, a velocidade média das moléculas de oxigênio é menor do que as de nitrogênio. C ( ) Mantendo-se a pressão constante, ao aquecer um mol de gás nitrogênio sua densidade irá aumentar. D ( ) Volumes iguais dos gases metano e dióxido de carbono, nas mesmas condições de temperatura e pressão, apresentam as mesmas densidades. E ( ) Comprimindo-se um gás a temperatura constante, sua densidade deve diminuir. 135. (ITA 2010 – Q13) Um recipiente contendo gás hidrogênio (H2) é mantido à temperatura constante de 0°C. Assumindo que, nessa condição, o H2 é um gás ideal e sabendo-se que a velocidade média das moléculas desse gás, nessa temperatura, é de 1,85 x 103 m s–1, assinale a alternativa CORRETA que apresenta o valor calculado da energia cinética média, em J, de uma única molécula de H2. A ( ) 3,1 x 10–24 B ( ) 5,7 x 10–24 C ( ) 3,1 x 10–21 D ( ) 5,7 x 10–21 E ( ) 2,8 x 10–18 FURO A B 42 APOSTILA 01 DE FÍSICO-QUÍMICA – PROF. PEDRO MADEIRA (2022) 136. (ITA 2011 – Q20) Considere dois cilindros idênticos (C1 e C2), de paredes rígidas e indeformáveis, inicialmente evacuados. Os cilindros C1 e C2 são preenchidos, respectivamente, com O2(g) e Ne(g) até atingirem a pressão de 0,5 atm e temperatura de 50ºC. Supondo comportamento ideal dos gases, são feitas as seguintes afirmações: I. O cilindro C1 contém maior quantidade de matéria que o cilindro C2. II. A velocidade média das moléculas no cilindro C1 é maior que no cilindro C2. III. A densidade do gás no cilindro C1 é maior que a densidade do gás no cilindro C2 . IV. A distribuição de velocidades das moléculas contidas no cilindro C1 é maior que a das contidas no cilindro C2. Assinale a opção que apresenta a(s) afirmação(ões) CORRETA(S). A ( ) Apenas I e III. B ( ) Apenas I e IV. C ( ) Apenas II. D ( ) Apenas II e IV. E ( ) Apenas III. 137. (ITA 2016 – Q15) Considerando um gás monoatômicoideal, assinale a opção que contém o gráfico que melhor representa como a energia cinética média (Ec) das partículas que compõem este gás varia em função da temperatura absoluta (T) deste gás A ( ) B ( ) C ( ) D ( ) E ( ) TÓPICO 03 – GASES REAIS SEÇÃO VESTIBULARES 138. (UFC 2004) “AR EM TUBULAÇÃO FAZ CONTA DE ÁGUA DISPARAR” (Folha de São Paulo, 27 de agosto de 2001). Esse fenômeno ocorre porque o ar ocupa rapidamente os espaços vazios nas tubulações de água. Quando o fornecimento é regularizado, a água empurra a solução gasosa acumulada nas tubulações fazendo o hidrômetro girar rapidamente. Sabendo que há uma pressão moderada na tubulação, analise as afirmativas I, II e III, e assinale a alternativa correta. I. O ar é constituído de uma solução gasosa real, cujos componentes nas CNTP experimentam interações de atração que o tornam mais denso, se comparado a uma mistura ideal de mesma composição. II. O ar ocupa rapidamente os espaços vazios nas tubulações devido a sua elevada densidade, uma vez que trata-se de uma mistura heterogênea. III. Deve-se esperar uma redução na velocidade de rotação do hidrômetro em dias frios. A) Somente I e II são verdadeiras. B) Somente II é verdadeira. C) Somente III é verdadeira. D) Somente I e III são verdadeiras. E) Somente II e III são verdadeiras. SEÇÃO ITA / IME 139. (ITA 1982 – Q08) O cilindro provido de um pistão móvel, esquematizado ao lado, contém apenas H2O e é mantido sob temperatura constante igual a 25oC. Assinale a alternativa que melhor representa a variação do volume com a pressão aplicada, abrangendo o H2O desde completamente vaporizado até totalmente liquefeito. A ( ) B ( ) C ( ) D ( ) E ( ) Ec (0,0) T Ec (0,0) T Ec (0,0) T Ec (0,0) T Ec (0,0) T 43 APOSTILA 01 DE FÍSICO-QUÍMICA – PROF. PEDRO MADEIRA (2022) PERGUNTA Exponha as razões físicas responsáveis pelas inclinações da cada um dos três trechos do gráfico correto e diga que tipo de equação descreve cada um dos três trechos, explicando se a equação é exata ou aproximada. 140. (ITA 1988 – Q10) Consideremos um recipiente de paredes inertes e indeformáveis. A capacidade desse recipiente é de aproximadamente 25 litros. Ele é provido de um manômetro absoluto e é mantido numa sala termostatada a 20oC. A única comunicação do recipiente com o exterior é feita através de um tubo provido de torneira. Inicialmente extraímos todo o ar contido no recipiente com o auxílio de uma bomba de vácuo. Feito isto, introduzimos no recipiente, contínua e lentamente, água pura (isenta de ar) até um total de 40 g de água. Qual dos gráficos a seguir descreve corretamente a variação da pressão no recipiente versus massa de água introduzida após evacuação prévia do recipiente? A ( ) B ( ) C ( ) D ( ) E ( ) Prof. Pedro Madeira 141. (ITA 1989 – Q33) Num grande cilindro provido de torneira e pistão com êmbolo, conforme figura abaixo, foi introduzido um pouco de água líquida, tomando o cuidado de não deixar entrar ar. Após a admissão da porção de água, a torneira foi fechada. Variando o volume, por movimento lento do pistão, mantendo a temperatura, no interior do cilindro, igual a 20oC, o gráfico de pressão no cilindro versus volume, corresponde a: A ( ) B ( ) C ( ) D ( ) E ( ) 142. (ITA 1993 – Q10) O cilindro de um pistão móvel, esquematizado abaixo, contém apenas etanol puro e é mantido sob temperatura constante de 20°C. Assinale a alternativa que melhor representa a variação do volume (v) com a pressão (p) aplicada, abrangendo etanol desde completamente vaporizado até totalmente liquefeito. A ( ) B ( ) C ( ) D ( ) E ( ) 143. (ITA 1998 – P10) A figura a seguir mostra de forma esquemática três isotermas, pressão versus volume, para o caso de um gás ideal. Trace isotermas análogas para o caso de um gás real que, por compressão, acaba totalmente liquefeito. No seu gráfico deve ficar claro, para cada isoterma, quais são os pontos que correspondem ao início e ao fim da liquefação em função da redução do volume. m P 0 m P 0 m P 0 m P 0 m P 0 V P 0 V P 0 V P 0 V P 0 V P 0 P V P V P V P V P V 44 APOSTILA 01 DE FÍSICO-QUÍMICA – PROF. PEDRO MADEIRA (2022) 144. (ITA 2012 – Q09) Considere volumes iguais dos gases NH3, CH4 e O2 nas CNTP. Assinale a opção que apresenta o(s) gás(es) que se comporta(m) idealmente. A ( ) Apenas NH3 B ( ) Apenas CH4 C ( ) Apenas O2 D ( ) Apenas NH3 e CH4 E ( ) Apenas CH4 e O2 145. (IME 2001 – Q03) A equação do gás ideal só pode ser aplicada para gases reais em determinadas condições especiais de temperatura e pressão. Na maioria dos casos práticos é necessário empregar uma outra equação, como a de van der Waals. Considere um mol do gás hipotético A contido num recipiente hermético de 1,1 litros a 27ºC. Com auxílio da equação de van der Waals, determine o erro cometido no cálculo da pressão total do recipiente quando se considera o gás A como ideal. Dados: Constante universal dos gases: R=0,082atm.L.mol–1.K–1. Constantes da equação de van der Waals: a = 1,21atm.L2.mol–2 e b = 0,10L.mol–1. 146. (IME 2009 – Q36) Assinale a alternativa correta. A) Um veículo de testes para redução de poluição ambiental, projetado para operar entre –40oC e 50oC, emprega H2 e O2, os quais são estocados em tanques a 13 MPa. Pode-se afirmar que a lei dos gases ideais não é uma aproximação adequada para o comportamento dos gases no interior dos tanques. (Dado: 1 atm = 101,3 kPa). B) A pressão de vapor de um líquido independe da temperatura. C) Um recipiente de 500 mL, inicialmente fechado e contendo um líquido em equilíbrio com seu vapor, é aberto. Pode-se afirmar que a pressão de vapor do líquido aumentará. D) Na equação PV = nRT, o valor numérico de R é constante e independe do sistema de unidades empregado. E) De acordo com o princípio de Avogadro, pode-se afirmar que, dadas as condições de temperatura e pressão, o volume molar gasoso depende do gás considerado. 147. (ITA 2018 – Q26) Uma dada reação (I), cujo calor liberado é desconhecido, é conduzida em um reator que utiliza um gás mantido a volume constante (V) como banho térmico. Outras duas reações (II e III) conduzidas em condições similares apresentam calor liberado a volume constante (QV) conforme apresentado na tabela abaixo: Reação Equação QV (kJ mol–1) I A + ½ B à D ? II A + B à C 400 III D + ½ B à C 300 Considere as deguintes informações sobre o gás do banho térmico, que tem comportamento não ideal e obedece à equação: !P+n 2a V2 ' (V - nb)=nRT, Em que: a = 62,5 L2 atm mol–1; b = 0,4 L mol–1; n = 0,4 mol; V = 10 L; capacidade calorífica molar a volume constante (CV,m) = 83,33 J K–1mol–1 ; temperatura inicial (Ti) = 300 K. a) Sabendo que 0,1 mol de A são utilizados na reação I, calcule o QV liberado nessa reação. b) Determine a temperatura final do banho térmico. c) Determine a pressão inicial e a pressão final do banho térmico. 148. (IME 2021 – Q06) O modelo dos gases ideais, ou perfeitos, descreve bem o comportamento para a maioria dos casos, no entanto, foi necessário desenvolver modelos mais precisos dentre os quais se destaca a equação de Van der Waals. Deduza a equação de Van der Waals, assumindo que o volume da partícula/molécula não seja desprezível e existam interações entre as partículas moléculas. Considere o seguinte: • V é o volume do recipiente do gás; • B é o volume total ocupado pelas moléculas do gás; • As forças de atração são praticamentenulas no seio da mistura do gás; e • Próximo às paredes do recipiente, as moléculas são atraídas ao centro com uma força proporcional ao quadrado da concentração do gás, o que reduz a intensidade dos impactos nas paredes do recipiente. 45 APOSTILA 01 DE FÍSICO-QUÍMICA – PROF. PEDRO MADEIRA (2022) TÓPICO 04 – QUÍMICA DESCRITIVA DE GASES SEÇÃO VESTIBULARES 149. (UPE 2014) A formulação de um determinado produto comercial contém, em massa, 58% de solvente e 40% de uma mistura gasosa formada por CH3(CH2)2CH3, (CH3)2CHCH3 e CH3CH2CH3, numa proporção de 65%, 15% e 20%, respectivamente. Qual alternativa apresenta o produto que atende à descrição acima? a) Desodorante aerossol b) Extintor de incêndio c) Gás de cozinha d) Gás natural veicular – GNV e) Gás refrigerante de geladeira 150. (UFPR 2013) Nos últimos dois anos, a imprensa divulgou notícias sobre o risco de explosão oferecido por condomínios de luxo e um Shopping Center de São Paulo. Os estabelecimentos foram construídos sobre antigos lixões. Nesses casos, o órgão responsável, ligado à Secretaria de Meio Ambiente, autuou os estabelecimentos, exigindo providências quanto à instalação de sistema de extração de gases. Em relação a esse risco, considere as seguintes afirmativas: 1. O risco de explosão deve-se principalmente à presença de metano, produzido por micro- organismos em condições anaeróbicas, na decomposição do material orgânico presente no lixão. 2. Os gases oferecem risco de explosão porque reagem vigorosamente com agentes oxidantes fortes. 3. O gás metano é facilmente detectado pelo odor característico. 4. Os gases que oferecem risco de explosão apresentam alta densidade, formando lençóis nos compartimentos de subsolo, como garagens subterrâneas. Assinale a alternativa correta. a) Somente as afirmativas 2 e 3 são verdadeiras. b) Somente as afirmativas 1 e 2 são verdadeiras. c) Somente as afirmativas 2, 3 e 4 são verdadeiras. d) Somente as afirmativas 1 e 4 são verdadeiras. e) As afirmativas 1, 2, 3 e 4 são verdadeiras. 151. (UERJ 2010) O oxigênio gasoso pode ser obtido em laboratório por meio da decomposição térmica do clorato de potássio. Em um experimento, o gás foi produzido em um frasco A e recolhido em um frasco B que, inicialmente, continha apenas água. Observe o esquema: Ao final do experimento, verificaram-se as seguintes medidas no interior do frasco B: • volume de gás recolhido: 123 mL • temperatura interna: 27 oC • pressão total no nível da água: 786,7 mmHg • pressão de vapor da água: 26,7 mmHg Determine a massa de oxigênio gasoso, em gramas, recolhida no frasco B, e apresente a equação química completa e balanceada correspondente a sua obtenção. Prof. Pedro Madeira 152. (FUVEST 2008) No seguinte trecho (adaptado) de uma peça teatral de C. Djerassi e R. Hoffmann, as esposas de três químicos do século XVIII conversam sobre um experimento feito com uma mistura de gases. "SENHORA POHL - Uma vez o farmacêutico Scheele estava borbulhando [a mistura gasosa] através de uma espécie de água. MADAME LAVOISIER - Deve ter sido água de cal. SENHORA PRIESTLEY - A água ficou turva, não ficou? MADAME LAVOISIER - É o mesmo gás que expiramos... o gás que removemos com a passagem através da água de cal. SENHORA POHL - Depois ele me pediu que colocasse no gás remanescente um graveto já apagado, apenas em brasa numa das extremidades. Já estava escurecendo. SENHORA PRIESTLEY - E o graveto inflamou-se com uma chama brilhante... e permaneceu aceso!" Empregando símbolos e fórmulas atuais, podem-se representar os referidos componentes da mistura gasosa por: a) CO2 e O2 b) CO2 e H2 c) N2 e O2 d) N2 e H2 e) CO e O2 153. (FUVEST 2009) Cinco cilindros, A, B, C, D e E, contêm gases diferentes. Cada um contém apenas um dos seguintes gases: monóxido de carbono, dióxido de carbono, dióxido de enxofre, amônia e metano, não se sabendo, porém, qual gás está em qual cilindro. Com amostras dos gases, retiradas de cada cilindro, foram feitos os seguintes experimentos, a fim de identificá-los. I) Cada gás foi borbulhado em água, contendo algumas gotas de solução incolor de fenolftaleína. Apenas o do cilindro A produziu cor vermelha. 46 APOSTILA 01 DE FÍSICO-QUÍMICA – PROF. PEDRO MADEIRA (2022) II) O gás de cada cilindro foi borbulhado em água de cal. Apenas os gases dos cilindros C e D produziram precipitado. III) Os gases dos cilindros C e D foram borbulhados em uma solução aquosa ácida de permanganato de potássio, de coloração violeta. Apenas o gás do cilindro D descorou essa solução. IV) Os gases dos cilindros restantes (B e E) mostraram- se combustíveis. Ao passar os produtos da combustão dos gases desses dois cilindros por um tubo contendo cloreto de cálcio anidro, houve aumento de massa desse tubo apenas no caso do gás do cilindro B. a) Identifique os gases contidos nos cilindros A, B, C, D e E. b) Escreva as equações químicas balanceadas das reações do item II. c) A reação que ocorre no item III é uma reação de precipitação, neutralização ou oxirredução? Explique, sem escrever a equação química, o que ocorre nessa transformação. Dados: Sais de cálcio pouco solúveis em água CaCO3 - Carbonato de cálcio CaSO3 - Sulfato de cálcio CaSO4 - Sulfato de cálcio CaC2O4 - Oxalato de cálcio O cloreto de cálcio anidro é usado para absorver água. 154. (FUVEST 2009) Michael Faraday (1791-1867), em fragmento de "A história química de uma vela", assim descreve uma substância gasosa que preparou diante do público que assistia a sua conferência: "Podemos experimentar do jeito que quisermos, mas ela não pegará fogo, não deixará o pavio queimar e extinguirá a combustão de tudo. Não há nada que queime nela, em circunstâncias comuns. Não tem cheiro, pouco se dissolve na água, não forma solução aquosa ácida nem alcalina, e é tão indiferente a todos os órgãos do corpo humano quanto uma coisa pode ser. Então, diriam os senhores: 'Ela não é nada, não é digna de atenção da química. O que faz no ar?'" A substância gasosa descrita por Faraday é: a) H2 (g) b) CO2 (g) c) CO (g) d) N2 (g) e) NO2 (g) SEÇÃO ITA / IME 155. (ITA 1981 – Q06) Nas condições ambientes, qual das substâncias abaixo é um gás incolor e inodoro quando puro e que, se muito comprimido, pode explodir? A ( ) SH2 B ( ) NH3 C ( ) C2H4 D ( ) C2H2 E ( ) LiH PERGUNTA Como e para que fins esse gás incolor, que pode explodir por compressão excessiva, é preparado e armazenado usualmente? 156. (ITA 1994 – Q06) Ao colocar-se um pedaço de magnésio em uma solução de ácido clorídrico, verifica-se que ocorre aumento da temperatura e desprendimento de gás. O gás que se desprende é, sobretudo: A ( ) Hidrogênio B ( ) Vapor de água C ( ) Vapor de magnésio. D ( ) Mistura de vapores de magnésio e água. E ( ) Mistura de vapores de magnésio e hidrogênio. 157. (ITA 2000 – Q20) Num tubo de ensaio dissolve-se açúcar em água e acrescenta-se uma porção de fermento biológico do tipo utilizado na fabricação de pães. Após certo tempo observa-se a liberação de gás nesta mistura. O borbulhamento deste gás em uma solução aquosa não saturada em Ba(OH)2 provoca, inicialmente, sua turvação. Esta desaparece com o borbulhamento prolongado do gás. A respeito das descrições feitas nestes experimentos são feitas as seguintes afirmações: I. O produto gasoso formado, e responsável pela turvação inicial da solução de Ba(OH)2, é o monóxido de carbono (CO). II. O produto gasoso formado, e responsável pela turvação inicial da solução de Ba(OH)2, é o etanol. III. A turvação inicial da solução de Ba(OH)2 é justificada pela precipitação do Ba(HCO3)2(c). IV. A turvação inicial da solução de Ba(OH)2 é justificada pela precipitação do Ba(OH)2(c). V. O desaparecimento da turvação inicial da solução de Ba(OH)2 é justificado pelareação química representada pela seguinte equação: Ba(OH)2(c) + HCO3 –(aq) ® BaCO3(aq) + H2O(l) + OH–(aq) Das informações acima estão ERRADAS: A ( ) apenas I e III. B ( ) apenas I e V. C ( ) apenas II e IV. D ( ) apenas II, IV e V. E ( ) todas. 47 APOSTILA 01 DE FÍSICO-QUÍMICA – PROF. PEDRO MADEIRA (2022) 158. (ITA 2001 – Q06) Quando carbeto de alumínio (Al4C3) é adicionado em um béquer contendo água líquida a 25ºC, ocorre a formação de hidróxido de alumínio e a liberação de um gás. O gás formado é o A ( ) H2 B ( ) CO C ( ) CO2 D ( ) CH4 E ( ) C2H2 159. (ITA 2007 – Q07) Embrulhar frutas verdes em papel jornal favorece o seu processo de amadurecimento devido ao acúmulo de um composto gasoso produzido pelas frutas. Assinale a opção que indica o composto responsável por esse fenômeno. A ( ) Eteno. B ( ) Metano. C ( ) Dióxido de carbono. D ( ) Monóxido de carbono. E ( ) Amônia. 160. (ITA 2010 – Q23) A nitroglicerina, C3H5(ONO2)3(!), é um óleo denso que detona se aquecido a 218ºC ou quando é submetido a um choque mecânico. Escreva a equação que representa a reação química do processo, sabendo que a reação de decomposição é completa, e explique porque a molécula é explosiva. 161. (ITA 2013 – Q04) Na temperatura ambiente, hidróxido de potássio sólido reage com o cloreto de amônio sólido, com a liberação de um gás. Assinale a alternativa CORRETA para o gás liberado nesta reação. A ( ) Cl2 B ( ) H2 C ( ) HCl D ( ) NH3 E ( ) O2 Prof. Pedro Madeira QUESTÕES EXTRAS: OLIMPÍADA 162. (USNCO 2011) Um cilindro de 5,00L evacuado foi carregado com 25,5g de NH3 e 36,5g de HCl. Calcule a pressão final a 85ºC após os dois compostos reagirem completamente. (A) 2,94 atm (B) 5,88 atm (C) 8,82 atm (D) 14,7 atm 163. (USNCO 2011) A velocidade média molecular em uma amostra gasosa a 300K é 500 m/s. A temperatura deste gás é aumentada até que a velocidade média das moléculas fique 1000 m/s. Qual é a nova temperatura? (A) 420 K (B) 573 K (C) 600 K (D) 1200 K 164. (USNCO 2011) Um recipiente rígido foi preenchido com quantidades iguais de N2 e H2 gasosos a uma pressão total de 10,0 atm. Os gases reagem produzindo amônia gasosa. Se a pressão total do gás diminui a uma velocidade de 0,20 atm/s, qual é a taxa de diminuição da pressão parcial do N2 no recipiente? (A) 0,40 atm/s (B) 0,30 atm/s (C) 0,20 atm/s (D) 0,10 atm/s 165. (USNCO 2012) Uma amostra de H2 coletada sobre H2O a 23ºC e sob pressão de 732 mmHg possui um volume de 245 mL. Qual será o volume do H2 seco a 0oC e 1 atm? Po vap = 21 mmHg (A) 211 mL (B) 218 mL (C) 224 mL (D) 249 mL 166. (USNCO 2012) Duas amostras gasosas, uma de argônio e outra de hélio, possuem a mesma pressão, temperatura e volume. Qual afirmativa é correta assumindo comportamento ideal? (A) A amostra de hélio contém mais átomos do que a amostra de argônio e os átomos de hélio possuem maior velocidade média. (B) As duas amostras possuem O mesmo número de átomos mas os átomos de Hélio possuem maior velocidade média. (C) As duas amostras possuem o mesmo número de átomos e ambos os tipos de átomos possuem a mesma velocidade média. (D) As duas amostras possui o mesmo número de átomos, porém o argônio possui maior velocidade média. 167. (USNCO 2013) Qual substância é usada em equipamentos de auto- respiração porque absorve CO2 e H2O e libera gás O2? (A) KO2 (B) Na2O2 (C) NaOH (D) Li2O 168. (USNCO 2013) Em um experimento para verificar o valor do zero absoluto, um estudante foi instruído para medir o volume do hélio em uma seringa de 10 mL em intervalos de 10°C entre 0°C e 100°C. Ela foi orientada a plotar o volume versus temperatura e a extrapolar este gráfico até o volume zero e ler a temperatura resultante. Que modificação do procedimento experimental fornecerá o melhor valor para o zero absoluto? (A) corrigir cada volume medido para pressão de um atmosfera antes de plantar. (B) dobrar o número de valores temperatura-volume entre 0°C e 100°C. (C) usar um termômetro que possa medir a temperatura a ±0,10ºC entre 0°C e 100ºC. (D) pedir o volume do Hélio em uma seringa a -40°C e -80°C. 169. (USNCO 2013) Uma amostra de gás medida a 20ºC e 4,0 atm foi aquecida até 40ºC sob volume constante. Qual(ais) afirmativa(s) é(são) verdadeira(s) após o aquecimento relativamente a sua situação inicial? I. A energia cinética molecular média aumentou. II. A velocidade média molecular não mudou. III. A pressão do gás foi aumentada para 8,0 atm. 48 APOSTILA 01 DE FÍSICO-QUÍMICA – PROF. PEDRO MADEIRA (2022) IV. O número de colisões moleculares por segundo não mudou. (A) apenas I (B) apenas I e IV (C) apenas II e III (D) apenas II e IV 170. (USNCO 2013) Sob que condições o comportamento dos gases reais sofre mais desvia daquele previsto pela lei dos gases ideais? (A) baixa P, baixa T (B) alta P, baixa T (C) baixa P, alta T (D) alta P, alta T 171. (USNCO 2013) Quando 0,25L de nitrogênio líquido (d = 0,807 g/mL) for vaporizado, qual volume o gás ocuparia a 25ºC e 5,00 atm? (A) 71 L (B) 54 L (C) 35 L (D) 32 L 172. (USNCO 2015) Uma mistura de 0,50 mol de gás H2 e 1,3 mol de gás Ar foi colocada em um recipiente com volume de 4,82L. Se a temperatura da mistura é 50,0oC, qual é a pressão de H2 na amostra? (A) 1,5 atm (B) 2,8 atm (C) 7,2 atm (D) 9,9 atm 173. (USNCO 2017) Um gás com P = 615 mmHg está contido no tubo em U mostrado. Se h = 65 mm, qual é a pressão atmosférica, Patm? (A) 550 mmHg (B) 615 mmHg (C) 680 mmHg (D) 760 mmHg 174. (USNCO 2017) Em ambas as extremidades de um longo tubo de vidro, amostras de gases foram introduzidas simultaneamente. Uma delas recebeu cloreto de hidrogênio gasoso enquanto a outra extremidade recebeu amônia gasosa. Onde no tubo o NH4Cl se formou? (A) No centro do tubo (B) Próximo a extremidade onde o cloreto de hidrogênio foi inserido (C) Próximo a extremidade onde a amônia foi inserida (D) Uniformemente em todas as posições posições do tubo 175. (USNCO 2018) Acetona possui uma pressão de vapor de 0,307bar a 25°C. Uma amostra de 0,100 mol de acetona foi adicionada um recipiente que continha 1,00 L de argônio gasoso a uma pressão de 1,00 bar a 25ºC. Volume do recipiente então aumentou para 4,00 L sendo mantida a mesma temperatura. Qual é a pressão no recipiente após a expansão? (A) 0,250 bar (B) 0,307 bar (C) 0,557 bar (D) 0,870 bar 176. Dois frascos (A e B), de mesmo volume, estão cheios de gás submetidos à mesma temperatura. O frasco A contém 0,32 g de oxigênio (O2) e o frasco B contém 0,68 g de um gás B, de fórmula molecular Z2Y. A pressão no frasco A é "x" e a pressão no frasco B é "2x". a) Identifique o gás B. b) Que frasco contém o maior número de moléculas? c) Compare os dois gases quanto à energia cinética média de suas moléculas. d) Compare os dois gases quanto à velocidade molecular média. 177. Estima-se que a concentração de NO2 no ar atmosférico, em zonas industriais, seja da ordem de 0,021 ppm. a) calcule a pressão parcial de NO2, numa amostra de ar, quando a pressão atmosférica for de 0,98 atm. b) quantas moléculas de NO2 estarão presentes, nestas condições e na temperatura de 20� C, num aposento de 4,5 m x 4,3 m x 2,4 m ? c) escreva as equações químicas correspondentes aos seguintes enunciados: c.1) o dióxido de nitrogênio dissolve-se em água, formando ácido nítrico e óxido nitroso; c.2) a molécula de óxido nítrico sofre fotodissociação na atmosfera superior; c.3) na estratosfera o óxido nítrico sofre oxidação pelo ozônio. 178. Em um laboratório dedicado ao estudo da toxicidade de produtos químicos, foi estabelecido que: I) Para evitar danos à saúde, não se pode expor uma pessoa, por mais que oito horas, a uma atmosfera que contenha 10 ppm de HCN. II)A concentração letal de HCN no ar é de 300 mg/kg de ar (d=0,0012 g/cm3). Pergunta-se: a) Quantos miligramas de HCN por kg de ar corresponde a 10 ppm ? b) A que fração da dose letal corresponde 10 ppm ? c) Qual a massa de HCN que deve estar contida no ar, em um pequeno laboratório que mede 5m x 4m x 2,2m, para atingir a concentração letal? 179. Se a densidade de uma mistura de gases metano e propano, apresenta a mesma densidade que o gás etano, então, a proporção entre os volumes dos gases metano e propano nesta mistura é de: A) 1:1 B) 1:2 C) 1:3 D) 2:1 E) 3:1 49 APOSTILA 01 DE FÍSICO-QUÍMICA – PROF. PEDRO MADEIRA (2022) 180. Dentre as amostras de gases citadas a seguir, todas ocupando o volume de 1 litro, assinale aquela que contém o maior número de átomos: A) Metano, a 300 K e 1 atm B) Neônio, a 273 K e 760 torr C) Oxigênio, a 27°C e 760 mmHg D) Monóxido de carbono, em CNTP E) Dióxido de carbono, a 0°C e 2 atm 181. A maioria dos automóveis modernos é equipada com air bags que têm sido muito eficazes na redução de acidentes fatais no trânsito. Durante uma colisão frontal, o sensor do air bag envia um sinal elétrico que “detona” a rápida decomposição de um composto X, liberando uma grande quantidade de um certo gás. Dentro de um curto espaço de tempo o air bag infla, protegendo motorista e passageiros de um impacto frontal. 1,0 g de amostra de X é decomposto liberando 507 mL (a 110 kPa e 18°C) de um gás A, que é um dos componentes do ar. A reação também produz um resíduo sólido. O tratamento deste resíduo com excesso de água produz 172 mL (em CNTP) de um gás B. a) Determine as fórmulas moleculares dos compostos A, B e X. Mostre os cálculos realizados para chegar à sua resposta. b) O composto X é esperado ser molecular ou iônico. c) Qual a estrutura de X no estado sólido? 182. A análise química de um composto orgânico (um líquido incolor em CNTP) dá 14,40% de hidrogênio (por massa). Quando 0,870 g deste composto é queimado completamente na presença de oxigênio e os produtos da combustão são borbulhados através de um excesso de água de cal, são formados 6,20 g de um precipitado. A gravidade específica dos vapores do composto acima citado, em relação ao ar seco, é aproximadamente 2,5 vezes a do eteno. a) Determine a fórmula empírica do composto orgânico sob investigação; b) Determine a fórmula molecular e desenhe todas as possíveis estruturas isoméricas para esse composto; c) Dê o nome IUPAC para um par de isômeros que pertença a diferentes classes; d) Que volume de ar (em CNTP) contendo 20,8% de oxigênio (por volume) é requerido para combustão de 10,4 g desse composto orgânico? 183. 11,2 dm3 (CNTP) de uma mistura de propano (C3H8) e butano (C4H10) foram queimadas na presença de excesso de oxigênio. Todo o dióxido de carbono obtido foi passado através de uma solução de NaOH, obtendo- se 95,4 g de carbonato de sódio e 84 g de bicarbonato de sódio: a) Escreva as equações das reações de combustão do propano e do butano. b) Escreva as equações das reações de formação de carbonato e bicarbonato de sódio. c) Calcule a quantidade de CO2 desprendida na combustão da mistura de propano e butano. d) Calcule as massas de propano e butano na mistura. 184. Um frasco de volume 5,00 L foi evacuado e 43,78g de N2O4 foram admitidos. Em –196oC, este composto é um sólido incolor. A amostra foi aquecida até 25oC e no processo, N2O4 se vaporiza e se dissocia parcialmente para formar o gás NO2. A pressão cresce lentamente e se estabiliza em 2,96 atm. (a) escreva uma equação para a reação. (b) se o gás que está no frasco fosse exclusivamente N2O4, qual seria a pressão? (c) se todo o gás que está no frasco fosse NO2, qual seria a pressão? (d) quais são as frações molares de N2O4 e NO2 quando a pressão se estabiliza em 2,96 atm? Prof. Pedro Madeira 185. (IChO) a) Mostre com base na equação de van der Waals que i. em pressões suficientemente altas, Z > 1. Em altas temperaturas e baixas pressões, Z se aproxima do valor de um gás ideal. ii. em baixas temperaturas, Z pode ser menor do que 1. iii. para a = 0, Z aumenta linearmente com a pressão. b) Em uma certa temperatura, a variação de Z com P para o He e N2 é mostrada esquematicamente na seguinte figura. a (bar L2 mol–2) b (L mol–1) He 3,46x10–2 2,38x10–2 N2 1,37 3,87x10–2 Identifique o gráfico correspondente ao He e ao N2. c) Duas isotermas de van der Waals estão mostradas esquematicamente abaixo. Identifique aquela que corresponde a uma temperatura menor do que a temperatura crítica (Tc) do gás. 50 APOSTILA 01 DE FÍSICO-QUÍMICA – PROF. PEDRO MADEIRA (2022) d) Para uma dada pressão P, as três raízes da equação de van der Waals em V coincidem a uma certa temperatura T = Tc. Determine Tc em termos de a e b, e use o resultado para mostrar que N2 é liquefeito mais prontamente do que o He. Dica do professor: Derivando 1 vez (dp/dv) e igualando a zero, temos: *'9 (%9 − (,)! = C(- %93 Derivando 2 vezes (d2p/dV2) e igualando a zero, temos: *'9 (%9 − (,)3 = E(- %9: 51 APOSTILA 01 DE FÍSICO-QUÍMICA – PROF. PEDRO MADEIRA (2022) CAPÍTULO 01 – O ESTUDO DOS GASES GABARITO TÓPICO 01 SEÇÃO VESTIBULARES 1. OPÇÃO D 2. n(NaN3) = 2,71 mol ou m(NaN3) = 176,15 g. 3. OPÇÃO C 4. t = 11 minutos. 5. OPÇÃO D 6. A) M = 131,5 g.mol–1 B) Xenônio. 7. OPÇÃO D 8. OPÇÃO C 9. OPÇÃO A 10. a) monofásico b) 0,90 atm; H2 < Ar < CO2 11. OPÇÃO B 12. OPÇÃO C 13. OPÇÃO E 14. OPÇÃO C 15. A) X(CH4) = 0,80 ou 80%; X(C2H6) = 0,20 ou 20%B) P(CH4) = 68%; P(C2H6) = 32% C) V = 16,6 m3. 16. A) e B) 17. OPÇÃO D 18. a) Curva que representa o melhor material para se armazenar o hidrogênio: curva B. Justificativa: O material A possui menor capacidade de armazenamento de H2, já os materiais B e C apresentam capacidades próximas, logo o material A é descartado. O material B apresenta uma menor distância entre as curvas de adsorção e dessorção, ou seja, neste caso a reversibilidade do processo de armazenamento e liberação de H2 é mais eficaz b) Uma possível desvantagem desta tecnologia alternativa estaria relacionada à massa ocupada pelo Mg2Ni. Conclusão: a porcentagem da massa do Mg2Ni é 9,25 vezes maior do que a da gasolina, sendo isto uma enorme desvantagem. 19. a) Íons OH – reagem com íons H+ da solução, deslocando o equilíbrio no sentido da dissolução do CO2. b) 40 kPa 20. OPÇÃO D 21. a) 113,83 kPa; b) 242,6g 22. a) No frasco 1, existirá a menor quantidade de moléculas; b) P2/P3 = 10 23. a) V(etileno) = 244 L; b) seria 224 L (o mesmo) 24. a) CO2 + 2 LiOH à Li2CO3 + H2O; b) 2,4kg de LiOH 25. a) 0,081 atm; b) 68378 kJ 26. a) 3SO2 + 2HNO3 + 2H2O à 2NO + 3H2SO4 b) 2,2g 27. a) O2, He, N2; b) P(He) = 2160kPa; c) M = 22,9g/mol 28. a) como a massa molar do metano é menor do que a massa molar média do ar, ele é menos denso e por isso ascende. P 1 / V Aumento da temperatura P T Diminuição do volume tgq = nRT tga =nR/V 52 APOSTILA 01 DE FÍSICO-QUÍMICA – PROF. PEDRO MADEIRA (2022) b) Não seria possível obter a mistura com a composição acima mencionada pela simples mistura desses gases. O ar tem, aproximadamente, 21% em volume de gás oxigênio. Se o metano fosse misturado ao oxigênio, este apresentaria uma porcentagem menor que 21% na mistura. 29. a) liberação de energia; b) V = 225 m3 30. a) P(O2) = 2,2x104 Pa está dentro da faixa aceitável. b) 4,2% em volume 31. a) 33,33 mL de água sanitária b) A combustão é mais rápida em oxigênio puro, pois no ar só há 21% de O2. 32. a) CnH2n+2 + (1,5n+0,5) O2 à n CO2 + (n+1) H2O b) Nas mesmas condições de temperatura e pressão,volumes iguais de gases quaisquer contêm o mesmo número de moléculas (ou mols). c) propano e butano. 33. a) Na2CO3(s)+H2SO4(aq) à H2O(l) + CO2(g) + Na2SO4(aq) b) Como a massa utilizada é a mesma e as substâncias possuem massas molares diferentes, o número de mol não é igual nos dois casos. Logo, a altura atingida pelo êmbolo não será a mesma, pois dependerá da quantidade de gás carbônico liberada. c) 34. OPÇÃO A 35. OPÇÃO B 36. OPÇÃO B 37. OPÇÃO B 38. OPÇÃO D 39. SOMA = 07 40. M = 58 g/mol 41. O ovo é empurrado para dentro da garrafa pela diferença de pressão existente. Quando a garrafa é aquecida, parte do gás escapa. Ao se retornar à temperatura original, a pressão é menor. 42. OPÇÃO A 43. SOMA = 25 44. SOMA = 05 45. OPÇÃO C 46. OPÇÃO B 47. SOMA = 34 48. a) H2PO4– + HCO3– → H2O + CO2(g) + HPO42– b) 4,58 g 49. OPÇÃO B PROF. PEDRO MADEIRA SEÇÃO ITA/IME 50. OPÇÃO C 51. OPÇÃO B 52. OPÇÃO E 53. OPÇÃO E 54. OPÇÃO E 55. QUESTÃO NULA 56. OPÇÃO D 57. OPÇÃO B 58. OPÇÃO D 59. OPÇÃO D 60. OPÇÃO C 61. OPÇÃO A 62. OPÇÃO B 63. OPÇÃO B 64. OPÇÃO D 65. OPÇÃO C 66. OPÇÃO C 67. OPÇÃO B 68. OPÇÃO D 69. OPÇÃO A 70. OPÇÃO B 71. OPÇÃO C 72. 1 Zn(s) + 2 HCl(aq) à 1 ZnCl2(aq) + 1 H2O(l) m(Zn) = 2,67 g 73. OPÇÃO A 74. V = 40,6 L 75. OPÇÃO C 76. OPÇÃO B (INCORRETA) 77. OPÇÃO D 78. d = 1,17 g.L–1. 79. OPÇÃO D 80. OPÇÃO D 2 m R Tx MM P r ´ ´ = ´ ´ ´π 53 APOSTILA 01 DE FÍSICO-QUÍMICA – PROF. PEDRO MADEIRA (2022) 81. OPÇÃO C 82. P(Mg) = 65,5% 83. A) V = 1,22 x 10–7 L B) m = 1,0 x 10–8 g 84. OPÇÃO D 85. OPÇÃO D 86. CaHb+(a+b/4)O2+(4a+b)N2àaCO2+b/2H2O+(4a+b)N2 87. X(C3H8) = 20% 88. OPÇÃO C 89. P = 120 atm 90. OPÇÃO D 91. OPÇÃO D 92. P(O2) = 4,41 atm; P(N2) = 16,4 atm 93. OPÇÃO D 94. OPÇÃO D 95. OPÇÃO D 96. OPÇÃO D 97. OPÇÃO E 98. A) 16 u B) CH4 99. d = 1,203 g.L–1 100. t = 5 semanas. 101. T = 250 K ou T = - 23oC 102. V = 95,0 cm3 103. (veja comentário) 104. m(Na) = 2,56 g 105. A) m(CO2) = 8,79 g B) m = 40 g 106. V = 7,61 L 107. Teor(Al) = 10,5% 108. OPÇÃO A 109. OPÇÃO E 110. OPÇÃO A 111. OPÇÃO D 112. OPÇÃO E 113. a) 2 HCl(conc) + Zn(s) à ZnCl2(aq) + H2(g) H2(g) + Br2(g) à 2 HBr(g) b) n(H2) = 0,015 mol 114. V = 100 cm3. 115. m = 454g deTNT 116. p = 1,97 atm 117. m(Mg) = 22g; m(Mg3N2) = 25g 118. OPÇÃO C 119. OPÇÃO B PROF. PEDRO MADEIRA TÓPICO 02 SEÇÃO VESTIBULARES 120. a) diminui a energia interna (pois diminui a temperatura). b) 0,91 atm 121. OPÇÃO A 122. SOMA 23 123. SOMA 03 124. OPÇÃO A 125. OPÇÃO A 126. OPÇÃO C 127. OPÇÃO D 128. a) NH3 pois possui menor massa molar b) o anel será formado em um tempo menor do que 10s c) Caso o algodão embebido de solução aquosa de NH3 seja colocado no tubo um pouco antes do algodão que libera HCl(g) (e não simultaneamente) o anel de NH4Cl será formado a uma distância maior da extremidade do algodão embebido com NH3 dando a impressão de que a velocidade de difusão do HCl é menor do que a verdadeira. De acordo com a expressão matemática fornecida, quanto menor a velocidade de difusão, maior a massa molar. Consequentemente, a massa molar do HCl parecerá maior do que a verdadeira. PROF. PEDRO MADEIRA SEÇÃO ITA / IME 129. OPÇÃO B 130. OPÇÃO A 131. OPÇÃO E 132. OPÇÃO B ( )- N A N 0 A B VP = P p + p V +V æ ö ç ÷ è ø 54 APOSTILA 01 DE FÍSICO-QUÍMICA – PROF. PEDRO MADEIRA (2022) 133. NH3(g) + HCl(g) à NH4Cl(s); v(NH3) > v(HCl) H(NP3) H(P5Q) = R ;(P5Q) ;(NP3) ≈ 2, IS (Lei de Graham) 134. OPÇÃO B 135. OPÇÃO D 136. OPÇÃO E 137. OPÇÃO E PROF. PEDRO MADEIRA TÓPICO 03 SEÇÃO VESTIBULARES 138. OPÇÃO D PROF. PEDRO MADEIRA SEÇÃO ITA / IME 139. OPÇÃO A 140. OPÇÃO C 141. OPÇÃO C 142. OPÇÃO A 143. Pontos A, B e C: início das liquefações em cada isoterma Pontos X, Y e Z: final das liquefações em cada isoterma 144. OPÇÃO E 145. Erro absoluto = - 1,2 atm; Erro relativo = - 5,1 % 146. OPÇÃO A 147. a) 10 kJ; b) 600K; Pi = 0,9 atm e PF = 1,9 atm. 148. è Tendo em mente a equação do gás ideal, pV = nRT, pode-se incluir as 2 correções propostas por van der Waals. è A correção no volume consiste em subtrair do volume total do recipiente, o volume ocupado pelas moléculas do gás: %; = %− ,′4 na qual Vb é o volume corrigido e b’ é uma constante característica de cada gás. è A pressão deve ser corrigida por um fator proporcional ao quadrado da concentração, de acordo com a diminuição da intensidade das colisões nas paredes do recipiente: = = =% − -. ( %/ ! ⟹=% = =+ -. ( %/ ! na qual “-” é uma constante característica de cada gás, P é a pressão do gás e Pa é a pressão original do gás antes da correção, a que deve ser aplicada na equação do gás ideal. è Logo, aplicando na equação do gás ideal: U= + -. ( %/ ! V (% − ,′4) = (*' ou = = (*' % − (, − -. ( %/ ! Na qual “-” e “,” são constantes características de cada gás. PROF. PEDRO MADEIRA TÓPICO 04 SEÇÃO VESTIBULARES 149. OPÇÃO A 150. OPÇÃO B 151. a) 0,16g; b) KClO3(s) à KCl(s) + 3/2 O2(g) 152. OPÇÃO A 153. a) A = NH3; B = CH4; C = CO2; D = SO2; E = CO b) Ca(OH)2 + CO2 CaCO3↓ + H2O Ca(OH)2 + SO2 CaSO3↓ + H2O c) Teremos uma reação de oxi-redução. Nesta transformação o Nox do manganês Mn (+7) diminui. Concluímos que ocorre descoramento da solução. 154. OPÇÃO D PROF. PEDRO MADEIRA SEÇÃO ITA / IME 155. OPÇÃO D 156. OPÇÃO A 157. OPÇÃO E 158. OPÇÃO D 159. OPÇÃO A ® ® 55 APOSTILA 01 DE FÍSICO-QUÍMICA – PROF. PEDRO MADEIRA (2022) 160. A nitroglicerina é uma substância líquida, incolor e oleosa, mas também altamente instável e explosiva. Basta um simples toque para que ocorra uma detonação espontânea. Como a molécula contém átomos de oxigênio, hidrogênio e carbono, quando ela explode libera novas moléculas (como O2, N2, H2 e CO2), mais estáveis, e com uma grande energia cinética, havendo uma conversão entre a energia de ligação para energia cinética. O poder dos explosivos deve-se ao fato de que o volume ocupado por um gás é muito maior do que o ocupado por um sólido ou por um líquido. A reação de decomposição da nitroglicerina é: 4 C3H5(ONO2)3(!) à 6N2(g) + 12CO2(g) + 10 H2O(g) + 1 O2(g) 161. OPÇÃO D PROF. PEDRO MADEIRA QUESTÕES EXTRAS: OLIMPÍADA 162. OPÇÃO A 163. OPÇÃO D 164. OPÇÃO D 165. OPÇÃO A 166. OPÇÃO B 167. OPÇÃO A 168. OPÇÃO D 169. OPÇÃO A 170. OPÇÃO B 171. OPÇÃO C 172. OPÇÃO B 173. OPÇÃO C 174. OPÇÃO B 175. OPÇÃO C 176. PARTE I A) M = 34 g.mol–1 B) Frasco B (o dobro è Princípio de Avogadro) C) mesma Ec (mesma Temperatura è Teoria cinética dos gases ideiais) D) v(O2) > v(H2S); 177. A) P(NO2) = 2,058 x 10–8 atm B) N = 2,39 x 1019 moléculas C) c.1) 2 NO2(g) + H2O(l) à HNO2(aq) + HNO3(aq) c.2) NO +hn à NO+ + e– c.3) NO + O3 à NO2 + O2 NO+ + e– à N + O NO + hn à N + O PROF. PEDRO MADEIRA QUESTÃO COMENTADA 103. (IME – 2004) RESOLUÇÃO FORMAL: i) Antes da abertura da válvula 1: Após N = 1: Após N = 2: Após N = N: ii) Chama-se e iii) OBSERVE A SOMA: iv) DIVINDO-SE A 1a EQUAÇÃO POR K, A 2a por K2, A N-ésima por KN: ........eq.01 2 2 2 2 v(O ) M(H S) = = 1,03 v(H S) M(O ) RT pV n RT VP n B B A A == ;0 0, ( ) BA B BA A BA BA VV pV VV VP P VV RT RT pV RT VP P + + + =Þ +÷÷ ø ö çç è æ += 0 1 0 1 ( ) BA B BA A BA BA VV pV VV VP P VV RT RT pV RT VP P + + + =Þ +÷÷ ø ö çç è æ += 1 2 1 2 RT pV n RT VP n B B A A == ;1 1, BA B BA AN N VV pV VV VP P + + + = -1 K VV V BA A = + C VV pV BA B = + 1 0 2 1 1 2 1 . . . . N N N N P P K C P P K C P P K C P P K C - - - = + + = + + + = + + = + ! 1 0 2 1 2 2 1 2 1 2 1 1 1 N N N N N N N N N N P CP K KP P C KK K P P C K K K P P C K K K - - - - - - - = + + = + + + = + + = + ! ÷ ø ö ç è æ +++++= - NNN N KKKK CP K P 1111 120 ! 56 APOSTILA 01 DE FÍSICO-QUÍMICA – PROF. PEDRO MADEIRA (2022) v) Obs: Soma de uma PG de razão q = 1/K e 1º termo a1 = 1/K: eq.02 vi) SUBSTITUINDO A EQ.02 NA EQ.01 PROF. PEDRO MADEIRA “Quem não perserverar nas pequenas tarefas, falhará nos grandes planos.” Provérbio Chinês ( )1 1 11 1.11 1111 12 - - = - - =÷ ø ö ç è æ ++++ - KK K K KKK KKKK N NN NN ! ( ) 1 1 1 1 00 - - +=Þ - - += K KCPKP KK KCP K P N N NN N N N Þ - + - ´ + +=Þ 1 1 0 BA A N BA BN N VV V K VV pV PKP BA B N BA BN N VV V K VV pV PKP + - - ´ + += 1 0 ( )10 -´-=Þ NN N KpPKP ( ) N N KpPpP -+=Þ 0 ( ) N BA A N VV V pPpP ÷÷ ø ö çç è æ + ´-+= 0 57 APOSTILA 01 DE FÍSICO-QUÍMICA – PROF. PEDRO MADEIRA (2022) CAPÍTULO 02 – TERMODINÂMICA QUÍMICA CONTEÚDO TÓPICO 01: CONCEITOS FUNDAMENTAIS TÓPICO 02: PRIMEIRA LEI DA TERMODINÂMICA TÓPICO 03: SEGUNDA E TERCEIRA LEIS DA TERMODINÂMICA TÓPICO 04: FUNDAMENTOS DE TERMOQUÍMICA TÓPICO 01: CONCEITOS FUNDAMENTAIS A) Sistema x vizinhança O sistema é a porção do universo que se tem como objeto de análise. A vizinhança é tudo aquilo que não é o sistema. Eles são separados pela fronteira que é classificada de acordo com a capacidade de transferência de calor. PROF. PEDRO MADEIRA B) Tipos de Fronteira (a) permite a passagem de calor = diatérmica (b) não permite a troca de calor = adiabática PROF. PEDRO MADEIRA C) Tipos de Sistema (a) permite a troca de matéria e energia. Ex: garrafa de refrigerante aberta. (b) só permite a troca de energia. Ex: garrafa de refrigerante fechada. (c) não permite a troca nem de matéria nem de energia. Ex: frasco de Dewar. RESOLVA AGORA 01 (IChO PP 2012) Na natureza, os elementos radioativos de longa vida (Th e U) dão origem a isótopos radioativos de vida curta. Se o decaimento nuclear ocorrer em um sistema fechado, as atividades dos núcleos filhos se tornam iguais à atividade do núcleo-mãe. Isto indica que outros processos além do decaimento radioativo afetam a abundância dos núcleos-filho. Na água de um lago, a radioatividade do 222Rn dissolvido (t1/2 = 3,8 dias) é de 4,2 átomos/min.(100L). Todos os 222Rn são produzidos a partir do 226Ra dissolvido (t1/2 = 1600 anos), o qual tem uma atividade de 6,7 átomos.min–1.(100L)–1. Isto implica que parte do 222Rn produzido está sendo perdido a partir do lago por um processo desconhecido. Baseado no conceito de sistema, explique porque a atividade do núcleo-filho não é igual à atividade do núcleo-mãe. PROF. PEDRO MADEIRA 58 APOSTILA 01 DE FÍSICO-QUÍMICA – PROF. PEDRO MADEIRA (2022) D) Processo endotérmico x Processo exotérmico OBSERVAÇÕES RESOLVA AGORA 02 (IME 2009) Foram introduzidos 10 mols de uma substância X no interior de um conjunto cilindro-pistão adiabático, sujeito a uma pressão constante de 1atm. X reage espontânea e irreversivelmente segundo a reação: X(s) à 2 Y(g) DH = – 200 cal Considere que a temperatura no início da reação é 300 K e que as capacidades caloríficas molares das substâncias X e Y são constantes e iguais a 5,0 cal.mol– 1.K–1 e 1,0 cal.mol–1.K–1, respectivamente. O volume final do conjunto cilindro-pistão é (Dado: R = 0,082 atm.L.mol–1.K–1) A ( ) 410,0 L B ( ) 492,0 L C ( ) 508,4 L D ( ) 656,0 L E ( ) 820,0 L EXO tempo ENDO tempo 59 APOSTILA 01 DE FÍSICO-QUÍMICA – PROF. PEDRO MADEIRA (2022) RESOLVA AGORA 03 (IME 2010) A transformação isovolumétrica de um gás triatômico hipotético A3 em outro diatômico A2 envolve a liberação de 54 kJ/mol de A3. A capacidade calorífica molar a volume constante do gás A2 é de 30 J/mol.K. Após a transformação isocórica de todo A3 em A2, determine o aumento percentual de pressão em um recipiente isolado contendo o gás A3 a 27ºC. Considere que a capacidade calorífica molar a volume constante do gás A2 não varia com a temperatura e que os gases se comportam idealmente. PROF. PEDRO MADEIRA RESOLVA AGORA 04 (IME 2021 – Q08) Um motor de 6 cilindros e volume total de 5.700 cm3, utilizado em viaturas leves e blindadas, consome 0,5 g do combustível gasoso de composição média C8H18, em cada cilindro, por segundo de operação. Considerações: • o ciclo termodinâmico do motor compreende o funcionamento em 4 tempos: admissão, compra assim combustão e exaustão (escape); • o motor executa 10 ciclos por segundo, ou seja, a mistura de ar e combustível enche o cilindros e depois é comprimida 10 vezes por segundo; • 20,0% da quantidade de combustível sofre combustão incompleta, sendo convertida em CO(g); • 80,0% da quantidade de combustível sofre combustão completa, sendo convertida em CO2(g); • a mistura de ar e combustível comporta-se como gás ideal; • as capacidades caloríficas molares são independentes da temperatura; e • as entalpias de formação a 25ºC. Determine: a) a vazão da entrada de ar no motor, em m3/s; e b) a composição percentual molar dos produtos e a temperatura de combustão, em K. FOLHA DE DADOS • Composição percentual do ar atmosférico = 79,0% de N2(g) e 21,0% de O2(g) Entalpias-padrão de formação a 25ºC: Substância Química C8H18(g) H2O(g) H2O(l) CO2(g) CO(g) DHf o (kJ.mol–1) -208,45 -241,82 -285,83 -393,51 -110,53 Capacidade calorífica molar: Substância Química N2(g) O2(g) H2O(g) CO2(g) CO(g) "#! (J.K–1 .mol–1) 29,13 29,36 33,58 37,11 29,14 ar e combustível motor 6 cilindros produtos trabalho 60 APOSTILA 01 DE FÍSICO-QUÍMICA – PROF. PEDRO MADEIRA (2022) E) Trabalho, calor e energia • Fazer trabalho é equivalente a elevar um peso em algum lugar nas vizinhanças. • A energia de um sistema é a sua capacidade de efetuar trabalho. • Calor à gradiente de temperatura. • Interpretação molecular: calor x trabalho PROF. PEDRO MADEIRA OBSERVAÇÕES F) Funções de estado x funções de linha (caminho) Energia interna, entalpia, entropia, energias livre (Gibbs e Helmholtz) = dependem só do estado. Calor e trabalho = dependem do caminho. PROF. PEDRO MADEIRA OBSERVAÇÕES 61 APOSTILA 01 DE FÍSICO-QUÍMICA – PROF. PEDRO MADEIRA (2022) RESOLVA AGORA 05 (ITA 2022 – Q67) Considere as seguintes afirmações sobre processos termodinâmicos, que podem ocorrer em uma ou mais etapas, em que DT se refere à variação de temperatura entre os estados inicial e final: I. Um processo termodinâmico é definido pelo estado final e estado inicial do sistema. II. DT é sempre nula em um processo isotérmico. III. A troca de calor envolvida em um processo isotérmico deve ser nula (q=0). IV. Todo processo em que DT = 0 é um processo isotérmico. V. DT = 0 para todo processo em sistema isolado. Assinale a opção que contém as afirmações ERRADAS: A ( ) Apenas I, II e IV B ( ) Apenas I, III, IV e V C ( ) Apenas I, III e V D ( ) Apenas II e IV E ( ) III e V G) A Distribuição de Boltzmann e a constante de Boltzmann OBSERVAÇÕES PROF. PEDRO MADEIRA • Distribuição de Maxwell-Boltzmann (velocidades gasosas). • Distribuição barométrica (distribuição gravimétrica). • Equação de Arrhenius. 23 1; 1,38 10 . ; i i E kT i AE kT i N e k J K R N k N e - -- -= = × = å 62 APOSTILA 01 DE FÍSICO-QUÍMICA – PROF. PEDRO MADEIRA (2022) TÓPICO 02: PRIMEIRA LEI DA TERMODINÂMICA A) Energia Interna • Energia Interna (U): soma das energias cinética e potencial que compõem o sistema. • U é função de estado e propriedade extensiva. • Unidades de energia: J, eV, cal. OBSERVAÇÕES PROF. PEDRO MADEIRA B) Teorema da Equipartição de Energia (Mecânica clássica) • Se então Ek = 3/2 kT (gás monoatômico). • Na prática, o teorema se aplica somente para translação e rotação moleculares. • Um = Um(0) + 3/2 RT. • Uma vez que Ep (interação) = 0, então a energia interna de um gás perfeito não depende do volume. OBSERVAÇÕES PROF. PEDRO MADEIRA ( )2 2 2 x y z 1E = m v +v +v ,k 2 63 APOSTILA 01 DE FÍSICO-QUÍMICA – PROF. PEDRO MADEIRA (2022) C) A conservação da energia • A energia interna de um sistema isolado é constante. • Até hoje não se conseguiu se construir um “moto- perpétuo de primeira espécie”. • PLT: DU = q + w (convenção aquisitiva) OBSERVAÇÕES PROF. PEDRO MADEIRA D) Trabalho de expansão volumétrica OBSERVAÇÕES RESOLVA AGORA 06 (ITA 2008) Dois cilindros (I e II) são providos de pistões, cujas massas são desprezíveis e se deslocam sem atrito. Um mol de um gás ideal é confinado em cada um dos cilindros I e II. São realizados, posteriormente, dois tipos de expansão, descritos a seguir: a) No cilindro I, é realizada uma expansão isotérmica à temperatura T, de um volume V até um volume 2V, contra uma pressão externa constante P. b) No cilindro II, é realizada uma expansão adiabática, de um volume V até um volume 2V, contra uma pressão externa constante P. Determine os módulos das seguintes grandezas: variação da energia interna, calor trocado e trabalho realizado para os dois tipos de expansão. PROF. PEDRO MADEIRA CASOS ESPECÍFICOS DE EXPANSÃO • Expansão Livre (pex = 0): w = 0 • Expansão contra pressão constante: w = – pex.DV • Expansão reversível (pex = p): Isotérmica: w = – nRT ln(VF/Vo) Adiabática: w = – n.Cv.DT 64 APOSTILA 01 DE FÍSICO-QUÍMICA – PROF. PEDRO MADEIRA (2022) E) Expansão Reversível x Expansão Irreversível Observe os diagramas de James-Watt: OBSERVAÇÕES 1º CASO: 2º CASO: 3º CASO: 65 APOSTILA 01 DE FÍSICO-QUÍMICA – PROF. PEDRO MADEIRA (2022) F) Entalpia • H = U + pV (por definição) è DH = qp (p constante, sem trabalho extra). • H é função de estado e propriedade extensiva. OBSERVAÇÕES PROF. PEDRO MADEIRA G) Calorimetria RESOLVA AGORA 07 (IME 2009) O valor experimental para o calor liberado na queima de benzeno líquido a 25oC, com formação de dióxido de carbono e água líquida, é 780 kcal/mol. A combustão é feita em uma bomba calorimétrica a volume constante. Considerando comportamento ideal para os gases formados e R = 2,0 cal/mol.K, determine: a) o calor padrão de combustão do benzeno a 25oC; b) se o calor calculado no item anterior é maior ou menor quando a água é formada no estado gasoso. Justifique sua resposta. PROF. PEDRO MADEIRA H) Capacidades Caloríficas Capacidade calorífica a volume constante, Cv • Cv é propriedade extensiva. • Cv,m é propriedade intensiva. Ex.: Cv,m = 12,47 J K–1mol–1 (gases monoatômicos) • OBS: capacidade calorífica específica. Ex.: c = 4 J K–1g–1 (H2O) • OBS: capacidade calorífica infinita = transição de fases. • dU = CV dT à DU = CV DT à qV = CV DT (intervalo pequeno de temperatura à CV constante). BOMBA CALORIMÉTRICA (V = constante) • dU = dq + dwexp + dwe • dU = dq (V constante, sem we) DU = qV • Calorimetria Bomba calorimétrica (adiabática) q = CDT; C = constante do calorímetro. CALORÍMETRO ISOBÁRICO • Calorímetro de chama adiabático (combustão). • Para reações envolvendo sólidos e líquidos, DU é praticamente igual ao DH. • Para reações envolvendo gases: • DH = DU + PDV = DU + DngasesRT 66 APOSTILA 01 DE FÍSICO-QUÍMICA – PROF. PEDRO MADEIRA (2022) • Gráficos típicos. (Em geral, quanto menor a temperatura, menor Cv): Capacidade calorífica a pressão constante, CP • dH = CP dT à DH = CP DT à qP = CP DT (intervalo pequeno de temperatura à CP constante). • Cp,m = a + bT + cT–2 (a,b,c são parâmetros empíricos). • devido ao trabalho de expansão, o aquecimento é menor a pressão constante do que a volume constante. CP > CV Relação de Meyer: CP – CV = nR (gás perfeito) OBSERVAÇÕES RESOLVA AGORA 08 (ITA 2009) Nos gráficos abaixo, cada eixo representa uma propriedade termodinâmica de um gás que se comporta idealmente. Com relação a estes gráficos, é CORRETO afirmar que A ( ) I pode representar a curva de pressão versus volume. B ( ) II pode representar a curva de pressão versus inverso do volume. C ( ) II pode representar a curva de capacidade calorífica versus temperatura. D ( ) III pode representar a curva de energia interna versus temperatura. E ( ) III pode representar a curva de entalpia versus o produto da pressão pelo volume. PROF. PEDRO MADEIRA P P HC = T ¶æ ö ç ÷¶è ø V V UC = T ¶æ ö ç ÷¶è ø 67 APOSTILA 01 DE FÍSICO-QUÍMICA – PROF. PEDRO MADEIRA (2022) I) Transformação Adiabática / Politrópica • q + w = DU à DU = wad • Como U é função de estado, DU = CV DT • wad = CV DT • Outras relações: e • Gráfico T x V para a transformação adiabática: • Adiabática x Isoterma: Prof. Pedro Madeira RESOLVA AGORA 09 Deduza a expressão para o trabalho de uma expansão adiabática reversível de um gás com coeficiente de Poisson igual a g. O volume e a pressão inicial são Vo e Po e o volume e a pressão final é VF e PF. OBSERVAÇÕES: VC R oF o F VT = T V æ ö ç ÷ è ø o o F FP V =P Vg g 68 APOSTILA 01 DE FÍSICO-QUÍMICA – PROF. PEDRO MADEIRA (2022) TÓPICO 03 – SEGUNDA E TERCEIRA LEIS DA TERMODINÂMICA A) O Enunciado de Kelvin “Não é possível um processo que tenha como único resultado a absorção de calor de um reservatório térmico e a sua completa conversão em trabalho” OBSERVAÇÕES PROF. PEDRO MADEIRA B) O sentido da transformação espontânea • Observações experimentais: à Uma bola sobre uma superfície quente jamais principia, espontaneamente, a pular sobre ela. à Um gás não se contrai espontaneamente. à Um corpo não fica espontaneamente mais quente do que as suas vizinhanças. • Justificativa: As mudanças espontâneas são sempre acompanhadas pela dispersão mais desordenada da energia. A localização de movimentos caóticos num único movimento ordenado é extremamente IMPROVÁVEL. PROF. PEDRO MADEIRA C) Entropia • A primeira lei aproveita-se da energia interna para identificar as mudanças permitidas; a segunda lei usa a entropia para identificar as mudanças espontâneas entre as mudanças permitidas. • A entropia de um sistema isolado aumenta numa mudança espontânea: DStot = DSsis + DSviz ≥ 0 Definição Termodinâmica: • O calor proporciona um aumento do movimento caótico das partículas das vizinhanças do sistema. • A vizinhança pode ser encarada como um reservatório a volume constante (dqviz = dUviz)ou a pressão constante (dqviz = dHviz). Em ambos os casos, uma função de estado. Logo, OBSERVAÇÕES PROF. PEDRO MADEIRA ò=DÞ= f i revrev T dqS T dqdS viz viz viz viz viz viz dq q dS S T T = ÞD = 69 APOSTILA 01 DE FÍSICO-QUÍMICA – PROF. PEDRO MADEIRA (2022) D) A entropia como função de estado Ciclo de Carnot Reversível: 1) Expansão isotérmica è DS = qh/Th 2) Expansão adiabática è DS = 0 3) Compressão isotérmica è DS = qc/Tc 4) Compressão adiabática è DS = 0 DSciclo = 0 • A eficiência do ciclo de Carnot: e = |w| / qQ e = 1 + (qF / qQ) e = 1 – (TF / TQ) Conclusão da Segunda Lei: todas as máquinas reversíveis têm a mesma eficiência, qualquer que seja o seu modo de operar. OBSERVAÇÕES RESOLVA AGORA 10 (ITA 2010) Uma maquina térmica opera segundo o ciclo JKLMJ mostrado no diagrama T-S da figura. Pode-se afirmar que A ( ) o processo JK corresponde a uma compressão isotérmica. B ( ) o trabalho realizado pela maquina em um ciclo é W = (T2 – T1)(S2 – S1). C ( ) o rendimento da maquina e dado por h = 1 – (T2/T1). D ( ) durante o processo LM uma quantidade de calor QLM = T1(S2 – S1) é absorvida pelo sistema. E ( ) outra maquina térmica que opere entre T2 e T1 poderia eventualmente possuir um rendimento maior que a desta. PROF. PEDRO MADEIRA 70 APOSTILA 01 DE FÍSICO-QUÍMICA – PROF. PEDRO MADEIRA (2022) E) A Temperatura termodinâmica • T = (1 – e)Th • O Zero da escala ocorre quando a eficiência de uma máquina de Carnot é 1. (Definido por Kelvin) • Atribui-se ao ponto triplo da água o valor de 273,16 K. Então, se uma máquina tem sua fonte quente no ponto triplo da água, a temperatura do sumidouro frio é encontrada pela determinação da eficiência da máquina. OBSERVAÇÕES PROF. PEDRO MADEIRA F) Variação de entropia de alguns processos • Transição de fase na temperatura de transição: DtrsS = DtrsH / Ttrs OBS.: Regra de Trouton (empírica) = muitos líquidos têm DvapSo » 85 J/K.mol OBS2: DfusSo (metais) = R OBSERVAÇÕES: • Expansão isotérmica de um gás perfeito DS = nR ln(VF / Vo) • Variação da entropia com a temperatura ( ) ( ) ( ) ( )F F o o T T prev F o F oT T C dTdqS T = S T + S T = S T + T T Þò ò 71 APOSTILA 01 DE FÍSICO-QUÍMICA – PROF. PEDRO MADEIRA (2022) • Variação da entropia entre dois pontos: • A medição da entropia das substâncias No gráfico de CP/T versus T, as áreas hachuradas correspondem às variações de entropia de aquecimento da respectiva fase de agregação: Prof. Pedro Madeira OBSERVAÇÕES PROF. PEDRO MADEIRA f eb f eb T p fus 0 f T Tp peb T Teb C (s)dT Δ HS(T) = S(0) + + + T T C (l)dT C (g)dTΔ H + + T T T + ò ò ò 72 APOSTILA 01 DE FÍSICO-QUÍMICA – PROF. PEDRO MADEIRA (2022) G) Terceira lei da Termodinâmica • O teorema do calor de Nernst: “A variação de entropia de qualquer transformação física ou química tende a zero quando a temperatura tende a zero: DS à 0 quando T à 0, admitindo-se que todas as substâncias envolvidas estão ordenadas perfeitamente” Exemplo: S(a) à S(b) DtrsS (369K) = (402 J.mol–1) / 369 K = 1,09 J.K–1.mol–1 S(a,369K) = S(a,0K) + 37 J.K–1 .mol–1 (pelo Cp) S(b,369K) = S(b,0K) + 38 J.K–1 .mol–1 (pelo Cp) S(a,OK) – S(b,0K) » 0 • Conclusão: todos os cristais perfeitos a 0 K têm a mesma entropia. • Todos os cristais perfeitos em T = 0 K, têm entropia ZERO (fixado arbitrariamente) • Entropias da terceira lei = So(T) [J/K.mol] Gases: So(298,15K)/R 1) Entropia x Massa He = 15,1591; Ne = 17,5856; Ar = 18,6101 HF = 20,8872; HCl = 22,4653; HBr = 23,8872 2) Entropia x Capacidade Calorífica (mesma massa) Ne = 17,5856; HF = 20,8872; H2O = 22,6984 3) Entropia x Simetria Molecular (mesma massa e mesma capacidade calorífica). N2 = 23,0325; CO = 23,7607 NH3 = 23,173; CH4 = 22,389 RESOLVA AGORA 11 a) Suponha que há quatro formas diferentes da molécula CFClBrI se orientar no espaço em um cristal a zero Kelvin. Qual é a sua entropia residual molar? b) Calcule a entropia residual da mistura metaestável de orto- e para-hidrogênio (abundância de 25% de p- H2), sabendo que as moléculas de para-hidrogênio atingem o menor estado rotacional J = 0 (1 possibilidade energética), enquanto o orto-hidrogênio assume J = 1 (degenerescência 3). OBSERVAÇÕES: Entropias a 0K So(HCl) = 0 J/mol.K So(CO) = 4,6 J/mol.K So(N2O) = 6 J/mol.K So(H2) = 6,66 J/mol.K So(FClO3) = 10,1 J/mol.K 73 APOSTILA 01 DE FÍSICO-QUÍMICA – PROF. PEDRO MADEIRA (2022) H) Espontaneidade e funções do sistema • Se V e T são constantes, uma transformação espontânea deve satisfazer a desigualdade: Se p e T são constantes, de forma análoga: Energias Livres: > de Helmholtz è A = U – TS > de Gibbs è G = H – TS dA = dU – TdS (isotérmico) dG = dH – TdS (isotérmico) • Critério para espontaneidade: dAT,V £ 0 e dGT,p £ 0 “A temperatura e pressão constantes, as reações químicas são espontâneas no sentido da diminuição da energia de Gibbs” OBS.: O significado de DG, além de indicar o sentido da transformação espontânea, consiste no valor do trabalho máximo possível diferente do de expansão. • Cálculo do DGo: DGo = DHo – TDSo RESOLVA AGORA 12 (ITA 2006) Para cada um dos processos listados abaixo, indique se a variação de entropia será maior, menor ou igual a zero. Justifique suas respostas. a) N2 (g,1 atm,T = 300 K) → N2 (g,0,1 atm,T = 300 K) b) C (grafite) → C(diamante) c) solução supersaturada → solução saturada d) sólido amorfo → sólido cristalino e) N2 (g) → N2 (g, adsorvido em sílica) 0dU dUdS - dS TdS dU dU - TdS 0 T T ³ Þ ³ Þ ³ Þ £ 0dH dHdS - dS TdS dH dH - TdS 0 T T ³ Þ ³ Þ ³ Þ £ 74 APOSTILA 01 DE FÍSICO-QUÍMICA – PROF. PEDRO MADEIRA (2022) PROF. PEDRO MADEIRA TÓPICO 04 – TERMOQUÍMICA • É um ramo da Termodinâmica que estuda o calor envolvido nas reações químicas e nas transições de fase. • Reação Endotérmica: DH > 0 Reação Exotérmica: DH < 0. A) Variações de entalpia padrão • O estado padrão de uma substância, numa certa temperatura, é o da substância na sua forma pura sob pressão de 1 bar. • Ex.: H2O(l) à H2O(g) DvapHo (373K) = + 40,66 kJ.mol– 1. • OBS: A temperatura adotada para o registro dos dados termodinâmicos é 298,15 K ou (25,00oC). PROF. PEDRO MADEIRA B) Entalpias de transformações físicas • Entalpia padrão de transição. DtrsHo • Entalpia padrão de fusão. DfusHo • Entalpia padrão de vaporização. DvapHo • Entalpia padrão de sublimação. • DsubHo = DfusHo + DvapHo (medidos na mesma temperatura, pois H é função de estado). OBSERVAÇÕES PROF. PEDRO MADEIRA 75 APOSTILA 01 DE FÍSICO-QUÍMICA – PROF. PEDRO MADEIRA (2022) C) Entalpias de transformações químicas • Equação Termoquímica. CH4(g) + 2O2(g) à CO2(g) + 2 H2O(l) DHo = – 890 kJ Reagentes isolados, puros, nos estados padrões à produtos isolados, puros, nos estados padrões. OBS.: Exceto no caso de reações iônicas em solução, DmixH e DsepH são insignificantes em relação ao DrH. • Entalpia padrãode reação: OBSERVAÇÕES PROF. PEDRO MADEIRA D) Referencial zero de entalpia e Entalpias padrão de formação • Estado de referência de um elemento: estado mais estável, numa certa temperatura, sob pressão de 1 bar. • OBS.: Alotropia. • O2(g); C(s,grafita); S8(s,rômbico). • Exceção: Fósforo branco é o estado de entalpia zero (apesar de ser o alótropo metaestável), pois é a forma mais reprodutível e fácil de se obter. OBSERVAÇÕES RESOLVA AGORA 13 (ITA 2008) Assinale a opção ERRADA que apresenta (em kJ/mol) a entalpia padrão de formação (ΔHf) da substância a 25oC. A ( ) ΔHf (H2(g)) = 0 B ( ) ΔHf (F2(g)) = 0 C ( ) ΔHf (N2(g)) = 0 D ( ) ΔHf (Br2(g)) = 0 E ( ) ΔHf (Cl2(g)) = 0 o r produtos reagentes produtos reagentes ΔH = H - Hu u× ×å å 76 APOSTILA 01 DE FÍSICO-QUÍMICA – PROF. PEDRO MADEIRA (2022) E) A entalpia como função de estado Lei de Hess RESOLVA AGORA 14 (IME 2018) A reforma com vapor d`água, a temperaturas altas, é um método industrial para produção de hidrogênio a partir de metano . Calcule a entalpia de reação desse processo. Dados: i) Entalpias de combustão: C(grafite) ... ΔHo = -394 kJ/mol H2(g) ... ΔHo = -286 kJ/mol (forma água líquida) CH4(g) ... ΔHo = -890 kJ/mol (forma água líquida) ii) CO(g) + H2(g) → C(grafite) + H2O (g) ΔHo = -131 kJ/mol Entalpia de fusão do gelo: ΔHfus = 330 kJ·kg–1 Capacidade calorífica específica média da água: CV = 4,2 kJ·kg–1·K–1 RESOLVA AGORA 15 Monte um ciclo de Born-Haber para o NaCl e calcule a sua energia reticular U. Dados para o Na (kJ/mol): DsubHo = 108; EI = 496 Dados para o Cl (kJ/mol de Cl): DdissHo = 122; AE = 349 Dados para o NaCl (kJ/mol): DfHo = - 411. PROF. PEDRO MADEIRA 77 APOSTILA 01 DE FÍSICO-QUÍMICA – PROF. PEDRO MADEIRA (2022) F) Métodos de cálculo de ΔH • A partir dos DHf o • A partir das energias de ligação Observe as energias de ligação: HX BX3 AlX3 CX4 X2 F 566 645 582 439 159 Cl 431 444 427 347 243 Br 366 368 360 276 193 I 299 272 285 238 151 Ligação Entalpia (kJ mol–1) Ligação Entalpia (kJ mol–1) H–H 436 C–O 360 H–O 464 C=O 799 N–N 163 C≡O 1074 N=N 514 C–C 348 N≡N 946 C–C (C6H6) 518 C–H 413 C=C 612 N–H 389 C≡C 837 O=O 498 N–O 210 O–O 134 N=O 630 OBSERVAÇÕES o o o r f produtos f reagentes produtos reagentes ΔH = H - Hu u×D ×Då å o r reagentes produtos reagentes produtos ΔH = EL - ELn n× ×å å 78 APOSTILA 01 DE FÍSICO-QUÍMICA – PROF. PEDRO MADEIRA (2022) • Lei de Hess. • Contribuição de grupos termoquímicos. Exemplo: n-hexano(l). DvapH = 28,9 kJ.mol–1. Grupo DfHo / kJ.mol–1 C(H)3(C) - 42,17 C(H)2(C)2 - 20,7 C(H)(C)3 - 6,91 C(C)4 + 8,16 DfH(método dos grupos de Benson) = - 196 kJ.mol–1 DfH (experimental) = - 198,7 kJ.mol–1 RESOLVA AGORA 16 (ITA 2001) A 25ºC e pressão de 1 atm, a queima completa de um mol de n-hexano produz dióxido de carbono e água no estado gasoso e libera 3883 kJ, enquanto que a queima completa da mesma quantidade de n-heptano produz as mesmas substâncias no estado gasoso e libera 4498 kJ. A) Escreva as equações químicas, balanceadas, para as reações de combustão em questão. B) Utilizando as informações fornecidas no enunciado desta questão, faça uma estimativa do valor do calor de combustão do n-decano. Deixe claro o raciocínio utilizando na estimativa realizada. C) Caso a água formada na reação de combustão estivesse no estado líquido, a quantidade de calor liberado seria MAIOR, MENOR OU IGUAL a 3883 kJ? Por quê? 79 APOSTILA 01 DE FÍSICO-QUÍMICA – PROF. PEDRO MADEIRA (2022) RESOLVA AGORA 17 (IME 2010) Uma dada massa de óxido ferroso é aquecida a 1273 K e, em seguida, exposta a uma mistura gasosa de monóxido de carbono e hidrogênio. Desta forma, o óxido é reduzido a metal sem qualquer fornecimento adicional de energia. Admita que ocorra uma perda de calor para as circunvizinhanças de 4,2 kJ/mol de óxido reduzido. Calcule a razão mínima entre as pressões parciais de monóxido de carbono e de hidrogênio (pCO/pH2) na mistura gasosa inicial, de modo que o processo seja auto-sustentável. Despreze a decomposição da água. Calores de reação a 1273 K (kJ/mol) redução do óxido ferroso 265 oxidação do hidrogênio – 250 oxidação do monóxido de carbono – 282 PROF. PEDRO MADEIRA 80 APOSTILA 01 DE FÍSICO-QUÍMICA – PROF. PEDRO MADEIRA (2022) G) Tipos específicos de reação Transição Processo Símbolo Transição fase a à fase b DtrsH Fusão s à ℓ DfusH Vaporização ℓ à g DvapH Sublimação s à g DsubH Misturação de fluidos puro à mistura DmixH Dissolução soluto à solução DsolH Hidratação X±(g) à X±(aq) DhidH Atomização espécies (s,ℓ,g) à átomos (g) DatH Ionização X(g)àX+(g) + e–(g) DionH (EI) Afinidade eletrônica X(g) + e–(g)à X–(g) DegH (AE) Reação reagentesàprodutos DrH Combustão composto(s,ℓ,g)+O2(g) à CO2(g) + H2O(ℓ,g) DcH Formação elementosàcomposto DfH Neutralização H+ + OH– à H2O DH OBSERVAÇÕES PROF. PEDRO MADEIRA 81 APOSTILA 01 DE FÍSICO-QUÍMICA – PROF. PEDRO MADEIRA (2022) RESOLVA AGORA 18 (ITA 2022 – Q07) Considere as seguintes informações: I. Primeira energia de ionização do cálcio: 590 kJ.mol–1 II. Segunda energia de ionização do cálcio: 1145 kJ.mol–1 III. Afinidade eletrônica do cloro: -340 kJ.mol–1 IV. Entalpia de solubilização do cloreto de cálcio: -81 kJ.mol–1 V. Entalpia de hidratação do íon de cálcio: -1579 kJ.mol–1 V. Entalpia de hidratação do íon de cloro: -378 kJ.mol–1 Com base nessas informações, responda os itens abaixo. a) Represente, na forma de equações químicas, as informações acima (I-VI). b) Equacione a reação de entalpia de rede do cloreto de cálcio a partir das equações I-VI, conforme a necessidade. c) Calcule o valor numérico da entalpia de rede do cloreto de cálcio (em kJ.mol–1). H) Casos Especiais Energia de hidratação de íons DhidHo kJ/mol DhidHo kJ/mol DhidHo kJ/mol H+ - 1090 Mg2+ - 1920 OH– - 460 Li+ - 520 Ca2+ - 1650 F– - 506 Na+ - 405 Sr2+ - 1480 Cl– - 364 K+ - 321 Ba2+ - 1360 Br– - 337 Rb+ - 300 Fe2+ - 1950 I– - 296 Cs+ - 277 Cu2+ - 2100 Zn2+ - 2050 Ag+ - 464 Al3+ - 4690 NH4 + - 301 Fe3+ - 4430 Energia de isômeros DHo f (equatorial) < DHo f (axial) OBSERVAÇÕES 82 APOSTILA 01 DE FÍSICO-QUÍMICA – PROF. PEDRO MADEIRA (2022) Interconversão entre isômeros (Cinética) Processo necessário para interconverter enanciômeros ou diastereoisômeros Barreira de energia kcal/mol Velocidade de conversão a 25oC Rompimento de ligações s 50 – 100 Nenhuma Rompimento de ligações p (rotação restrita em torno de ligações duplas) 30 – 50 Nenhumaou muito pouca Rotação restrita em torno de ligações simples (impedimento estérico) 15 – 30 Muito pouca ou pouca Inversal piramidal em aminas 10 – 13 Rápida Isomeria conformacional (rotação livre em torno de ligações simples) 0 – 12 Rápida ou muito rápida RESOLVA AGORA 19 (FUVEST 2006) Considere os seguintes dados: a) Qual dos alcenos (A ou B) é o mais estável? Justifique. Neste caso, considere válido raciocinar com entalpia. A desidratação de álcoois, em presença de ácido, pode produzir uma mistura de alcenos, em que predomina o mais estável. b) A desidratação do álcool a seguir, em presença de ácido, produz cerca de 90% de um determinado alceno. Qual deve ser a fórmula estrutural desse alceno? Justifique. Energia de tensão nos cicloalcanos Ciclo-alcano [(CH2)n] n Tensão total (kcal/mol) Ciclo-propano 3 27,6 Ciclo-butano 4 26,3 Ciclo-pentano 5 6,5 Ciclo-hexano 6 0,0 Ciclo-heptano 7 6,4 Ciclo-octano 8 10,0 Ciclo-nonano 9 12,9 Ciclo-decano 10 12,0 Ciclo-pentadecano 15 1,5 n-alcano de cadeia aberta n 0,0 Fonte: Química orgânica – Allinger, Cava, De Johng OBSERVAÇÕES CH3OH 83 APOSTILA 01 DE FÍSICO-QUÍMICA – PROF. PEDRO MADEIRA (2022) Energia de conjugação / ressonância RESOLVA AGORA 20 As reações de hidrogenação do ciclohexeno, do 1,3- ciclohexadieno e do benzeno conduzem ao ciclohexano com libertação de entalpia DH. Comente os valores observados experimentalmente, seguindo o roteiro a seguir: (a) Justifique a liberação de calor a partir das energias de ligação. (b) Explicando porque razão não se observa aditividade das entalpias com o número de ligações duplas. RESOLVA AGORA 21 (ATKINS) R. F. Curl, R. E. Smalley e H. W. Kroto receberam o prêmio Nobel em química em 1996 pela descoberta da molécula C60 com a forma de bola de futebol. A entalpia de combustão do C60 é -25937 kJ.mol–1 e sua entalpia de sublimação é + 233 kJ.mol–1. Existem 90 ligações em C60, das quais 60 são simples e 30 são duplas. O C60 é como o benzeno, em que há um conjunto de ligações múltiplas para as quais as estruturas ressonantes podem ser desenhadas. (a) Determine a entalpia de formação do C60 a partir de sua entalpia de combustão. (b) Calcule a entalpia de formação esperada do C60 a partir das entalpias de ligação, assumindo que as ligações duplas e simples possam ser isoladas. (c) O C60 é mais ou menos estável que o previsto com base no modelo de ligações isoladas? (d) Quantifique a resposta em (c) dividindo a diferença entre a entalpia de formação calculada dos dados de combustão e aqueles obtidos dos cálculos de entalpia do ligação por 60 para obter o valor por carbono. (e) Como o número em (d) se compara à energia de estabilização ressonante por carbono no benzeno (a energia de estabilização ressonante do benzeno é aproximadamente 150 kJ.mol–1)? (f) Por que esses números poderiam ser diferentes? A entalpia de atomização do C(graf) é +717 kJ.mol–1. Dados (kJ/mol): DHf°(CO2) = – 394; DHat°(grafite) = 717 EL (C–C) = 348; EL (C=C) = 612 Prof. Pedro Madeira + H2 + 2 H2 + 3 H2 DH (kcal/mol) - 28,6 - 53,4 - 49,8 84 APOSTILA 01 DE FÍSICO-QUÍMICA – PROF. PEDRO MADEIRA (2022) Comparação entre alótropos DfHo kJ/mol DfGo kJ/mol Smo J/K/mol Cp,mo J/K.mol C(graf) 0 0 5,740 8,527 C(diam) +1,895 +2,900 2,377 6,113 S(a,romb) 0 0 31,80 22,64 S(b,mono) +0,33 +0,1 32,6 23,6 O2(g) 0 0 205,138 29,355 O3(g) +142,7 +163,2 238,93 39,20 P(verm) 22,8 P4(bran) 164 Observe as estruturas dos alótropos naturais do carbono: c/2 = 3,35 Å C–C = 1,42 Å Comparação entre isótopos do hidrogênio DfHo kJ/mol DfGo kJ/mol Smo J/K.mol Cp,mo J/K.mol H2(g) 0 0 130,684 28,824 D2(g) 0 0 144,96 29,20 HD(g) + 0,138 - 1,464 143,80 29,196 H2O(l) - 285,83 - 237,13 69,91 75,291 D2O(l) - 294,60 - 243,44 75,94 84,35 H2O(g) - 241,82 - 228,57 188,83 33,58 D2O(g) - 249,60 - 234,54 198,34 34,27 HDO(g) - 245,30 - 233,11 199,51 33,81 HDO(l) - 289,89 - 241,86 79,29 H2O2(l) - 187,78 - 120,35 109,6 89,1 H+(aq) 0 0 0 0 H+(g) 1536,20 H(g) + 217,97 + 203,25 114,71 20,784 Constante Física H2 D2 T2 Massa atômica (uma) 1,0078 2,0141 3,0160 Ponto de fusão (oC) - 259,0 - 254,3 - 252,4 Ponto de ebulição (oC) - 252,6 - 249,3 - 248,0 Comprimento de ligação (Å) 0,7414 0,7414 0,7414 Calor de dissociação (kJ/mol) 435,9 443,4 446,9 Calor de fusão (kJ/mol) 0,117 0,197 0,250 Calor de vaporização (kJ/mol) 0,904 1,226 1,393 Constante Física H2O D2O Ponto de fusão (oC) 0 3,82 Ponto de ebulição (oC) 100 100,42 Densidade a 20oC 0,917 1,017 Temperatura da densidade máxima (oC) 4 11,6 Kw a 25oC 1,0 x 10–14 3,0 x 10–15 Constante dielétrica a 20oC 82 80,5 Solubilidade do NaCl a 25oC (g/100g água) 35,9 30,5 85 APOSTILA 01 DE FÍSICO-QUÍMICA – PROF. PEDRO MADEIRA (2022) Família 1A Raio iônico (Å) Mobilidade iônica à diluição infinita Raio do íon hidratado (Å) Li+ 0,76 33,5 3,40 Na+ 1,02 43,5 2,76 K+ 1,38 64,5 2,32 Rb+ 1,52 67,5 2,28 Cs+ 1,67 68,0 2,28 No de hidratação aproximado Parâmetros termodinâmicos da hidratação DHo DSo DGo Li+ 25,3 - 544 - 134 - 506 Na+ 16,6 - 435 - 100 - 406 K+ 10,5 - 352 - 67 - 330 Rb+ 10,0 - 326 - 54 - 310 Cs+ 9,9 - 296 - 50 - 276 Energia de coesão (entalpia de atomização) (kJ/mol) Ponto de Fusão (oC) Ponto de Ebulição (oC) Li 161 181 1347 Na 108 98 881 K 90 63 766 Rb 82 39 688 Cs 78 28,5 705 PROF. PEDRO MADEIRA I)Dependência das entalpias com a temperatura Lei de Kirchoff OBSERVAÇÕES 86 APOSTILA 01 DE FÍSICO-QUÍMICA – PROF. PEDRO MADEIRA (2022) o o o r p p,m p,m produtos reagentes Δ C = -C Cu u× ×å å Prof. Pedro Madeira RESOLVA AGORA 22 (ITA 2002) A figura abaixo mostra como a capacidade calorífica, CP, de uma substância varia com a temperatura, sob pressão constante. Considerando as informações mostradas na figura acima, é ERRADO afirmar que A ( ) a substância em questão, no estado sólido, apresenta mais de uma estrutura cristalina diferente. B ( ) a capacidade calorífica da substância no estado gasoso é menor do que aquela no estado líquido. C ( ) quer esteja a substância no estado sólido, líquido ou gasoso, sua capacidade calorífica aumenta com o aumento da temperatura. D ( ) caso a substância se mantenha no estado líquido em temperaturas inferiores a Tf, a capacidade calorífica da substância líquida é maior do que a capacidade calorífica da substância na fase sólida estável em temperaturas menores do que Tf. E ( ) a variação de entalpia de uma reação envolvendo a substância em questão no estado líquido aumenta com o aumento da temperatura. RESOLVA AGORA 23 (IME 2015) Monóxido de carbono a 473 K é queimado, sob pressão atmosférica, com 90% em excesso de ar seco, em base molar, a 773 K. Os produtos da combustão abandonam a câmara de reação a 1273 K. Admita combustão completa e considere que 1 mol de ar é constituído por 0,20 mol de oxigênio e 0,80 mol de nitrogênio. Calcule a quantidade de energia, em kJ, que é liberada no decorrer da reação, por mol de monóxido de carbono queimado. Considere que os gases apresentam comportamento ideal. DADOS: Calor de combustão do monóxido de carbono (a 298 K e 1 atm) = –283 kJ·mol–1 T(K) = t(oC) + 273 Substância CO CO2 O2 N2 Cp médio (kJ/mol.K) 0,03 0,04 0,03 0,03( ) ( ) 2 1 Tθ θ θ r 2 r 1 r pT Δ H T = Δ H T + Δ C dTò 87 APOSTILA 01 DE FÍSICO-QUÍMICA – PROF. PEDRO MADEIRA (2022) 88 APOSTILA 01 DE FÍSICO-QUÍMICA – PROF. PEDRO MADEIRA (2022) J) Tabela de entalpias de formação e de energias de ligação Calores de formação (kJ/mol) a 25ºC e 1 atm AgBr(s) – 99,5 C2H2(g) + 226,7 H2O(l) – 285,8 NH4Cl(s) – 315,4 AgCl(s) – 127,0 C2H4(g) + 52,3 H2O2(l) – 187,6 NH4NO3(s) – 365,1 AgI(s) – 62,4 C2H6(g) – 84,7 H2S(g) – 20,2 NO(g) + 90,4 Ag2O(s) – 30,6 C3H8(g) – 103,8 H2SO4(l) – 811,3 NO2(g) + 33,9 Ag2S(s) – 31,8 n-C4H10(g) – 124,7 HgO(s) – 90,7 NiO(s) – 244,3 Al2O3(s) – 1669,8 n-C5H12(l) – 173,1 HgS(s) – 58,2 PbBr2(s) – 277,0 BaCl2(s) – 860,1 C2H5OH(l) – 277,6 KBr(s) – 392,2 PbCl2(s) – 359,2 BaCO3(s) – 1218,8 CaO(s) – 239,3 KCl(s) – 453,9 PbO(s) – 217,9 BaO(s) – 558,1 Cr2O3(s) – 1128,4 KClO4(s) – 391,4 PbO2(s) – 276,6 BaSO4(s) – 1465,2 CuO(s) – 155,2 KF(s) – 562,6 Pb3O4(s) – 734,7 CaCl2(s) – 795,0 Cu2O(s) – 166,7 MgCl2(s) – 641,8 PCl3(g) – 306,4 CaCO3(s) – 1207,0 CuS(s) – 48,5 MgCO3(s) – 111,3 PCl5(g) – 398,9 CaO(s) – 635,5 CuSO4(s) – 769,9 MgO(s) – 601,8 SiO2(s) – 859,4 Ca(OH)2(s) – 986,6 Fe2O3(s) – 822,2 Mg(OH)2(s) – 924,7 SnCl2(s) – 349,8 CaSO4(s) – 1432,7 Fe3O4(s) – 1120,9 MgSO4(s) – 1278,2 SnCl4(l) – 545,2 CCl4(l) – 139,5 HBr(g) – 36,2 MnO(s) – 384,9 SnO(s) – 286,2 CH4(g) – 74,8 HCl(g) – 92,3 MnO2(s) – 519,7 SnO2(s) – 580,7 CHCl3(l) – 131,8 HF(g) – 268,6 NaCl(s) – 411,0 SO2(g) – 296,1 CH3OH(l) – 238,6 HI(g) + 25,9 NaF(s) – 569,0 SO3(g) – 395,2 CO(g) – 110,5 HNO3(l) – 173,2 NaOH(s) – 426,7 ZnO(s) – 348,0 CO2(g) – 393,5 H2O(g) – 241,8 NH3(g) – 46,2 ZnS(s) – 202,9 (Princípios de Química, Masterton, Editora LTC) Energias de ligação (kJ/mol) Br–Br 193 C–H 414 Cl–N 201 H–S 339 Br–C 276 C–I 218 Cl–O 205 I–I 151 Br–Cl 218 C–N 293 Cl–S 255 I–O 201 Br–F 255 C=N 615 F–F 153 N–N 159 Br–H 368 C≡N 890 F–H 565 N=N 418 Br–I 180 C–O 351 F–I 277 N≡N 941 Br–N 243 C=O 715 F–N 272 N–O 222 Br–O 201 C≡O 1075 F–O 184 N=O 607 Br–S 213 C–S 259 F–S 258 O–O 138 C–C 347 C=S 477 H–H 436 O=O 498 C=C 612 Cl–Cl 243 H–I 297 O–S 347 C≡C 820 Cl–F 255 H–N 389 O=S 498 C–Cl 331 Cl–H 431 H–O 464 S–S 226 C–F 485 Cl–I 209 89 APOSTILA 01 DE FÍSICO-QUÍMICA – PROF. PEDRO MADEIRA (2022) CAPÍTULO 02 – TERMODINÂMICA QUÍMICA EXERCÍCIOS TÓPICO 01: CONCEITOS FUNDAMENTAIS SEÇÃO VESTIBULARES 1. (FMJ 2021) Liga eutética Sn-Pb Ligas metálicas podem ser produzidas a partir da fusão dos metais puros e posterior mistura dos metais derretidos. Uma liga metálica muito utilizada no cotidiano é formada pela mistura de chumbo com estanho, na proporção de 37% de chumbo e 63% de estanho. Os gráficos mostram a curva de aquecimento dos metais isolados e da liga formada pela mistura dos metais. Considere que, para aquecer os metais até sua fusão, utiliza-se como combustível o metano. a) Com base na análise das curvas de aquecimento, dê o nome da mistura que constitui a liga Sn-Pb. b) Considerando que o calor de fusão do estanho é igual a 7 kJ/mol, e que as entalpias-padrão de CH4, CO2 e H2O são, respectivamente, –76 kJ/mol, –394 kJ/mol e –286 kJ/mol, calcule a massa de estanho, em gramas, que pode ser derretida pelo calor gerado na queima de 1 mol de metano. 2. (FUVEST 2020) Equipamentos domésticos chamados de vaporizadores para roupa utilizam o vapor de água gerado por um sistema de resistências elétricas a partir de água líquida. Um equipamento com potência nominal de 1600 W foi utilizado para passar roupas por 20 minutos, consumindo 540 mL de água. Em relação ao gasto total de energia do equipamento, o gasto de energia utilizado apenas para vaporizar a água, após ela já ter atingido a temperatura de ebulição, equivale a, aproximadamente, Note e adote: Entalpia de vaporização da água a 100ºC = 40 kJ/mol; a) 0,04% b) 0,062% c) 4,6% d) 40% e) 62% 3. (UNICAMP 2020) “O sal faz a água ferver mais rápido?” Essa é uma pergunta frequente na internet, mas não tente responder com os argumentos lá apresentados. Seria muito difícil responder à pergunta tal como está formulada, pois isso exigiria o conhecimento de vários parâmetros termodinâmicos e cinéticos no aquecimento desses líquidos. Do ponto de vista termodinâmico, entre tais parâmetros, caberia analisar os valores de calor específico e de temperatura de ebulição da solução em comparação com a água pura. Considerando massas iguais (água pura e solução), se apenas esses parâmetros fossem levados em consideração, a solução ferveria mais rapidamente se o seu calor específico fosse a) menor que o da água pura, observando-se ainda que a temperatura de ebulição da solução é menor. b) maior que o da água pura, observando-se ainda que a temperatura de ebulição da solução é menor. c) menor que o da água pura, observando-se, no entanto, que a temperatura de ebulição da solução é maior. d) maior que o da água pura, observando-se, no entanto, que a temperatura de ebulição da solução é maior. 4. (UNICAMP 2018) Aluminotermia é exotérmica Em 12 de maio de 2017 o Metrô de São Paulo trocou 240 metros de trilhos de uma de suas linhas, numa operação feita de madrugada, em apenas três horas. Na solda entre o trilho novo e o usado empregou-se uma reação química denominada térmita, que permite a obtenção de uma temperatura local de cerca de 2000ºC. A reação utilizada foi entre um óxido de ferro e o alumínio metálico. De acordo com essas informações, uma possível equação termoquímica do processo utilizado seria a) Fe2O3 + 2 Aℓ → 2 Fe + Aℓ2O3; DH = +852 kJ.mol–1 b) FeO3 + Aℓ → Fe + AℓO3; DH = –852 kJ.mol–1 c) FeO3 + Aℓ → Fe + AℓO3; DH = +852 kJ.mol–1 d) Fe2O3 + 2 Aℓ → 2 Fe + Aℓ2O3; DH = –852 kJ.mol–1 90 APOSTILA 01 DE FÍSICO-QUÍMICA – PROF. PEDRO MADEIRA (2022) 5. (UEMA 2015) Um técnico de laboratório de química, para destilar certa massa de água, usou um aquecedor elétrico para colocar em ebulição 80% dessa massa, pois o mesmo não pode funcionar a seco. Considere que essa massa estava a 20ºC e que levou 5 min para ferver a 100ºC. Adotando-se um regime estacionário e sem perda de energia, o calor de vaporização igual a 540 cal / g e o calor específico igual a 1 cal/goC calcule o tempo total programado pelo técnico para o desligamento do temporizador do aquecedor, considerando que o mesmo não tenha sofrido qualquer danificação. 6. (UEM 2011) Sistemas domésticos de aquecimento de água estão cada vez mais presentes nos empreendimentos imobiliários. Esses sistemas são constituídos de uma unidade de aquecimento que utiliza a radiação solar como fonte de aquecimento e de um reservatório de água. Considere um sistema desse tipo com volume total de 500 litros, que seja capaz de aumentar a temperatura desse volume de água em 2 ºC a cada hora de exposição à luz solar. A temperatura inicial da água é de 23 ºC, e o sistema é exposto à luz solar das 8 às 18 horas. Desprezando a possível troca de calor do sistema com o meio ambiente, assinale o que for correto. 01) A temperatura da água às 18 horas é 43 ºC. 02) A quantidade de calor recebido pelos 500 litros de água até as 18 horas é 1x107 cal. 04) Se ao meio dia, metade do volume de água for retirado e imediatamente reposto com água a 23ºC, a temperatura de equilíbrio térmico é de aproximadamente 300 K. 08) A capacidade térmica dos 500 litros de água é 1x105 cal / oC. 16) Às onze horas, a temperatura da água é inferior a 35oF. 7. (UFSCAR 2008) Após ter estudado calorimetria, um aluno decide construir um calorímetro usando uma lata de refrigerante e isopor. Da latinhade alumínio removeu parte da tampa superior. Em seguida, recortou anéis de isopor, de forma que estes se encaixassem na latinha recortada, envolvendo-a perfeitamente (Figura 1). Em seu livro didático, encontrou as seguintes informações (Figura 2): a) Determine a capacidade térmica desse calorímetro, sabendo que a massa da latinha após o recorte realizado era de 15 . 10-3 kg. b) Como a capacidade térmica do calorímetro era muito pequena, decidiu ignorar esse valor e então realizou uma previsão experimental para o seguinte problema: Determinar a temperatura que deve ter atingido um parafuso de ferro de 0,1 kg aquecido na chama de um fogão. Dentro do calorímetro, despejou 0,2 L de água. Após alguns minutos, constatou que a temperatura da água era de 19 °C. Aqueceu então o parafuso, colocando-o em seguida no interior do calorímetro. Atingido o equilíbrio térmico, mediu a temperatura do interior do calorímetro, obtendo 40 °C. Nessas condições, supondo que houvesse troca de calor apenas entre a água e o parafuso, determine aproximadamente a temperatura que este deve ter atingido sob o calor da chama do fogão. 8. (PUCCAMP 1995) Um calorímetro de capacidade térmica 50 cal/oC contém 520 g de gelo a 0oC. Injeta-se no calorímetro vapor de água a 120ºC, na quantidade necessária e suficiente para fundir totalmente o gelo. A massa de água, em gramas, que se forma no interior do calorímetro vale a) 520 b) 584 c) 589 d) 620 e) 700 9. (UFC 2003) Dentre as diversas utilidades da água líquida, cita-se sua elevada capacidade de extinguir incêndios, que é decorrente, principalmente, do seu elevado calor de vaporização (DHvap= 9,72 kcal/mol, a 100oC). Assinale a alternativa correta. A) O calor de vaporização de uma substância resulta do rompimento das interações intermoleculares do estado gasoso. B) A evaporação de 18 g de água, a 100oC, libera 9.720 calorias, acarretando o resfriamento da vizinhança. C) A evaporação de 1g de água, a 100oC, pode causar o resfriamento da vizinhança, correspondente a 540 calorias. 91 APOSTILA 01 DE FÍSICO-QUÍMICA – PROF. PEDRO MADEIRA (2022) D) O elevado calor de vaporização da água resulta do fato de ela ocupar maior volume no estado líquido do que no estado sólido. E) A evaporação de um líquido, por se constituir em um fenômeno químico, envolve o rompimento de ligações químicas. 10. (FUVEST 2012) Observe o gráfico Um aluno efetuou um experimento para avaliar o calor envolvido na reação de um ácido com uma base. Para isso, tomou 8 tubos de ensaio e a cada um deles adicionou 50 mL de uma mesma solução aquosa de e diferentes volumes de água. Em seguida, acondicionou esses tubos em uma caixa de isopor, para minimizar trocas de calor com o ambiente. A cada um desses tubos, foram adaptados uma rolha e um termômetro para medir a temperatura máxima atingida pela respectiva solução, após o acréscimo rápido de volumes diferentes de uma mesma solução aquosa de NaOH. O volume final da mistura, em cada tubo, foi sempre 100 mL. Os resultados do experimento são apresentados na tabela. Tubo Volume de HCl (aq) (mL) Volume de H2O (mL) Volume de NaOH (aq) (mL) Temperatura máxima (ºC) 1 50 50 0 23,0 2 50 45 5 24,4 3 50 40 10 25,8 4 50 35 15 27,2 5 50 30 20 28,6 6 50 25 25 30,0 7 50 20 30 30,0 8 50 15 35 30,0 a) Construa um gráfico, no quadriculado apresentado na página de resposta, que mostre como a temperatura máxima varia em função do volume de solução aquosa de NaOH acrescentado. b) A reação do ácido com a base libera ou absorve calor? Justifique sua resposta, considerando os dados da tabela. c) Calcule a concentração, em mol L–1, da solução aquosa de HCl, sabendo que a concentração da solução aquosa de NaOH utilizada era 2,0 mol L–1. 11. (FUVEST 2013) Fotólise Em uma reação de síntese, induzida por luz vermelha de frequência f igual a 4,3X1014 Hz, ocorreu a formação de 180 g de glicose. Determine a) o número N de mols de glicose produzido na reação; b) a energia E de um fóton de luz vermelha; c) o número mínimo n de fótons de luz vermelha necessário para a produção de 180 g de glicose; d) o volume V de oxigênio produzido na reação (CNTP). Note e adote: Constante de Planck: h = 6,6x10 – 34 J.s. Nessa reação são necessários 2800 kJ de energia para a formação de um mol de glicose. Prof. Pedro Madeira 12. (FUVEST 2008) Método de Job Em um exame, para o preenchimento de uma vaga de químico, as seguintes fórmulas estruturais foram apresentadas ao candidato: A seguir, o examinador pediu ao candidato que determinasse, experimentalmente, o calor liberado ao fazer-se a mistura de volumes definidos de duas soluções aquosas, de mesma concentração, uma de hidróxido de sódio e outra de um dos três ácidos carboxílicos apresentados, sem revelar qual deles havia sido escolhido. Foi informado ao candidato que, quando o ácido e a base reagem na proporção estequiométrica, o calor liberado é máximo. Os resultados obtidos foram os seguintes: Volume da solução de base/mL 0 15 30 35 40 45 50 Volume da solução de ácido/mL 50 35 20 15 10 5 0 Calor liberado /J 0 700 1400 1500 1000 500 0 Diante dos resultados obtidos, o examinador pediu ao candidato que determinasse qual dos ácidos havia sido utilizado no experimento. Para responder, o candidato construiu uma tabela e um gráfico do calor liberado versus Xbase, definido como: , equivalente a onde n = quantidade de ácido ou de base (em mol); V = volume da solução de ácido ou de base (em mL) a) Reproduza, na página ao lado, a tabela e o gráfico que devem ter sido obtidos pelo candidato. Pelos pontos do gráfico, podem ser traçadas duas retas, cujo cruzamento corresponde ao máximo calor liberado. b) Determine o valor de xbase que corresponde ao ponto de cruzamento das retas em seu gráfico. c) Qual foi o ácido escolhido pelo examinador? Explique. d) Indique qual é o reagente limitante para o experimento em que o calor liberado foi 1400 J e para aquele em que o calor liberado foi 1500 J. Explique. HC! 2 2 6 12 6 26H O 6CO energia C H O 6O ;+ + ® + ácidobase base base VV V X + = ácidobase base base nn n X + = 92 APOSTILA 01 DE FÍSICO-QUÍMICA – PROF. PEDRO MADEIRA (2022) SEÇÃO ITA / IME 13. (ITA 2011 – Q25) Em um frasco de vidro, uma certa quantidade de Ba(OH)2.8H2O(s) é adicionada uma quantidade, em excesso, de NH4NO3(s), ambos pulverizados. Quando os dois reagentes são misturados, observa-se a ocorrência de uma reação química. Imediatamente após a reação, o frasco é colocado sobre um bloco de madeira umedecido, permanecendo aderido a ele por um certo período de tempo. Escreva a equação química balanceada que representa a reação observada. Explique por que o frasco ficou aderido ao bloco de madeira, sabendo que o processo de dissolução em água do NH4NO3(s) é endotérmico. 14. (IME 2014 – Q37) Uma mistura “A”, cuja composição percentual volumétrica é de 95% de água e 5% de álcool etílico, está contida no bécher 1. Uma mistura “B”, cuja composição percentual volumétrica é de 95% de água e 5% de gasolina, está contida no bécher 2. Essas misturas são postas em repouso a 25oC e 1 atm, tempo suficiente para se estabelecer, em cada bécher, a situação de equilíbrio. Em seguida, aproximam-se chamas sobre as superfícies de ambas as misturas. O que ocorrerá? (A) Nada, ou seja, não ocorrerá combustão em nenhuma das superfícies devido à grande similaridade de polaridade e densidade entre os líquidos. (B) Nada, ou seja, não ocorrerá combustão em nenhuma das superfícies devido à grande diferença de polaridade e densidade entre os líquidos. (C) Ambas as superfícies entrarão em combustão, simultaneamente, devido à elevada diferença de polaridade e densidade entre os três líquidos. (D) Ocorrerá combustão somente sobre a superfícielíquida no bécher 1, devido à diferença de polaridade e densidade entre os líquidos. (E) Ocorrerá combustão somente sobre a superfície líquida no bécher 2, devido à diferença de polaridade e densidade entre os líquidos. 15. (ITA 2014 – Q12) Considere três cubos maciços de 2 cm de aresta, constituídos, respectivamente, de Cr, Ni e Ti puros. Os três cubos são aquecidos até 80ºC e cada cubo é introduzido em um béquer contendo 50 g de água a 10ºC. Com base nas informações constantes da tabela abaixo, assinale a opção que apresenta a relação CORRETA entre as temperaturas dos cubos, quando o conteúdo de cada béquer atingir o equilíbrio térmico. Substância Massa específica (g.cm–3) Calor específico (J.g–1.K–1) H2O 1,00 4,18 Ti 4,54 0,52 Cr 7,18 0,45 Ni 8,90 0,44 A ( ) TCr > TNi > TTi B ( ) TNi = TTi > TCr C ( ) TNi > TCr > TTi D ( ) TTi > TCr > TNi E ( ) TTi > TCr = TNi 16. (ITA 2014 – Q15) São feitas as seguintes afirmações sobre o que Joule demonstrou em seus experimentos do século XIX: I. A relação entre calor e trabalho é fixa. II. Existe um equivalente mecânico do calor. III. O calor pode ser medido. Das afirmações acima, está(ão) CORRETA(S) apenas A ( ) I. B ( ) I, II e III. C ( ) I e III. D ( ) II. E ( ) II e III. 17. (ITA 2018 – Q06) Deseja-se aquecer 586g de água pura da temperatura ambiente até 91ºC, em pressão ambiente. Utilizando um forno de micro-ondas convencional que emite radiação eletromagnética com frequência de 2,45 GHz e considerando a capacidade calorífica da água constante e igual a 4,18 Jg–1oC–1 , assinale a alternativa que apresenta o número aproximado de fótons necessário para realizar este aquecimento. A ( ) 3 x 1027 B ( ) 4 x 1028 C ( ) 1 x 1029 D ( ) 5 x 1030 E ( ) 2 x 1031 93 APOSTILA 01 DE FÍSICO-QUÍMICA – PROF. PEDRO MADEIRA (2022) 18. (ITA 2022 – Q67) – RESOLVA AGORA Considere as seguintes afirmações sobre processos termodinâmicos, que podem ocorrer em uma ou mais etapas, em que DT se refere à variação de temperatura entre os estados inicial e final: I. Um processo termodinâmico é definido pelo estado final e estado inicial do sistema. II. DT é sempre nula em um processo isotérmico. III. A troca de calor envolvida em um processo isotérmico deve ser nula (q=0). IV. Todo processo em que DT = 0 é um processo isotérmico. V. DT = 0 para todo processo em sistema isolado. Assinale a opção que contém as afirmações ERRADAS: A ( ) Apenas I, II e IV B ( ) Apenas I, III, IV e V C ( ) Apenas I, III e V D ( ) Apenas II e IV E ( ) III e V TÓPICO 02: PRIMEIRA LEI DA TERMODINÂMICA SEÇÃO VESTIBULARES 19. (UFG 2014) Em um recipiente com paredes perfeitamente condutoras de calor encontra-se uma solução altamente concentrada de ácido clorídrico à temperatura de 27oC e à pressão atmosférica. Certa quantidade de pó de magnésio é colocada na solução e, imediatamente depois, o recipiente é tampado com um pistão de massa desprezível, que fica em contato com a superfície do líquido e que pode deslizar sem atrito ao longo do recipiente. Quando a situação de equilíbrio é alcançada observa-se que o magnésio reagiu completamente com o ácido e que o pistão levantou-se em relação à superfície da solução devido à produção de gás. Sabendo que no processo todo o sistema realizou um trabalho de 240 J, e considerando o gás produzido como ideal, conclui-se que a massa, em gramas, de magnésio inicialmente colocada na solução foi: a) 0,243 b) 0,486 c) 0,729 d) 1,215 e) 2,430 20. (UEM-PAS 2016) A formação de CO2(g), segundo a reação O2(g) + C(graf) à CO2(g), possui uma variação de entalpia ΔH = –395 kJ. Parte dessa energia foi utilizada para realizar o processo mostrado no diagrama pV no qual se têm 4 mols de um gás monoatômico ideal. Considerando as informações apresentadas, assinale o que for correto sobre esse diagrama pV. 01) A temperatura inicial desse gás monoatômico ideal é de aproximadamente 500K. 02) A temperatura final do gás monoatômico ideal é de aproximadamente 900K. 04) A variação da energia interna do gás monoatômico ideal foi de aproximadamente 29,9 kJ. 08) O trabalho realizado pelo sistema descrito no diagrama pV foi de 15kJ. 16) O calor utilizado pelo sistema descrito no diagrama pV foi de aproximadamente 50% daquele gerado pela reação química do CO2(g). 21. (UPE 2011) Um recipiente cilíndrico, de área de secção reta de 0,100 m2 contém 20,0 g de gás hélio. Esse recipiente contém um êmbolo que pode se mover sem atrito. Uma fonte fornece calor ao recipiente a uma taxa constante. Num determinado instante, o gás sofre a transformação termodinâmica representada no diagrama PV abaixo, e o êmbolo se move com velocidade constante v=8,31x10–3 m/s Considere que o gás hélio se comporta como um gás monoatômico ideal. Depois de decorrido um intervalo de tempo de 25 s, analise as proposições a seguir e conclua. ( ) A variação de temperatura do gás durante o processo foi ΔT = 50K. ( ) O calor específico molar à pressão constante do hélio é Cp = 2,5R. ( ) A energia adicionada ao hélio sob a forma de calor durante o processo foi Q = 375R. ( ) A variação na energia interna do hélio durante o processo foi ΔE = 125R. ( ) O trabalho realizado pelo hélio durante a transformação foi W = 250R. 94 APOSTILA 01 DE FÍSICO-QUÍMICA – PROF. PEDRO MADEIRA (2022) 22. (UPE 2010) No diagrama PV, a seguir, está representada uma série de processos termodinâmicos. No processo ab, 250 J de calor são fornecidos ao sistema, e, no processo bd, 600 J de calor são fornecidos ao sistema. Analise as afirmações que se seguem. I. O trabalho realizado no processo ab é nulo. II. A variação de energia interna no processo ab é 320 J. III. A variação de energia interna no processo abd é 610 J. IV. A variação de energia interna no processo acd é 560 J. É CORRETO afirmar que apenas as(a) afirmações(ão) a) II e IV estão corretas. b) IV está correta. c) I e III estão corretas. d) III e IV estão corretas. e) II e III estão corretas. Prof. Pedro Madeira 23. (UPE 2010) O diagrama PV para uma determinada amostra de gás está representado na figura a seguir. Se o sistema é levado do estado a para o estado b, ao longo do percurso acb, fornece-se a ele uma quantidade de calor igual a 100 cal, e ele realiza um trabalho de 40 cal. Se, por meio do percurso adb, o calor fornecido é de 72 cal, então o trabalho realizado vale em cal: a) 28 b) 60 c) 12 d) 40 e) 24 24. (UFC 1998) Complete a afirmativa abaixo: A energia fornecida pelos alimentos é determinada em condições de temperatura e pressão constantes, constituindo-se numa medida de ______________. Assinale a opção escolhida para completar a afirmativa. A) Entalpia. B) Capacidade térmica. C) Calor específico. D) Calor latente. E) Energia interna. SEÇÃO ITA / IME 25. (ITA 1992 – Q09) Dentre as opções seguintes assinale aquela que contém, afirmação FALSA relativa ao comportamento de gases. A ( ) Para uma mesma temperatura e pressão iniciais, o calor específico sob volume constante é maior do que sob pressão constante. B ( ) A energia cinética média das moléculas é diretamente proporcional à temperatura absoluta e independe da pressão. C ( ) Na mesma pressão e temperatura, ar úmido é menos denso que ar seco. D ( ) No equilíbrio, a concentração de um gás dissolvido num liquido é diretamente proporcional à pressão parcial do referido gás na fase gasosa sobre o líquido. E ( ) Na expressão yq = y0(1+ aq), relativa à dilatação isobárica de um gás, onde q é a temperatura em graus Celsius, foi notado que a = (1 / 273°C) independentemente da natureza química do gás. 26. (ITA 1992 – Q13) Numa garrafa térmica, de capacidade caloríficadesprezível, são misturados um volume V1 de uma solução aquosa 2 molar de ácido clorídrico com um volume V2 de uma solução aquosa 1 molar de hidróxido de sódio. Antes da mistura, as duas soluções estavam na mesma temperatura. Em qual das misturas a seguir haverá maior aumento de temperatura? A ( ) V1 = 0,10 !; V2 = 0,20 ! B ( ) V1 = 0,20 !; V2 = 0,20 ! C ( ) V1 = 0,40 !; V2 = 0,40 ! D ( ) V1 = 0,20 !; V2 = 0,10 ! E ( ) V1 = 0,40 !; V2 = 0,20 ! PERGUNTA – Comparando as alternativas B e D, em qual dessas duas haverá maior aumento de ternperatura? Justifique a sua resposta. 27. (ITA 1995 – Q07) Em um calorímetro adiabático, com capacidade térmica desprezível, são introduzidos, sob pressão constante de 1 atm, um volume V1 de solução aquosa 1,0 molar de ácido clorídrico e um volume V2 de solução aquosa 1,0 molar de hidróxido de sódio. A reação que ocorre é aquela representada pela equação química: H+(aq) + OH–(aq) ® H2O(l) As misturas efetuadas são as seguintes: 95 APOSTILA 01 DE FÍSICO-QUÍMICA – PROF. PEDRO MADEIRA (2022) I. V1 = 100 ml e V2 = 100 ml e observa-se um aumento de temperatura DT1. II. V1 = 50 ml e V2 = 150 ml e observa-se um aumento de temperatura DT2. III. V1 = 50 ml e V2 = 50 ml e observa-se um aumento de temperatura DT3. Com relação ao efeito térmico que se observa, é correto prever que: A ( ) DT1 @ DT3 > DT2 B ( ) DT1 > DT2 > DT3 C ( ) DT1 > DT2 @ DT3 D ( ) DT1 > DT3 > DT2 E ( ) DT1 @ DT3 @ DT2 28. (ITA 2003 – Q13) Considere as seguintes comparações de calores específicos dos respectivos pares das substâncias indicadas. I. tetracloreto de carbono (ℓ, 25oC) > metanol(ℓ, 25oC) II. água pura (ℓ, -5oC) > água pura (s, -5oC) III. alumina (s, 25oC) > alumínio (s, 25oC) IV. isopor (s, 25oC) > vidro de janela (s, 25oC) Das comparações feitas, está(ao) CORRETA(S) A ( ) apenas I e II B ( ) apenas I, II e III C ( ) apenas II D ( ) apenas III e IV E ( ) apenas IV 29. (ITA 2004 – Q05) Considere as reações representadas pelas seguintes equações químicas balanceadas: a) C2H5OH(ℓ) + O2(g) → 2 C(s) + 3 H2O(g); ΔHI(T); ΔEI(T), b) C2H5OH(ℓ) + 2 O2(g) → 2 CO(g) + 3 H2O(l); ΔHII(T); ΔEII(T), sendo ΔH(T) e ΔE(T), respectivamente, a variação da entalpia e da energia interna do sistema na temperatura T. Assuma que as reações acima são realizadas sob pressão constante, na temperatura T, e que a temperatura dos reagentes é igual à dos produtos. Considere que, para as reações representadas pelas equações acima, sejam feitas as seguintes comparações: I. |ΔEI| = |ΔEII|. II. |ΔHI| = |ΔHII|. III. |ΔHII| > |ΔEII|. IV. |ΔHI| < |ΔEI|. Das comparações acima, está(ão) CORRETA(S) A ( ) apenas I B ( ) apenas I e II C ( ) apenas II D ( ) apenas III E ( ) apenas IV 30. (ITA 2005 – Q06) Um cilindro provido de um pistão móvel, que se desloca sem atrito, contém 3,2 g de gás hélio que ocupa um volume de 19,0 L sob pressão 1,2 x 105 N.m−2. Mantendo a pressão constante, a temperatura do gás é diminuída de 15 K e o volume ocupado pelo gás diminui para 18,2 L. Sabendo que a capacidade calorífica molar do gás hélio à pressão constante é igual a 20,8 J K−1 mol−1, a variação da energia interna neste sistema é aproximadamente igual a A ( ) - 0,35 k J B ( ) - 0,25 k J C ( ) - 0,20 k J D ( ) - 0,15 k J E ( ) - 0,10 k J 31. (ITA 2006 – Q19) Uma reação química hipotética é representada pela seguinte equação: X(g) + Y(g)→ 3Z(g). Considere que esta reação seja realizada em um cilindro provido de um pistão, de massa desprezível, que se desloca sem atrito, mantendo-se constantes a pressão em 1 atm e a temperatura em 25oC. Em relação a este sistema, são feitas as seguintes afirmações: I. O calor trocado na reação é igual à variação de entalpia. II. O trabalho realizado pelo sistema é igual a zero. III. A variação da energia interna é menor do que a variação da entalpia. IV. A variação da energia interna é igual a zero. V. A variação da energia livre de Gibbs é igual à variação de entalpia. Então, das afirmações acima, estão CORRETAS A ( ) apenas I, II e IV B ( ) apenas I e III C ( ) apenas II e V D ( ) apenas III e IV E ( ) apenas III, IV e V PROF. PEDRO MADEIRA 32. (ITA 2007 – Q01) Amostras de massas iguais de duas substâncias, I e II, foram submetidas independentemente a um processo de aquecimento em atmosfera inerte e a pressão constante. O gráfico abaixo mostra a variação da temperatura em função do calor trocado entre cada uma das amostras e a vizinhança. Dados: ΔHf e ΔHv representam as variações de entalpia de fusão e de vaporização, respectivamente, e cp é o calor específico. Assinale a opção ERRADA em relação à comparação das grandezas termodinâmicas. A ( ) ΔHf (I) < ΔHf (II) B ( ) ΔHv (I) < ΔHv (II) C ( ) cp,I (s) < cp,II (s) D ( ) cp,II (g) < cp,I (g) E ( ) cp,II (ℓ) < cp,I (ℓ) Prof. Pedro Madeira 33. (ITA 2007 – Q02) Um recipiente aberto contendo inicialmente 30 g de um líquido puro a 278 K, mantido à pressão constante de 1 atm, é colocado sobre uma balança. A seguir, é imersa no líquido uma resistência elétrica de 3 Ω conectada, por meio de uma chave S, a uma fonte que fornece uma 96 APOSTILA 01 DE FÍSICO-QUÍMICA – PROF. PEDRO MADEIRA (2022) corrente elétrica constante de 2 A. No instante em que a chave S é fechada, dispara-se um cronômetro. Após 100 s, a temperatura do líquido mantém-se constante a 330 K e verifica-se que a massa do líquido começa a diminuir a uma velocidade constante de 0,015 g/s. Considere a massa molar do líquido igual a M. Assinale a opção que apresenta a variação de entalpia de vaporização (em J/mol) do líquido. A ( ) 500 M B ( ) 600 M C ( ) 700 M D ( ) 800 M E ( ) 900 M 34. (ITA 2007 – Q03) Utilizando o enunciado da questão anterior, assinale a opção que apresenta o valor do trabalho em módulo (em kJ) realizado no processo de vaporização após 180 s de aquecimento na temperatura de 330 K. A ( ) 4,4 / M B ( ) 5,4 / M C ( ) 6,4 / M D ( ) 7,4 / M E ( ) 8,4 / M 35. (ITA 2008 – Q13) 300 gramas de gelo a 0oC foram adicionados a 400 gramas de água a 55oC. Assinale a opção CORRETA para a temperatura final do sistema em condição adiabática. Dados: calor de fusão do gelo = 80 cal.g–1; calor específico do gelo = 0,50 cal.g–1.K–1; calor específico da água líquida = 1 cal.g–1.K–1. A ( ) – 4oC B ( ) – 3oC C ( ) 0oC D ( ) +3oC E ( ) +4oC 36. (ITA 2008 – Q18) Considere que os quatro processos químicos, descritos a seguir nos itens I a IV, são realizados isobárica e isotermicamente: I. KNO3(s) à K+(aq) + NO3 –(aq) II. H2O(l) à H2O(g) III. C(grafita) à C(diamante) IV. 2 Na(s) + ½ O2(g) → Na2O(s) Qual das opções abaixo contém os processos químicos cuja variação de energia interna é nula? A ( ) Apenas I e II B ( ) Apenas I, II e III C ( ) Apenas II e III D ( ) Apenas III e IV E ( ) Nenhum processo 37. (ITA 2008 – Q26) Dois cilindros (I e II) são providos de pistões, cujas massas são desprezíveis e se deslocam sem atrito. Um mol de um gás ideal é confinado em cada um dos cilindros I e II. São realizados, posteriormente, dois tipos de expansão, descritos a seguir: a) No cilindro I, é realizada uma expansão isotérmica à temperatura T, de um volume V até um volume 2V, contra uma pressão externa constante P. b) No cilindro II, é realizada uma expansão adiabática, de um volume V até um volume 2V, contra uma pressão externa constante P. Determine os módulos das seguintes grandezas: variação da energia interna, calor trocado e trabalho realizado para os dois tipos de expansão. 38. (ITA 2009 – Q08) O diagrama temperatura (T) versus volume (V) representa hipoteticamente as transformações pelas quais um gás ideal no estado 1 pode atingir o estado 3. Sendo DU a variação de energia interna e q a quantidade de calor trocadocom a vizinhança, assinale a opção com a afirmação ERRADA em relação às transformações termodinâmicas representadas no diagrama. A ( ) |DU12| = |q12| B ( ) |DU13| = |DU23| C ( ) |DU23| = |q23| D ( ) |DU23| > |DU12| E ( ) q23 > 0 39. (ITA 2009 – Q18) Nos gráficos abaixo, cada eixo representa uma propriedade termodinâmica de um gás que se comporta idealmente. Com relação a estes gráficos, é CORRETO afirmar que A ( ) I pode representar a curva de pressão versus volume. B ( ) II pode representar a curva de pressão versus inverso do volume. C ( ) II pode representar a curva de capacidade calorífica versus temperatura. D ( ) III pode representar a curva de energia interna versus temperatura. E ( ) III pode representar a curva de entalpia versus o produto da pressão pelo volume. 40. (ITA 2010 – Q01) A figura ao lado apresenta a curva de aquecimento de 100 g de uma substância pura genérica no estado sólido. Sabe-se que calor é fornecido a uma velocidade constante de 500 cal min–1. Admite-se que não há perda de calor para o meio ambiente, que a pressão é de 1 atm durante toda a transformação e que a substância sólida apresenta apenas uma fase cristalina. Considere que sejam feitas as seguintes afirmações em relação aos estágios de aquecimento descritos na figura: I. No segmento PQ ocorre aumento da energia cinética das moléculas. 97 APOSTILA 01 DE FÍSICO-QUÍMICA – PROF. PEDRO MADEIRA (2022) II. No segmento QR ocorre aumento da energia potencial. III. O segmento QR é menor que o segmento ST porque o calor de fusão da substância é menor que o seu calor de vaporização. IV. O segmento RS tem inclinação menor que o segmento PQ porque o calor específico do sólido é maior que o calor específico do líquido. Das afirmações acima, está(ão) ERRADA(S): A ( ) apenas I. B ( ) apenas I, II e III. C ( ) apenas II e IV. D ( ) apenas III. E ( ) apenas IV. 41. (ITA 2010 – Q19) Considere duas reações químicas, mantidas à temperatura e pressão ambientes, descritas pelas equações abaixo: I. H2(g) + 1/2 O2(g) à H2O(g) ΙΙ. H2(g) + 1/2 O2(g) à H2O(ℓ) Assinale a opção que apresenta a afirmação ERRADA sobre estas reações. A ( ) As reações I e II são exotérmicas. B ( ) Na reação I, o valor, em módulo, da variação de entalpia é menor que o da variação de energia interna. C ( ) O valor, em módulo, da variação de energia interna da reação I é menor que o da reação II. D ( ) O valor, em módulo, da variação de entalpia da reação I é menor que o da reação II. E ( ) A capacidade calorífica do produto da reação I é menor que a do produto da reação II. 42. (ITA 2011 – Q10) São descritos abaixo dois experimentos, I e II, nos quais há sublimação completa de uma mesma quantidade de dióxido de carbono no estado sólido a 25ºC: I – O processo é realizado em um recipiente hermeticamente fechado, de paredes rígidas e indeformáveis. II – O processo é realizado em cilindro provido de um pistão, cuja massa é desprezível e se desloca sem atrito. A respeito da variação da energia interna do sistema (DU), calor (q) e trabalho (w), nos experimentos I e II, assinale a opção que contém a afirmação ERRADA. A ( ) qI > 0 B ( ) |wII| > |wI| C ( ) DUI > DUII D ( ) |wII| ≠ 0 E ( ) DUII = qII Prof. Pedro Madeira 43. (ITA 2011 – Q14) Um sistema em equilíbrio é composto por no mol de um gás ideal a pressão Po, volume Vo, temperatura To e energia interna Uo. Partindo sempre deste sistema em equilíbrio, são realizados isoladamente os seguintes processos: I. Processo isobárico de To até To / 2. II. Processo isobárico de Vo até 2Vo. III. Processo isocórico de Po até Po / 2. IV. Processo isocórico de To até 2To. V. Processo isotérmico de Po até Po / 2. VI. Processo isotérmico de Vo até Vo / 2. Admitindo que uma nova condição de equilíbrio para esse sistema seja atingida em cada processo x (x = I, II, III, IV, V e VI), assinale a opção que contém a informação ERRADA. A ( ) UV = UVI / 2 B ( ) UVI = Uo C ( ) PIV = PVI D ( ) TII = 4 TIII E ( ) VI = VV / 4 44. (ITA 2012 – Q21) A tabela mostra a variação de entalpia de formação nas condições-padrão a 25ºC de algumas substâncias. Calcule a variação da energia interna de formação, em kJ.mol–1, nas condições-padrão dos compostos tabelados. Mostre os cálculos realizados. Substância DHo f (kJ.mol–1) AgCl(s) –127 CaCO3(s) –1207 H2O(l) –286 H2S(g) –20 NO2(g) +34 45. (ITA 2013 – Q13) Assinale a opção que representa a afirmação CORRETA. A ( ) Um paciente com calor de 42ºC apresenta-se febril. B ( ) A adição de energia térmica à água líquida em ebulição sob pressão ambiente causa um aumento na sua capacidade calorífica. C ( ) Na temperatura de –4oC e pressão ambiente, 5g de água no estado líquido contêm uma quantidade de energia maior do que a de 5g de água no estado sólido. D ( ) A quantidade de energia necessária para aquecer 5g de água de 20ºC até 25ºC é igual àquela necessária para aquecer 25g de água no mesmo intervalo de temperatura e pressão ambiente. E ( ) Sob pressão ambiente, a quantidade de energia necessária para aquecer massas iguais de alumínio (calor específico 0,89 J.g–1.K–1) e de ferro (calor específico 0,45 J.g–1.K–1), respectivamente, de um mesmo incremento de temperatura, DT, é aproximadamente igual. 46. (ITA 2014 – Q29) Considere os seguintes dados: Entalpia de vaporização da água a 25 °C: DvapH = 44 kJ.mol–1 Massa específica da água a 25 °C: rH2O = 1,0 g.cm–3 Temperaturas de ebulição a 1 bar: Teb(H2O)=100ºC; Teb(H2S)= – 60ºC e Teb(H2Se)= –2ºC; Com base nestas informações: (a) determine o valor numérico da energia liberada, em J, durante a precipitação pluviométrica de 20 mm de chuva sobre uma área de (10 x 10) km2. (b) justifique, em termos moleculares, por que H2O apresenta Teb muito maior que outros calcogenetos de hidrogênio. (c) como se relaciona, em termos moleculares, a elevada Teb(H2O) com a quantidade de energia liberada durante uma precipitação pluviométrica? 98 APOSTILA 01 DE FÍSICO-QUÍMICA – PROF. PEDRO MADEIRA (2022) 47. (ITA 2015 – Q17) São feitas as seguintes comparações sobre as capacidades caloríficas de diferentes substâncias puras, todas à temperatura ambiente: I. A capacidade calorífica da água é menor que a do peróxido de hidrogênio. II. A capacidade calorífica do bromo é menor que a do tetracloreto de carbono. III. A capacidade calorífica do metanol é menor que a do mercúrio. Assinale a opção que apresenta a(s) comparação(ões) CORRETA(S). A ( ) Apenas I B ( ) Apenas I e II C ( ) Apenas II D ( ) Apenas II e III E ( ) Apenas III 48. (ITA 2016 – Q16) Considere a expansão de um gás ideal inicialmente contido em um recipiente de 1 L sob pressão de 10 atm. O processo de expansão pode ser realizado de duas maneiras diferentes, ambas à temperatura constante: I. Expansão em uma etapa, contra a pressão externa constante de 1 atm, levando o volume final do recipiente a 10 L. II. Expansão em duas etapas: na primeira, o gás expande contra a pressão externa constante de 5 atm até atingir um volume de 2L; na segunda etapa, o gás expande contra uma pressão constante de 1 atm atingido o volume final de 10 L. Com base nestas informações, assinale a proposição CORRETA. A ( ) O trabalho realizado pelo gás é igual nos dois processos de expansão. B ( ) O trabalho realizado no primeiro processo é metade do trabalho realizado no segundo processo. C ( ) A variação da energia interna do gás é igual em ambos os processos. D ( ) A variação da energia interna do gás no primeiro processo é metade da do segundo processo. E ( ) O calor trocado pelo gás é igual em ambos os processos. 49. (ITA 2016 – Q28) A toda reação química corresponde uma variação de energia interna, DU, e uma variação de entalpia,DH. Explique em que condições DU tem valor igual ao de DH. 50. (ITA 2017 – Q15) Um motor pulso-jato é uma máquina térmica que pode ser representada por um ciclo termodinâmico ideal de três etapas: I. Aquecimento isocórico (combustão). II. Expansão adiabática (liberação de gases). III. Compressão isobárica (rejeição de calor a pressão atmosférica). Considerando que essa máquina térmica opere com gases ideais, indique qual dos diagramas pressão versus volume a seguir representa o seu ciclo termodinâmico. A ( ) B ( ) C ( ) D ( ) 99 APOSTILA 01 DE FÍSICO-QUÍMICA – PROF. PEDRO MADEIRA (2022) E ( ) 51. (ITA 2017 – Q29) Sobre um motor pulso jato como o apresentado na Questão anterior, considere verdadeiras as seguintes afirmações: I. A temperatura de fusão do material que compõe a câmara de combustão é 1500 K, e acima de 1200 K o material do motor começa a sofrer desgaste considerável pelos gases de combustão; II. O material do motor resiste a pressões de até 30 atm; III. O motor opera, em cada ciclo termodinâmico, com 0,2 mol de uma mistura de gases com comportamento ideal, iniciando o ciclo em pressão atmosférica e a temperatura de 300 K. a) A partir destas informações e considerando que se deseja obter, de forma segura, o máximo de trabalho por ciclo, quais devem ser a pressão e a temperatura no ponto de intersecção entre os processos I e II do ciclo termodinâmico (vide Questão anterior)? b) Na mistura de gases que opera em cada ciclo há uma fração de combustível, o qual tem a reação de combustão dada por: CH4(g) + 2 O2(g) à CO2(g) + 2 H2O(g) QV = 45 kJ/g em que QV é o calor liberado a volume constante, por grama de metano. Considerando a capacidade calorífica molar a volume constante da mistura de gases igual a 25 J·K–1·mol–1, qual é a massa de metano utilizada pelo ciclo projetado no item anterior? 52. (ITA 2021 – Q08) Considere a reação de oxirredução não balanceada de um mol de sulfato de chumbo com ácido hipocloroso, a 25ºC. 1PbSO4(s) + HOCl(aq) → Cl2(g) + PbO2(s) + HSO4 –(aq) + H+(aq) Para esta reação, a variação de entalpia padrão é ∆X<==+19,9 kJ. Sabe-se que o potencial de eletrodo padrão da espécie que sofre oxidação é +1,63 V e o da espécie que sofre redução é +1,61 V. a) Escreva as semirreações, a reação global balanceada e o potencial padrão da reação global. b) Determine a variação de energia interna da reação (∆Y<=), considerando comportamento ideal das espécies. c) Justifique termodinamicamente a diferença entre os valores de ∆X<= e ∆Y<= para a reação acima. 53. (IME 1991 – Q09) A combustão completa de 0,436g de uma mistura de carvão, na forma alotrópica de grafite rômbico, realizada em atmosfera de oxigênio, elevou a temperatura de 2,00 ℓ d’água do calorímetro de 24,67oC para 25,40oC. Desprezando as perdas de calor para as partes metálicas do calorímetro, determine a percentagem de enxofre na mistura, sabendo que: a) a massa específica da água é 1,00 g/cm3; b) o calor específico da água é 1,00 cal/g oC; c) os calores padrões de formação do CO2 e do SO2 a 25oC são: (∆Hfo)CO2 = – 94,1 kcal/mol; (∆Hfo)SO2 = – 71 kcal/mol Prof. Pedro Madeira 54. (IME 1999 – Q05) Considerando que 100% do calor liberado na combustão de CH4 sejam utilizados para converter 100 kg de água a 100C em vapor a 100 0C, calcule o volume de metano consumido, medido nas CNTP, supondo que ele se comporte como um gás ideal. Dados: Constante universal dos gases (R) = 0,082 atm.l / mol.K Calor latente de vaporização da água = 2260 J / g; Calor específico da água = 4,2 J / g.0C Calor de combustão do metano = 890 kJ / mol 55. (IME 2001 – Q08) Uma mistura de metano e ar atmosférico, a 298K e 1atm, entra em combustão num reservatório adiabático, consumindo completamente o metano. O processo ocorre a pressão constante e os produtos formados (CO2, H2O, N2 e O2) permanecem em fase gasosa. Calcule a temperatura final do sistema e a concentração molar final de vapor d’água, sabendo-se que a pressão inicial do CH4 é de 1/16atm e a do ar é de 15/16atm. Considere o ar atmosférico constituído somente por N2 e O2 e o trabalho de expansão desprezível. Dados: Constante universal dos gases: R=0,082atm L.mol–1K–1 Entalpia de formação a 298K: CO2(g) = - 98.050 cal/mol H2O(g) = - 57.800 cal/mol CH4(g) = - 17.900 cal/mol Variação de entalpia (HºT-Hº298K) em cal/mol: T (K) CO2(g) H2O(g) N2(g) O2(g) 1.700 17.500 13.740 10.860 11.470 2.000 21.900 17.260 13.420 14.150 56. (IME 2002 – Q07) Uma amostra de 0,640 g de naftaleno sólido (C10H8) foi queimada num calorímetro de volume constante, produzindo somente dióxido de carbono e água. Após a reação, verificou-se um acréscimo de 2,4oC na temperatura do calorímetro. Sabendo-se que a capacidade calorífica do calorímetro era de 2.570 cal/oC e considerando-se que a variação de pressão foi muito pequena, calcule a entalpia de formação do naftaleno. Dados: 1) entalpia de formação do CO2 (g): - 94,1 kcal/mol 2) entalpia de formação da água (l): - 68,3 kcal/mol 100 APOSTILA 01 DE FÍSICO-QUÍMICA – PROF. PEDRO MADEIRA (2022) 57. (IME 2003 – Q06) O valor experimental para o calor liberado na queima de benzeno líquido a 25oC, com formação de dióxido de carbono e água líquida, é 780 kcal/mol. A combustão é feita em uma bomba calorimétrica a volume constante. Considerando comportamento ideal para os gases formados e R = 2,0 cal/mol.K, determine: a) o calor padrão de combustão do benzeno a 25oC; b) se o calor calculado no item anterior é maior ou menor quando a água é formada no estado gasoso. Justifique sua resposta. 58. (IME 2005 – Q03) O consumo de água quente de uma casa é de 0,489 m3 por dia. A água está disponível a 10,0oC e deve ser aquecida até 60,0oC pela queima de gás propano. Admitindo que não haja perda de calor para o ambiente e que a combustão seja completa, calcule o volume (em m3) necessário deste gás, medido a 25,0oC e 1,00 atm, para atender à demanda diária. DADOS: massa específica da água: 1,00 x 103 kg/m3 calor específico da água: 1,00 kcal/kgoC calores de formação a 298 K a partir de seus elementos: C3H8(g) = - 25,0 kcal/mol H2O(g) = - 58,0 kcal/mol CO2(g) = - 94,0 kcal/mol R = 82,0 x 10-6 m3 atm / K mol 59. (IME 2009 – Q35) Foram introduzidos 10 mols de uma substância X no interior de um conjunto cilindro-pistão adiabático, sujeito a uma pressão constante de 1atm. X reage espontânea e irreversivelmente segundo a reação: X(s) à 2 Y(g) DH = – 200 cal Considere que a temperatura no início da reação é 300 K e que as capacidades caloríficas molares das substâncias X e Y são constantes e iguais a 5,0 cal.mol– 1.K–1 e 1,0 cal.mol–1.K–1, respectivamente. O volume final do conjunto cilindro-pistão é (Dado: R = 0,082 atm.L.mol–1.K–1) A ( ) 410,0 L B ( ) 492,0 L C ( ) 508,4 L D ( ) 656,0 L E ( ) 820,0 L 60. (IME 2010 – Q07) A transformação isovolumétrica de um gás triatômico hipotético A3 em outro diatômico A2 envolve a liberação de 54 kJ/mol de A3. A capacidade calorífica molar a volume constante do gás A2 é de 30 J/mol.K. Após a transformação isocórica de todo A3 em A2, determine o aumento percentual de pressão em um recipiente isolado contendo o gás A3 a 27ºC. Considere que a capacidade calorífica molar a volume constante do gás A2 não varia com a temperatura e que os gases se comportam idealmente. 61. (IME 2012 – Q04) Na reação de formação de água líquida, a 1 atm e 298 K, o módulo da variação da entropia é 39,0 cal.K–1. mol– 1 e o módulo da variação da energia livre de Gibbs é 56.678 cal.mol–1. Considerando a combustão de 4,00 g de hidrogênio, a 1 atm e 298 K, calcule: a) a variação de energia interna na formação da água líquida; b) a variação de energia interna na formação da água gasosa; c) a variação de energia interna na vaporização de 1,00 mol de água. Considere,ainda, que todos os gases envolvidos comportam-se idealmente e que: H2O(g) à H2(g) + ½ O2(g) ΔH = 57.800 cal 62. (IME 2013 – Q08) Considere um recipiente adiabático conforme a ilustração abaixo, no qual 1000 g de uma solução aquosa de NaOH, a 30% em massa, e a uma temperatura inicial ti = 25oC, são diluídos a 20% em massa, com água à mesma temperatura. Calcule a temperatura tf da solução após a diluição. Dados: - Para o sistema NaOH – água a 25oC: a 30%: H = 104 J/g de solução; cP = 3,54 J/g.oC a 20%: H = 76 J/g de solução; cP = 3,63 J/g.oC - Calor específico da água líquida: cP = 4,18 J/g.oC - Estado de referência para entalpia: água líquida a 0oC. 63. (IME 2017 – Q35) Um isótopo de cromo, de massa atômica 54, constitui 53% da massa de um óxido formado exclusivamente pelo isótopo e por oxigênio. A partir dessa informação, pode-se estimar que a fórmula mínima do óxido e o calor específico do cromo-54 são: (A) CrO3 e 0,12 cal/(g.oC) (B) CrO3 e 0,18 cal/(g,oC) (C) Cr2O6 e 0,12 cal/(g.oC) (D) Cr2O3 e 0,16 cal/(g.oC) (E) Cr4O e 0,18 cal/(g.oC) Prof. Pedro Madeira 101 APOSTILA 01 DE FÍSICO-QUÍMICA – PROF. PEDRO MADEIRA (2022) TÓPICO 03: SEGUNDA E TERCEIRA LEIS DA TERMODINÂMICA SEÇÃO VESTIBULARES 64. (UECE 2006) A partir de seus conhecimentos sobre entropia, marque a alternativa que apresenta uma reação onde a variação de entropia é negativa: A) CaCO3(s) à CaO(s) + CO2(g) B) Zn(s) + 2 H+(aq) à H2(g) + Zn2+(aq) C) 2 C2H6(g) + 7 O2(g) à 4 CO2(g) + 6 H2O(g) D) N2(g) + 3 H2(g) à 2 NH3(g) 65. (UECE 2008) O conhecimento da energia livre é aplicado na indústria para a redução de gastos e otimização de alguns processos de produção. Considerando a reação: COCl2(g) CO(g) + Cl2(g) e os valores DH = – 108,28 kJ e DS = – 131,63 J/K a 25oC, assinale a alternativa que indica a temperatura na qual a reação é espontânea. A) 549ºC B) 627ºC C) 727ºC D) 823ºC 66. (CEFET 2003) Considere a reação abaixo a 1 atm para a qual se sabe que DH = + 22 Kcal e DS = + 3 cal/K: N2 (g) + O2(g) à 2NO(g) a) Calcule DG a 25ºC e 1 atm para a reação. b) A reação acima é espontânea em temperaturas baixas? Explique. c) Discuta o efeito das variações de temperatura sobre a posição do equilíbrio químico. 67. (CEFET 2004) A geração de metano, a partir do metanol, pode ser representada pela reação química abaixo: CH3OH(l) ® CH4(g) + 1/2 O2(g) a) Esta reação é espontânea a 25ºC e 1 atm? b) Qual o sinal de ΔS para esta reação? Qual a sua grandeza? c) Acima de que temperatura esta reação é espontânea? d) Que pode impedir a produção de CH4 por essa reação? Dados: 1. Energia Livre de Formação (kcal/mol) a 25ºC e 1 atm. Substância DGfº (kcal/mol) CH3OH(l) CH4(g) - 39,7 - 12,1 2. Calor de Formação (kcal/mol) a 25ºC e 1 atm. Substância DHfº (kcal/mol) CH3OH(l) CH4(g) - 57,0 - 17,9 68. (CEFET 2005) Estime a temperatura na qual o CuSO4 . 5 H2O sofre desidratação, formando o anidrido sólido. Composto DHf o (kJ.mol–1) Sm o(J.K.mol–1) CuSO4.5H2O(s) – 2279,7 300,4 CuSO4(s) – 771,36 109 H2O(g) – 241,82 188,83 69. (CEFET 2006) À temperatura e à pressão constantes, a energia de Gibbs (G) constitui um critério de espontaneidade para os fenômenos físicos e químicos da natureza. A variação da energia de Gibbs, nesse caso, obedece à equação: ΔG = ΔH – TΔS onde as contribuições, para o valor de ΔG, são energética (ΔH) e entrópica (ΔS). Nesse sentido, responda aos itens abaixo: a) Qual é o valor de ΔG para a ebulição da H2O a 100ºC e 1,00 atm? E o sinal do ΔS para o processo? b) Quais os sinais de ΔG, ΔH e ΔS para a decomposição da H2O(l), em H2(g) e O2(g)? c) A vaporização da H2O(l) é um processo espontâneo? Explique em termos das variações de G, H e S. SEÇÃO ITA / IME 70. (ITA 1999 – Q13) O processo de decomposição de peróxido de hidrogênio, H2O2, resulta na formação de água e oxigênio. Em relação a esse processo considere que sejam feitas as seguintes afirmações: I. Todas as moléculas de H2O2 são reduzidas. II. Todas as moléculas de H2O2 são oxidadas. III. A variação da energia livre de Gibbs é positiva. IV. Metade das moléculas de H2O2 é reduzida e a outra metade é oxidada. Qual das opções abaixo se refere à(s) afirmação(ões) CORRETA(S)? A ( ) I B ( ) II C ( ) III D ( ) IV E ( ) III e IV 71. (ITA 2006 – Q26) Para cada um dos processos listados abaixo, indique se a variação de entropia será maior, menor ou igual a zero. Justifique suas respostas. a) N2 (g,1 atm,T = 300 K) → N2 (g,0,1 atm,T = 300 K) b) C (grafite) → C(diamante) c) solução supersaturada → solução saturada d) sólido amorfo → sólido cristalino e) N2 (g) → N2 (g, adsorvido em sílica) Prof. Pedro Madeira 72. (ITA 2009 – Q02) No ciclo de Carnot, que trata do rendimento de uma máquina térmica ideal, estão presentes as seguintes transformações: A ( ) duas adiabáticas e duas isobáricas. B ( ) duas adiabáticas e duas isocóricas. C ( ) duas adiabáticas e duas isotérmicas. D ( ) duas isobáricas e duas isocóricas. E ( ) duas isocóricas e duas isotérmicas. 102 APOSTILA 01 DE FÍSICO-QUÍMICA – PROF. PEDRO MADEIRA (2022) 73. (ITA 2012 – Q13) Considere as reações representadas pelas seguintes equações químicas: I. C(s) + 2 H2(g) à CH4(g) II. N2O(g) à N2(g) + 1/2 O2(g) III. 2 NI3(s) à N2(g) + 3 I2(g) IV. 2 O3(g) à 3 O2(g) Assinale a opção que apresenta a(s) reação(ões) química(s) na(s) qual(is) há uma variação negativa de entropia. A ( ) apenas I B ( ) apenas II e IV C ( ) apenas II e III e IV D ( ) apenas III E ( ) apenas IV 74. (ITA 2013 – Q23) Em um gráfico de pressão versus volume, representa o diagrama do ciclo idealizado por Carnot (máquina térmica) para uma transformação cíclica, ininterrupta, e sem perdas de calor e de trabalho, e vice-versa. Identifique e denomine as quatro etapas dessa transformação cíclica. 75. (ITA 2014 – Q04) Considere que 1 mol de uma substância sólida está em equilíbrio com seu respectivo líquido na temperatura de fusão de −183°C e a 1 atm. Sabendo que a variação de entalpia de fusão dessa substância é 6,0 kJ ∙ mol−1, assinale a opção que apresenta a variação de entropia, em J ∙ K−1∙mol−1. A ( ) −20 B ( ) −33 C ( ) +50 D ( ) +67 E ( ) + 100 76. (ITA 2015 – Q15) Para uma molécula diatômica, a energia potencial em função da distância internuclear é representada pela figura ao lado. As linhas horizontais representam os níveis de energia vibracional quanticamente permitidos para uma molécula diatômica. Uma amostra contendo um mol de moléculas diatômicas idênticas, na forma de um sólido cristalino, pode ser modelada como um conjunto de osciladores para os quais a energia potencial também pode ser representada qualitativamente pela figura. Em relação a este sólido cristalino, são feitas as seguintes proposições: I. À temperatura de 0 K, a maioria dos osciladores estará no estado vibracional fundamental, cujo número quântico vibracional, n, é igual a zero. II. À temperatura de 0 K, todos os osciladores estarão no estado vibracional fundamental, cujo número quântico vibracional, n, é igual a zero. III. O movimento vibracional cessa a 0 K. IV. O movimento vibracional não cessa a 0 K. V. O princípio de incerteza de Heisenberg será violado se o movimento vibracional cessar. Das proposições acima estão CORRETAS A ( ) apenas I e III. B ( ) apenas II e III. C ( ) apenas I, IV e V. D ( ) apenas II, IV e V. E ( ) apenas II, III e V. 77. (ITA 2017 – Q11) Em relação às funções termodinâmicas de estado de um sistema, assinale a proposição ERRADA. A ( ) A variação de energia interna é nula na expansão de n mols de um gás ideal a temperatura constante. B ( ) A variação de energia interna é maior do que zero em um processo endotérmico a volume constante.C ( ) A variação de entalpia é nula em um processo de várias etapas em que os estados inicial e final são os mesmos. D ( ) A variação de entropia é maior do que zero em um processo endotérmico a pressão constante. E ( ) A variação de entropia é nula quando n mols de um gás ideal sofrem expansão livre contra pressão externa nula. 78. (ITA 2018 – Q13) Um recipiente de paredes adiabáticas e de volume constante contém duas amostras de água pura separadas por uma parede também adiabática e de volume desprezível. Uma das amostras consiste em 54g de água a 25ºC e, a outra, em 126g a 75ºC. Considere que a parede que separa as amostras é retirada e que as amostras de água se misturam até atingir o equilíbrio. Sobre esse processo são feitas as seguintes afirmações: I. A temperatura da mistura no equilíbrio é de 323 K. II. A variação de entalpia no processo é nula. III. A variação de energia interna no processo é nula. IV. A variação de entropia no processo é nula. Assinale a opção que apresenta a(s) afirmação(ões) CORRETA(S) sobre a mistura das amostras de água. A ( ) Apenas I B ( ) Apenas I e II C ( ) Apenas II e III D ( ) Apenas III e IV E ( ) Apenas IV 79. (ITA 2021 – Q63) Considere as seguintes proposições sobre processos termodinâmicos: I. A entropia permanece constante em um sistema fechado que sofre a ação de um processo reversível. 103 APOSTILA 01 DE FÍSICO-QUÍMICA – PROF. PEDRO MADEIRA (2022) II. A variação de entropia é nula dentro do sistema quando ele opera em um ciclo de Carnot. III. O valor absoluto da variação da energia interna de um gás ideal numa expansão reversível adiabática é maior que numa expansão reversível isotérmica. IV. Energia interna é uma propriedade cuja variação pode ser medida pelo trabalho adiabático realizado entre dois estados. Das afirmações acima, está(ão) ERRADA(S) apenas A ( ) I. B ( ) I,II e IV. C ( ) II e III. D ( ) III. E ( ) IV. 80. (IME 1992 – Q07) Calcule o valor da variação da energia livre, a 25ºC, para a reação representada a seguir. 2 Na2O2(s) + 2 H2O(l) à 4 NaOH(s) + O2(g) Substância Entalpia de formação a 25ºC kJ mol–1 Sº a 25ºC J mol–1K–1 H2O(l) – 286,0 69,69 Na2O2(s) – 510,9 94,60 NaOH(s) – 426,8 64,18 O2(g) 0 205,00 81. (IME 1993 – Q04) Calcule a mudança de energia interna, em kJ, para a reação de formação de dois moles de SOCl2 (g) a partir de S(g), O2 (g) e Cl2 (g) a 298K. Dados: ∆P8 1 S(g) = 277 kJ.mol–1 ∆P8 1 SOCl2(g) = – 210kJ.mol–1 82. (IME 1994 – Q07) A variação da energia livre (DG) e a variação de entropia (DS), para a transformação do enxofre ortorrômbico em sua forma alotrópica monoclínica, são positivas nas CNTP. Responda: a) Qual das duas formas alotrópicas é mais estável a 273 K e 101325 Pa; e b) Qual o sinal para a variação de entalpia ( DH) da transformação, também a 273 K e 101325 Pa? 83. (IME 1996 – Q09) Uma fábrica, que produz cal (Ca(OH)2), necessita reduzir o custo da produção para se manter no mercado com preço competitivo para seu produto. A direção da fábrica solicitou ao departamento técnico o estudo da viabilidade de reduzir a temperatura do forno de calcinação de carbonato de cálcio, dos atuais 1500 K, para 800 K. Considerando apenas o aspecto termodinâmico, pergunta-se: o departamento técnico pode aceitar a nova temperatura de calcinação? Em caso, afirmativo, o departamento técnico pode fornecer uma outra temperatura de operação que proporcione maior economia? Em caso negativo, qual é a temperatura mais econômica para se operar o forno de calcinação? Dados: DS° (J mol-1.K-1) DH° (kJ mol-1) CaCO3(s) 92,9 - 1206.9 CaO(s) 39,8 - 635,1 CO2(g) 213,6 - 393,5 Observação: desconsidere a variação das propriedades com a temperatura. 84. (IME 2007 – Q38) Considere os seguintes processos conduzidos a 25ºC e 1 atm: (1) 4Fe(s) + 3O2 (g) → 2Fe2O3(s) (2) H2O(s) → H2O(l) (3) CH4(g) + 2O2 (g) → CO2(g) + 2H2O(g) (4) Cu2S(s) → 2Cu(s) + S(s), com ΔG = + 86,2 kJ (5) S(s) + O2(g) → SO2(g), com ΔG = − 300,4 kJ (6) Cu2S(s) + O2(g) → 2Cu(s) + SO2(g) (7) 2NO(g) + O2(g) → 2NO2(g) Assinale a afirmativa correta. A ( ) Os processos (1), (4) e (5) não são espontâneos. B ( ) O processo (2) é exotérmico e apresenta variação de entropia positiva. C ( ) O processo (3) é endotérmico e apresenta variação de entropia negativa. D ( ) Os processos (2) e (7) apresentam variação de entropia positiva. E ( ) Os processos (1), (2) e (6) são espontâneos. 85. (IME 2010 – Q36) Considere as supostas variações de entropia (ΔS) nos processos abaixo: I) cristalização do sal comum (ΔS > 0) II) sublimação da naftalina (naftaleno) (ΔS > 0) III) mistura de água e álcool (ΔS < 0) IV) ferro(s) ferro (l) (ΔS > 0) V) ar ar comprimido (ΔS < 0) As variações de entropia indicadas nos processos que estão corretas são: A) I, III e IV B) III, IV e V C) II, III e V D) I, II e IV E) II, IV e V 86. (IME 2011 – Q36) A entalpia de fusão de uma determinada substância é 200 kJ/kg, e seu ponto de fusão normal é 27oC. Após a solidificação de 3 kg do material, pode-se afirmar que a entropia desse sistema: A) diminuiu 2 kJ/K B) diminuiu 600 kJ/K C) não variou D) aumentou 2 kJ/K E) aumentou 600 kJ/K 87. (IME 2013 – Q37) O dispositivo a seguir utiliza a radiação solar para quantificar variações em propriedades termodinâmicas. Este dispositivo é composto por uma lente convergente e por um porta-amostras. A lente possui área útil de 80,0 cm2, absortividade (a) de 20% e transmissividade (t) de 80%. O porta-amostras possui absortividade de 100% e ¾¾ ®¾fusão ¾¾¾ ®¾compressão 104 APOSTILA 01 DE FÍSICO-QUÍMICA – PROF. PEDRO MADEIRA (2022) volume variável, operando à pressão constante de 1,0 atm. Em um procedimento experimental, injetou-se 0,100 mol de uma substância pura líquida no porta-amostras do dispositivo. Em seguida, mediu-se um tempo de 15,0 min para a vaporização total da amostra, durante o qual a irradiação solar permaneceu constante e igual a 750 W/m2. Nesse processo, a temperatura do porta- amostras estabilizou-se em 351 K. No experimento, o calor sensível da amostra e a radiação emitida pelo porta-amostras são desprezíveis. Pode-se concluir que na vaporização total da substância, as variações de entalpia molar padrão e de entropia molar padrão são, respectivamente: (A) 4,32 kJ/mol e 12,3 J/(mol K) (B) 5,40 kJ/mol e 15,4 J/(mol K) (C) 43,2 kJ/mol e 123 J/(mol K) (D) 54,0 kJ/mol e 154 J/(mol K) (E) 31,6 kJ/mol e 90,0 J/(mol K) 88. (IME 2014 – Q34) A variação de entropia de um sistema fechado constituído por um gás ideal, quando sofre uma transformação, pode ser calculada pela expressão genérica: em que os subscritos 1 e 2 representam dois estados quaisquer. Assinale a única afirmativa correta. (A) Se o estado inicial 1 é diferente do estado final 2, a variação da entropia do gás ideal não depende da quantidade de gás presente no sistema. (B) Se a mudança de estado é isotérmica, a variação da entropia é dada por . (C) Se o sistema realiza um processo cíclico, a variação de entropia é positiva. (D) Se a mudança de estado é isobárica, a variação de entropia é dada por . (E) Se a mudança de estado é isocórica, a variação da entropia do sistema é nula. 89. (IME 2016 – Q38) Um sistema A transfere, naturalmente, uma determinada quantidade de energia, na forma de calor, para um sistema B, que envolve totalmente A. Assinale a única alternativa correta. (A) A entropia do Universo decrescerá. (B) A entropia do sistema A crescerá. (C) O aumento da entropia do sistema B será maior do que o decréscimo da entropia do sistema A. (D) O aumento da entropia do sistema B será menor do que o decréscimo da entropia do sistema A. (E) O aumento da entropia do sistema B será necessariamente igual ao decréscimoda entropia do sistema A. 90. (IME 2016 – Q40) Um sistema é composto por dois balões idênticos resistentes, porém não inquebráveis, A e B, os quais estão conectados por meio de um tubo, também resistente, no qual se encontra uma válvula, tipo torneira. Este sistema encontra-se perfeitamente isolado termicamente do universo. Inicialmente as condições do sistema são as seguintes: temperatura constante; a válvula encontra-se fechada; o balão A contém um mol de um gás ideal monoatômico; e o balão B encontra-se perfeitamente evacuado. No tempo t = 0, a torneira é aberta repentinamente, permitindo que o gás ideal se expanda em direção ao balão B por um orifício pequeno. Indique qual das alternativas abaixo é a correta. (A) O balão B quebrar-se-á devido ao impacto do gás ideal, liberado bruscamente, contra sua parede. (B) O trabalho gerado pela expansão do gás aquecerá o sistema. (C) O gás em expansão absorverá calor da vizinhança fazendo o sistema se resfriar. (D) O valor da variação da energia interna ∆U da expansão será igual a zero. (E) Na expansão, a variação da energia interna ∆U do sistema será menor que zero. 91. (IME 2016 – Q05) Um bloco de gelo a 0,00oC é colocado em contato com um recipiente fechado que contem vapor de água a 100 oC e 1 atm. Após algum tempo, separa-se o bloco de gelo do recipiente fechado. Nesse instante observa-se que 25,0 g de gelo foram convertidos em água líquida a 0,00oC, e que no recipiente fechado existem água líquida e vapor d’água em equilíbrio. Considerando que o bloco de gelo e o recipiente fechado formam um sistema e que só trocam calor entre si, calcule a variação de entropia do sistema. DADO: ∆X@ABã= = EEB [\/[^ 92. (IME 2018 – Q39) Considere as seguintes afirmativas: I – Uma reacao quimica a temperatura e pressao constantes sera espontanea se a variacao da energia livre de Gibbs (ΔG) for menor que zero. II – Em um sistema reacional onde a unica forma de trabalho observavel e o trabalho de expansao, a variação da entalpia (ΔH) e igual a quantidade de calor liberada ou absorvida pela reacao, a pressao constante. III – Para uma substancia simples que admite mais de uma forma alotropica, nao ha variacao de entalpia na conversao de uma forma em outra. 2 2 1 1 T p ΔS= nC ln nRlnp T p - 2 1 p ΔS= nC lnp p - 2 1 T ΔS= nC lnp T 105 APOSTILA 01 DE FÍSICO-QUÍMICA – PROF. PEDRO MADEIRA (2022) Sao corretas: (A) Somente I. (B) Somente II. (C) Somente III. (D) I e II. (E) I e III. 93. (IME 2019 – Q33) Considere as reações abaixo: H2(g) + ½ O2(g) → H2O(ℓ) (I) H2(g) + ½ O2(g) → H2O(g) (II) Assinale a alternativa correta. (A) O decréscimo de entropia é menor na reação (I) do que na reação (II). (B) O acréscimo de entropia na reação (I) é maior do que na reação (II). (C) O decréscimo de entropia é menor na reação (II) do que na reação (I). (D) O acréscimo de entropia na reação (II) é maior do que na reação (I). (E) A variação de entropia é igual em ambas as reações. 94. (IME 2019 – Q03) Mistura-se a água contida em dois recipientes, designados por A e B, de forma adiabática. Cada um contém a mesma massa m de água no estado líquido. Inicialmente, as temperaturas são T no recipiente A e T + ΔT no recipiente B. Após a mistura, a água atinge a temperatura final de equilíbrio térmico. Mostre que a variação de entropia do processo de mistura é positiva. Dado: ΔS = m Cp ln D# D$ , onde T2 e T1 são duas temperaturas em dois estados diferentes do processo e cp é o calor específico da água, considerado constante. 95. (IME 2020 – Q36) O astrônomo britânico Arthur Eddington cunhou o termo “seta do tempo” para distinguir uma direção no tempo nos fenômenos naturais, ou seja, o fato de que o estado 2 de um sistema macroscópico ocorre após o estado 1. Podemos afirmar que o valor da entropia do estado 2 de um sistema fechado que evoluiu a partir do estado 1: (A) é igual ao valor da entropia do estado 1. (B) é menor que o valor da entropia do estado 1. (C) é maior que o valor da entropia do estado 1. (D) independe do valor da entropia do estado 1. (E) depende do caminho percorrido entre os estados. Prof. Pedro Madeira 96. (IME 2022 – Q32) Uma reação entre dois líquidos A e B produz dois compostos gasosos C e D, de acordo com a estequiometria A + B → C + D. Se conduzida a pressão e temperatura constantes, pode-se afirmar que: (A) a reação será sempre espontânea, se for endotérmica. (B) a reação será sempre espontânea, se for exotérmica. (C) a reação será sempre espontânea, independentemente de ser exotérmica ou endotérmica. (D) a reação nunca será espontânea, independentemente de ser exotérmica ou endotérmica. (E) não há como prever a espontaneidade da reação, mesmo que informações adicionais sobre o calor de reação estejam disponíveis. 97. (IME 2022 – Q03) Sob determinadas condições, a água pode ser super- resfriada, ou seja, permanecer no estado líquido em temperaturas inferiores ao seu ponto de congelamento, em uma situação termodinamicamente instável. Considere um processo em que 5,0 mol de água super- resfriada a –10ºC e 1,0 atm sejam convertidos em gelo à mesma temperatura. Determine a variação de entropia: a) do sistema; b) na vizinhança; e c) do universo. TÓPICO 04: FUNDAMENTOS DE TERMOQUÍMICA SEÇÃO VESTIBULARES 98. (UFC 2001) A ligação tripla é muito forte A natureza atua na fixação do nitrogênio de diversas maneiras. Uma destas, que é responsável por cerca de somente 10% do processo natural total, é proveniente da ação da descarga elétrica dos raios sobre a massa atmosférica, que transforma o nitrogênio em óxido nítrico e, posteriormente, em dióxido de nitrogênio. O NO2, por sua vez, reage com a água das chuvas produzindo HNO3, que é, então, incorporado ao solo. Dadas as energias de ligação: N2 225 kcal/mol O2 118 kcal/mol NO 162 kcal/mol Assinale a alternativa correta. A) O processo descrito é acompanhado da formação seqüenciada de espécies de mais baixos estados de oxidação do nitrogênio. B) A fixação de nitrogênio é acompanhada de processos seqüenciados de redução, conduzindo à elevação do estado de oxidação do nitrogênio. C) Uma dificuldade admitida para a fixação do nitrogênio é a elevada quantidade de energia requerida para quebrar a tripla ligação N≡N. D) Somente com base nos valores das energias das ligações, espera-se que o processo de formação do NO seja termoquimicamente espontâneo. E) O processo descrito constitui-se de uma fonte natural de inibição da formação de chuvas ácidas, seguido de neutralização. 99. (UFC 2007) A força dos retículos A energia de rede (U) para um composto iônico MX pode ser definida como a energia necessária para ocorrer a seguinte reação: MX(s) à M+(g) + X–(g) 2 2 2 2 2 3 O ,h O H ON NO NO HNOn¾¾¾¾® ¾¾¾® ¾¾¾® 106 APOSTILA 01 DE FÍSICO-QUÍMICA – PROF. PEDRO MADEIRA (2022) Considere os seguintes compostos: NaF, NaCl, CaF2, CaCl2, LiF e LiCl. Com base nas informações, assinale a alternativa correta. A) Todos os compostos apresentados são espécies apolares. B) A temperatura de fusão do LiCl é maior que a temperatura de fusão do LiF. C) A temperatura de fusão do NaF é menor que a temperatura de fusão do NaCl. D) O módulo da energia de rede do LiCl é maior que o módulo da energia de rede do LiF. E) O módulo da energia de rede do CaF2 é maior que o módulo da energia de rede do CaCl2. 100. (UFC 1997) Hidrogenações são exotérmicas A reação de hidrogenação de compostos olefínicos é amplamente utilizada na fabricação de margarinas a partir de óleos vegetais. a) Determine o DHo de hidrogenação, sabendo-se que o DHº do C2H4(g) é 12,50 Kcal.mol–1, e DHº do C2H6(g) é –20,20 Kcal.mol–1. b) Utilizando-se de fórmulas estruturais, mostre a equação de hidrogenação catalítica (H2/Pt) do Eteno. 101. (UFC 2001)Observe o gás d’água O gás d’água, uma importante mistura constituída de CO e H2, utilizada na indústria, é preparado através da passagem de um fluxo de água gasosa sobre carvão, a 1000 oC. (I) C(s) + H2O(g) à CO(g) + H2(g) Dadas as seguintes informações: (II) C(s) + O2(g) à CO2(g); DHo = -393,5kJ (III) 2H2(g) + O2(g) à 2H2O(g); DHo = -483,6kJ (IV) 2CO(g) + O2(g) à 2CO2(g); DHo =-566,0kJ A) calcule o valor de DHo para a formação do gás d’água (I), e classifique a reação termoquimicamente. B) represente as estruturas de Lewis para os agentes oxidante e redutor da reação (I), somente os que constituem substâncias químicas compostas. 102. (UFC 2003) Comparação entre combustíveis A quantidade de energia liberada na queima de combustíveis é denominada entalpia de combustão. As entalpias de combustão de algumas substâncias são dadas na tabela I. Para comparar a eficiência da combustão de diferentes combustíveis, são necessárias especificações de algumas condições. TABELA I Substância Entalpia de combustão (kJ.mol-1) Densidade (g/mL) Hidrogênio, H2 –286 0,07 (*) Metanol , CH3OH –726 0,79 Etanol, C2H5OH –1367 0,80 Octano, C8H18 –5470 0,70 (*) calculada para o gás comprimido a 0 oC Identifique, dentre as substâncias relacionadas na tabela I, a que teria a maior eficiência nas seguintes situações: A) aplicações nas quais o uso da menor massa do combustível é o mais importante requerimento, por exemplo nos foguetes espaciais. Justifique. B) aplicações nas quais o uso do menor volume do combustível é o mais importante requerimento, por exemplo, nos veículos automotores. Justifique. 103. (FUVEST 2019) O tricloreto de nitrogênio (NCℓ3), ou tricloramina, é um composto de aparência oleosa muito perigoso quando puro, pois se decompõe explosivamente formando N2 e Cℓ2. Ele era utilizado, até sua proibição na década de 1940, para o branqueamento de farinhas comerciais, no chamado Processo Agene®. Atualmente, é conhecido como um dos subprodutos tóxicos indesejáveis formados no processo de desinfecção de piscinas. Ele se forma, por exemplo, pela reação do cloro usado nas piscinas com ureia proveniente da urina de nadadores que as utilizam. Pede-se: a) Represente a molécula do NCℓ3 utilizando fórmula de Lewis. b) Escreva a equação balanceada para a decomposição do NCℓ3 em N2 e Cℓ2. Qual é o número de oxidação do nitrogênio no reagente e no produto? c) Calcule a entalpia da reação de decomposição do NCℓ3, considerando os valores de energia de ligação fornecidos. A reação é endotérmica ou exotérmica? Demonstre seus cálculos e justifique sua resposta. Note e adote: Ligação Energia de ligação N≡N 940 Cℓ–Cℓ 240 N–Cℓ 200 104. (UFC 2006) O processo Haber é exotérmico Dadas as reações: I. H2 (g) + Cl2 (g) à 2 HCl (g) II. N2 (g) + 3 H2 (g) à 2 NH3 (g) e as energias de ligação: Ligação Entalpia de ligação (kJ/mol) H–H 432 NºN 942 H–Cl 428 Cl–Cl 240 N–H 386 A) Determine o DH para as reações I e II B) Baseado apenas nos valores de DH, qual das reações é mais favorável? Prof. Pedro Madeira 105. (UFC 2008) Será que evapora tudo? Considere um recipiente hermeticamente fechado com capacidade de 1000 L e a uma temperatura de 27oC, (kJ mol) 107 APOSTILA 01 DE FÍSICO-QUÍMICA – PROF. PEDRO MADEIRA (2022) onde é adicionado 1 L de água. Despreze os efeitos da temperatura sobre a densidade da água. Dados: densidade da água = 1g.mL–1; pressão de vapor da água a 27oC = 0,035 atm. A) Nessas condições, haverá a evaporação completa desta massa de água? Justifique numericamente a sua resposta, considerando gás com comportamento ideal. B) Sabendo que o calor de vaporização da água a 100oC é 40,7 kJ.mol–1, qual deverá ser a quantidade de calor necessária para vaporizar 1 L de água? 106. (UNESP 2022) Ciclo de Born-Haber clássico A variação de entalpia, associada à formação de um cristal iônico sólido a partir de seus íons no estado gasoso, é conhecida como energia reticular. Essa energia é difícil de ser medida diretamente, mas pode ser calculada de forma indireta, utilizando-se a Lei de Hess, a partir de outras transformações, cuja variação de entalpia é conhecida. Esse caminho para a determinação da energia reticular é conhecido como ciclo de Born-Haber. O diagrama a seguir mostra as etapas desse ciclo para o cloreto de sódio. Nesse diagrama, a sublimação do sódio metálico, a primeira energia de ionização do elemento sódio e a afinidade eletrônica do elemento cloro correspondem, respectivamente, aos valores de a) DH2, DH3 e DH4 b) DH1, DH0 e DH5 c) DH1, DH4 e DH3 d) DH2, DH4 e DH3 e) DH1, DH3 e DH4 107. (UECE 2010) Um Born-Haber estranho O fluoreto de cálcio, CaF2, usado como fluxo na fabricação de ácido; como fundente, na obtenção de HF e esmaltação de utensílios de cozinha, pode ser preparado seguindo o ciclo de Born Haber a partir do cálcio e do gás flúor. Solicitou-se a um aluno para elaborar o ciclo, que, após concluído, apresentou um equivoco do aluno. Assinale a alternativa que mostra esse equívoco. A) Não são 2xDHAE, mas somente DHAE. B) Ao invés de Edis o correto são 2Edis. C) As posições de DEret e DHo f foram trocadas entre si. D) Como os 2 elétrons do cálcio são retirados de uma única vez não deve ser usado a energia do 1o Potencial de Ionização. 108. (CEFET 2003) Hidrazina é combustível de foguete Os foguetes espaciais utilizam como propelente uma mistura de hidrazina (N2H4), como combustível, e peróxido de hidrogênio (H2O2), como oxidante. Esses reagentes iniciam a reação pelo simples contato (reagentes hipergólicos). A reação em questão é: N2H4(ℓ) + 2H2O2(ℓ) ¾® N2(g) + 4H2O(g) 25ºC Calcule o calor liberado na reação, na temperatura considerada, sabendo que as entalpias padrão de formação são as seguintes: N2H4(ℓ) = + 12 Kcal/mol 2H2O2(ℓ) = -46 Kcal/mol H2O(g) = -58 Kcal/mol 109. (UNICAMP 1999) Hidrazina é combustível de foguete A hidrazina (H2N–NH2) tem sido utilizada como combustível em alguns motores de foguete. A reação de combustão que ocorre pode ser representada, simplificadamente, pela seguinte equação: H2N–NH2(g) + O2(g) = N2(g) + 2 H2O(g) A variação de entalpia dessa reação pode ser estimada a partir dos dados de entalpia das ligações químicas envolvidas. Para isso, considera-se uma absorção de energia quando a ligação é rompida, e uma liberação de energia quando a ligação é formada. A tabela abaixo apresenta dados de entalpia por mol de ligações rompidas. DEret Ca(s) + F2(g) CaF2(s) Edis DHo f DHo sub 2xDHAE 2F–(g) 2F–(g) DH(1oPI) + Ca(g) Ca2+(g) DH(2oPI) 108 APOSTILA 01 DE FÍSICO-QUÍMICA – PROF. PEDRO MADEIRA (2022) a) Calcule a variação de entalpia para a reação de combustão de um mol de hidrazina. b) Calcule a entalpia de formação da hidrazina sabendo- se que a entalpia de formação da água no estado gasoso é de – 242 kJ mol–1. 110. (UNICAMP 2000) Considere uma gasolina constituída apenas de etanol e de n-octano, com frações molares iguais. As entalpias de combustão do etanol e do n-octano são –1368 e – 5471 kJ/mol, respectivamente. A densidade dessa gasolina é 0,72 g/cm3 e a sua massa molar aparente, 80,1 g/mol. a) Escreva a equação química que representa a combustão de um dos componentes dessa gasolina. b) Qual a energia liberada na combustão de 1,0 mol dessa gasolina? c) Qual a energia liberada na combustão de 1,0 litro dessa gasolina? 111. (UNICAMP 2002) Examinando os copos com restos de café e de café com leite, Rango observa que apenas o de café apresenta impressões digitais,as quais coincidem com as do guarda. – Estranho! – disse ele. – Este outro copo não apresenta impressões! Talvez alguém usando luvas... – Ou talvez uma criança! – emendou Estrondosa. A observação de Estrondosa se baseou no fato de que a impressão digital de uma criança é composta principalmente por ácidos graxos (ácidos orgânicos) de cadeia contendo até 13 átomos de carbono, enquanto as dos adultos se compõem, principalmente, de ésteres contendo 32 átomos de carbono. O gráfico a seguir mostra a entalpia de sublimação de ésteres e de ácidos orgânicos em função do número de átomos de carbono na cadeia. a) Considerando o mesmo número de átomos de carbono na molécula, os ácidos apresentam maior entalpia de sublimação. Que tipo de interação entre suas moléculas poderia justificar esse fato? Explique. b) Determine a entalpia de sublimação do éster contendo 32 átomos de carbono, admitindo que as curvas se comportam do mesmo modo para moléculas contendo maior número de átomos de carbono. Prof. Pedro Madeira 112. (UNICAMP 2005) Uma das grandes novidades em comunicação é a fibra óptica. Nesta, a luz é transmitida por grandes distâncias sem sofrer distorção ou grande atenuação. Para fabricar fibra óptica de quartzo, é necessário usar sílica de alta pureza, que é preparada industrialmente usando uma seqüência de reações cujas equações (não balanceadas) estão representadas a seguir: I – SiO2(s) + C(s) à Si(s) + CO2(g) II – Si(s) + Cl2(g) à SiCl4(g) III – SiCl4(g) + O2(g) à SiO2(s) + Cl2(g) a) Na obtenção de um tarugo de 300g de sílica pura, qual a quantidade de energia (em kJ) envolvida? Considere a condição padrão. Dados de entalpia padrão de formação em kJ.mol–1: SiO2(s) = – 910; CO2 = – 394; SiCl4(g) = – 657. b) Com a sílica produzida (densidade = 2,2 g.cm–3), foi feito um tarugo que, esticado, formou uma fibra de 0,06 mm de diâmetro. Calcule o comprimento da fibra esticada, em metros. PROF. PEDRO MADEIRA 113. (FUVEST 2003) Comparação de isômeros O 2-metilbutano pode ser obtido pela hidrogenação catalítica, em fase gasosa, de qualquer dos seguintes alcenos isoméricos: 2-metil-2-buteno + H2 à 2-metilbutano; ΔH1=–113 kJ/mol 2-metil-1-buteno + H2 à 2-metilbutano ΔH2 = –119 kJ/mol 3-metil-1-buteno + H2 à 2-metilbutano ΔH3 = –127 kJ/mol a) Complete o esquema da página ao lado com a fórmula estrutural de cada um dos alcenos que faltam. Além disso, ao lado de cada seta, coloque o respectivo ΔH de hidrogenação. b) Represente, em uma única equação e usando fórmulas moleculares, as reações de combustão completa dos três alcenos isoméricos. c) A combustão total de cada um desses alcenos também leva a uma variação negativa de entalpia. Essa variação é igual para esses três alcenos? Explique. 109 APOSTILA 01 DE FÍSICO-QUÍMICA – PROF. PEDRO MADEIRA (2022) 114. (FUVEST 2004) O Veículo Lançador de Satélites brasileiro emprega, em seus propulsores, uma mistura de perclorato de amônio sólido (NH4ClO4) e alumínio em pó, junto com um polímero, para formar um combustível sólido. a) Na decomposição térmica do perclorato de amônio, na ausência de alumínio, formam-se quatro produtos. Um deles é a água e os outros três são substâncias simples diatômicas, duas das quais são componentes naturais do ar atmosférico. Escreva a equação balanceada que representa essa decomposição. b) Quando se dá a ignição do combustível sólido, todo o oxigênio liberado na decomposição térmica do perclorato de amônio reage com o alumínio, produzindo óxido de alumínio (Al2O3). Escreva a equação balanceada representativa das transformações que ocorrem pela ignição do combustível sólido. c) Para uma mesma quantidade de NH4ClO4, haverá uma diferença de calor liberado se sua decomposição for efetuada na presença ou na ausência de alumínio. Quanto calor a mais será liberado se 2 mols de NH4ClO4 forem decompostos na presença de alumínio? Mostre o cálculo. Dado: Calor de formação do óxido de alumínio = –1,68x103 kJ/mol 115. (FUVEST 2005) BOX de explosivos Define-se balanço de oxigênio de um explosivo, expresso em percentagem, como a massa de oxigênio faltante (sinal negativo) ou em excesso (sinal positivo), desse explosivo, para transformar todo o carbono, se houver, em gás carbônico e todo o hidrogênio, se houver, em água, dividida pela massa molar do explosivo e multiplicada por 100. O gráfico ao lado traz o calor liberado na decomposição de diversos explosivos, em função de seu balanço de oxigênio. Um desses explosivos é o tetranitrato de pentaeritritol (PETN, C5H8N4O12). A equação química da decomposição desse explosivo pode ser obtida, seguindo-se as seguintes regras: - Átomos de carbono são convertidos em monóxido de carbono. - Se sobrar oxigênio, hidrogênio é convertido em água. - Se ainda sobrar oxigênio, monóxido de carbono é convertido em dióxido de carbono. - Todo o nitrogênio é convertido em nitrogênio gasoso diatômico. a) Escreva a equação química balanceada para a decomposição do PETN. b) Calcule, para o PETN, o balanço de oxigênio. c) Calcule o ΔH de decomposição do PETN, utilizando as entalpias de formação das substâncias envolvidas nessa transformação. d) Que conclusão é possível tirar, do gráfico apresentado, relacionando calor liberado na decomposição de um explosivo e seu balanço de oxigênio? Massa Molar PETN = 316 g.mol–1. Substância PETN(s) CO2(g) CO(g) H2O(g) Entalpia de formação kJ mol–1 –538 –394 –110 –242 Prof. Pedro Madeira 116. (ENEM PPL 2018) Ele nela, nunca ela nele Sobre a diluição do ácido sulfúrico em água, o químico e escritor Primo Levi afirma que, “está escrito em todos os tratados, é preciso operar às avessas, quer dizer, verter o ácido na água e não o contrário, senão aquele líquido oleoso de aspecto tão inócuo está sujeito a iras furibundas: sabem-no até os meninos do ginásio”. (furibundo: adj. furioso) LEVI, P. A tabela periódica. Rio de Janeiro: Relume- Dumará, 1994 (adaptado). O alerta dado por Levi justifica-se porque a a) diluição do ácido libera muito calor. b) mistura de água e ácido é explosiva. c) água provoca a neutralização do ácido. d) mistura final de água e ácido separa-se em fases. e) água inibe a liberação dos vapores provenientes do ácido. SEÇÃO ITA / IME 117. (ITA 1986 – Q16) Chamando de H1 a entalpia da mistura de 1 mol de C(diam.) + 1 mol de O2(g); chamando de H2 a entalpia da mistura de 1 mol de C(graf.) + 1 mol de O2(g); chamando de H3 a entalpia de 1 mol de CO2(g) e sabendo que 110 APOSTILA 01 DE FÍSICO-QUÍMICA – PROF. PEDRO MADEIRA (2022) (H3–H1)25oC = – 94,50 kcal e (H3–H2)25oC = – 94,05 kcal, são feitas as seguintes afirmações: I. A queima de 1 quilate de diamante libera mais calor do que a de 1 quilate de grafite. II. É impossível determinar os valores absolutos de H1, H2 e H3, embora seja fácil determinar os valores dos DH. III. Para a transformação 2 C(graf.) à 2 C(diam.) podemos concluir que DH = – 0,90 kcal. IV. Admitindo que o calor específico médio da água líquida seja 1 cal/(g.oC), o calor necessário para aquecer 31,4 kg de água da temperatura ambiente (25oC) até a ebulição (P = 1 atm) é praticamente igual ao liberado na queima de 3,0 g de C(graf.). V. Na presença de excesso de oxigênio, a queima do diamante resulta no mesmo produto que a queima do grafite. Das afirmações feitas é(são) FALSA(S) apenas: A ( ) I, II e V B ( ) I e III C ( ) II e V D ( ) III e IV E ( ) IV PERGUNTA. Qual o sentido físico de (H1–H2)? 118. (ITA 1989 – Q24) Esta questão se refere à comparação do efeito térmico verificado ao se misturarem 100 cm3 de solução aquosa 0,10 molar de cada um dos ácidos abaixo com 100 cm3 de solução aquosa 0,10 molar de cada uma das bases abaixo. A tabela a seguir serve para deixar claro a notação empregada para designar os caloresdesprendidos. ácido base HCl HNO3 ácido acético NaOH | DH11 | | DH12 | | DH13 | KOH | DH21 | | DH22 | | DH23 | NH4OH | DH31 | | DH32 | | DH33 | Lembrando que o processo de dissociação de eletrólitos fracos é endotérmico, é CORRETO esperar que: A ( ) | DH33 | seja o maior dos | DH | citados. B ( ) | DH11 | = | DH13 | C ( ) | DH23 | = | DH33 | D ( ) | DH31 | = | DH32 | E ( ) | DH21 | > | DH22 | 119. (ITA 1992 – Q05) Nitrato de amônio pode explodir porque a sua decomposição é exotérmica. Qual das opções a seguir contém a equação química, envolvendo este composto que representa a reação mais exotérmica? A ( ) NH4NO3(c) à 2 N(g) + 4 H(g) + 3 O(g) B ( ) NH4NO3(c) à NH3(g) + HNO3(g) C ( ) NH4NO3(c) à N2(g) + H2O(g) + H2O2(g) D ( ) NH4NO3(c) à N2(g) + 2 H2O(g) + 1/2 O2(g) E ( ) NH4NO3(c) à N2(g) + 2 H2(g) + 3/2 O2(g) 120. (ITA 1992 – Q11) Assinale qual das reações a seguir é a mais endoenergética: A ( ) B2(g) à 2B(g) B ( ) C2(g) à 2C(g) C ( ) N2(g) à 2N(g) D ( ) O2(g) à 2O(g) E ( ) F2(g) à 2F(g) Prof. Pedro Madeira 121. (ITA 1992 – P11) Relacione a energia de ligação entre átomos de uma molécula biatômica com comprimento de onda da luz capaz de provocar a sua clivagem homolítica. 122. (ITA 1995 – Q28) Sob 1 atm e a 25°C, qual das reações abaixo equacionadas deve ser a mais exotérmica? A ( ) H2(g) + F2(g) ® 2 HF(g) B ( ) H2(g) + Cl2(g) ® 2 HCl(g) C ( ) H2(g) + I2(g) ® 2 HI(g) D ( ) Br2(g) + I2(g) ® 2 BrI(g) E ( ) Cl2(g) + Br2(g) ® 2 ClBr(g) 123. (ITA 1996 – Q09) Considere as duas amostras seguintes, ambas puras e a 25°C e 1 atm: P à 1 litro de propano (g) B à 1 litro de butano (g) Em relação a estas duas amostras são feitas as afirmações seguintes: I. P é menos densa que B. II. A massa de carbono em B é maior que em P. III. O volume de oxigênio consumido na queima completa de B é maior que aquele consumido na queima completa de P. IV. O calor liberado na queima completa de B é maior que aquele liberado na queima completa de P. V. B contém um número total de átomos maior que P. VI. B e P são mais densas que o ar na mesma pressão e temperatura. Das afirmações acima são CORRETAS: A ( ) Todas B ( ) Nenhuma C ( ) Apenas I, II e III D ( ) Apenas I, III e V E ( ) Apenas II, IV e VI 124. (ITA 1996 – Q30) Considere as informações contidas nas seguintes equações termoquímicas, todas referentes à temperatura de 25°C e pressão de uma atmosfera: 1. H2O(ℓ) à H2O(g); DH1 = 44,0 kJ / mol. 2. CH3CH2OH(ℓ) à CH3CH2OH(g); DH2 = 42,6 kJ / mol. 3. CH3CH2OH(ℓ) + 7/2 O2(g) à 2 CO2(g) + 3 H2O(l); DH3 = – 1366,8 kJ / mol. 4. CH3CH2OH(ℓ) + 7/2 O2(g) à 2 CO2(g) + 3 H2O(g); DH4 = ? 5. CH3CH2OH(g) + 7/2 O2(g) à 2 CO2(g) + 3 H2O(l); DH5 = ? 111 APOSTILA 01 DE FÍSICO-QUÍMICA – PROF. PEDRO MADEIRA (2022) 6. CH3CH2OH(g) + 7/2 O2(g) à 2 CO2(g) + 3 H2O(g); DH6 = ? Em relação ao exposto acima, é ERRADO afirmar que: A ( ) As reações representadas pelas equações 1 e 2 são endotérmicas. B ( ) As reações representadas pelas equações 3, 4, 5 e 6 são exotérmicas. C ( ) DH4 = – 1234,8 kJ / mol. D ( ) DH5 = – 1324,2 kJ / mol. E ( ) DH6 = – 1277,4 kJ / mol. 125. (ITA 1997 – Q16) Considere as afirmações sobre os óxidos de nitrogênio NO, N2O e NO2: I. A formação destes óxidos, a partir de N2 e O2, é endotérmica. II. Os números de oxidação dos átomos de nitrogênio nos óxidos NO, N2O e NO2 são, respectivamente, +2, +1, e +4. III. O N2O é chamado de gás hilariante. IV. O NO é o anidrido do ácido nítrico. V. O NO2 é um gás colorido. Estão CORRETAS: A ( ) Apenas II, IV. B ( ) Apenas III e V. C ( ) Apenas I, II, III e V. D ( ) Apenas I, II, IV e V. E ( ) Todas. 126. (ITA 1998 – Q18) Considere os valores das seguintes variações de entalpia (DH) para as reações químicas representadas pelas equações I e II, onde (graf) significa grafite. I. C(graf) + O2(g) à CO2(g); DH (298 K; 1 atm) = – 393 kJ II. CO(g) + 1/2 O2(g) à CO2(g); DH (298 K; 1 atm) = – 283 kJ Com base nestas informações e considerando que todos DH se referem à temperatura e pressão citadas acima, assinale a opção CORRETA: A ( ) C(graf) + 1/2 O2(g) à CO(g); DH = + 110 kJ B ( ) 2 C(graf) + O2(g) à 2CO(g); DH = – 110 kJ C ( ) 2 C(graf) + 1/2 O2(g) à C(graf) + CO(g); DH = + 110 kJ D ( ) 2 C(graf) + 2 O2(g) à 2 CO(g) + O2(g); DH = + 220 kJ E ( ) C(graf) + O2(g) à CO(g) + 1/2 O2(g); DH = – 110 kJ Prof. Pedro Madeira 127. (ITA 2000 – Q17) Na temperatura e pressão ambientes, a quantidade de calor liberada na combustão completa de 1,00 g de etanol (C2H5OH) é igual a 30 J. A combustão completa de igual massa de glicose (C6H12O6) libera 15 J. Com base nestas informações é CORRETO afirmar que: A ( ) a quantidade de calor liberada na queima de 1,00 mol de etanol é igual a 2 vezes a quantidade de calor liberada na queima de 1,00 mol de glicose. B ( ) a quantidade de oxigênio necessária para queimar completamente 1,00 mol de etanol é igual a 2 vezes aquela necessária para queimar a mesma quantidade de glicose. C ( ) a relação combustível / comburente para a queima completa de 1,00 mol de etanol é igual a 1/2 da mesma relação para a queima completa de 1,00 mol de glicose. D ( ) a quantidade de calor liberada na queima de etanol será igual àquela liberada na queima de glicose quando a relação massa de etanol / massa de glicose queimada for igual a 1/2. E ( ) a quantidade de calor liberada na queima de etanol será igual àquela liberada na queima de glicose quando a relação mol de etanol / mol de glicose for igual a 1/2. 128. (ITA 2001 – Q11) A figura abaixo mostra como a entalpia dos reagentes e dos produtos de uma reação química do tipo A(g) + B(g) ® C(g) varia com a temperatura. Levando em consideração as informações fornecidas nesta figura, e sabendo que a variação de entalpia (DH) é igual ao calor trocado pelo sistema à pressão constante, é ERRADO afirmar que A ( ) na temperatura T1 a reação ocorre com liberação de calor. B ( ) na temperatura T1 a capacidade calorífica dos reagentes é maior que a dos produtos. C ( ) no intervalo de temperatura compreendido entre T1 e T2, a reação ocorre com absorção de calor (DH > zero). D ( ) o DH, em módulo, da reação aumenta com o aumento de temperatura. E ( ) tanto a capacidade calorífica dos reagentes como a dos produtos aumenta com o aumento da temperatura. 129. (ITA 2001 – Q29) A 25ºC e pressão de 1 atm, a queima completa de um mol de n-hexano produz dióxido de carbono e água no estado gasoso e libera 3883 kJ, enquanto que a queima completa da mesma quantidade de n-heptano produz as mesmas substâncias no estado gasoso e libera 4498 kJ. A) Escreva as equações químicas, balanceadas, para as reações de combustão em questão. B) Utilizando as informações fornecidas no enunciado desta questão, faça uma estimativa do valor do calor de combustão do n-decano. Deixe claro o raciocínio utilizando na estimativa realizada. C) Caso a água formada na reação de combustão estivesse no estado líquido, a quantidade de calor liberado seria MAIOR, MENOR OU IGUAL a 3383 kJ? Por quê? 112 APOSTILA 01 DE FÍSICO-QUÍMICA – PROF. PEDRO MADEIRA (2022) 130. (ITA 2002 – Q05) A figura abaixo mostra como a capacidade calorífica, CP, de uma substância varia com a temperatura, sob pressão constante. Considerando as informações mostradas na firgura acima, é ERRADO afirmar que A ( ) a substância em questão, no estado sólido, apresenta mais de uma estrutura cristalina diferente. B ( ) a capacidade calorífica da substância no estado gasoso é menor do que aquela no estado líquido. C ( ) quer esteja a substância no estado sólido, líquido ou gasoso, sua capacidade calorífica aumenta com o aumento da temperatura. D ( ) caso a substãncia se mantenha no estado líquidoem temperaturas inferiores a Tf, a capacidade calorífica da substância líquida é maior do que a capacidade calorífica da substância na fase sólida estável em temperaturas menores do que Tf. E ( ) a variação de entalpia de uma reação envolvendo a substância em questão no estado líquido aumenta com o aumento da temperatura. Prof. Pedro Madeira 131. (ITA 2004 – Q01) Qual das opções a seguir apresenta a equação química balanceada para a reação de formação de óxido de ferro (II) sólido nas condições-padrão? A ( ) Fe(s) + Fe2O3(s) → 3FeO(s). B ( ) Fe(s) + 1/2 O2(g) → FeO(s). C ( ) Fe2O3(s) → 2FeO(s) + 1/2 O2(g). D ( ) Fe(s) + CO(g) → FeO(s) + C(graf). E ( ) Fe(s) + CO2(g) → FeO(s) + C(graf) + 1/2 O2(g). 132. (ITA 2004 – Q26) O gráfico abaixo mostra a variação, com o tempo, da velocidade de troca de calor durante uma reação química. Admita que 1 moI de produto tenha se formado desde o início da reação até o tempo t = 11 min. Utilizando as informações contidas no gráfico, determine, de forma aproximada, o valor das quantidades abaixo, mostrando os cálculos realizados. a) Quantidade, em mols, de produto formado até t = 4 min. b) Quantidade de calor, em kJ mol–1, liberada na reação até t = 11 min. 133. (ITA 2005 – Q08) Assinale a opção que contém a substância cuja combustão, nas condições-padrão, libera maior quantidade de energia. A ( ) Benzeno B ( ) Ciclohexano C ( ) Ciclohexanona D ( ) Ciclohexeno E ( ) n-Hexano 134. (ITA 2005 – Q21) Qualitativamente (sem fazer contas), como você explica o fato de a quantidade de calor trocado na vaporização de um mol de água no estado líquido ser muito maior do que o calor trocado na fusão da mesma quantidade de água no estado sólido? 135. (ITA 2006 – Q05) Considere as seguintes afirmações a respeito da variação, em módulo, da entalpia (ΔH) e da energia interna (ΔU) das reações químicas, respectivamente representadas pelas equações químicas abaixo, cada uma mantida a temperatura e pressão constantes: I. H2O(g) + 1/2 O2(g) → H2O2(g); |DHI| > |DUI| II. 4 NH3(g) + N2(g) → 3 N2H4(g); |ΔHII| < |ΔUII| III. H2(g) + F2(g) → 2 HF(g); |ΔHIII| > |ΔUIII| IV. HCl(g) + 2 O2(g) → HClO4; |DHIV| < |DUIV| V. CaO(s) + 3 C(s) → CO(g) + CaC2(s); |DHV| > |DUV| Das afirmações acima, estão CORRETAS A ( ) apenas I, II e V B ( ) apenas I, III e IV C ( ) apenas II, IV e V D ( ) apenas III e V E ( ) todas 136. (ITA 2006 – Q25) Uma substância A apresenta as seguintes propriedades: Temperatura de fusão a 1 atm = − 20oC Temperatura de ebulição a 1 atm = 85oC Variação de entalpia de fusão = 180 J g−1 Variação de entalpia de vaporização = 500 J g−1 Calor específico de A(s) = 1,0 J g−1 oC−1 Calor específico de A(ℓ) = 2,5 J g−1 oC−1 Calor específico de A(g) = 0,5 J g−1 oC−1 À pressão de 1 atm, uma amostra sólida de 25 g da substância A é aquecida de −40oC até 100oC, a uma velocidade constante de 450 J min−1. Considere que todo calor fornecido é absorvido pela amostra. Construa o gráfico de temperatura (oC) versus tempo (min) para todo o processo de aquecimento considerado, indicando claramente as coordenadas dos pontos iniciais e finais de cada etapa do processo. Mostre os cálculos necessários. Prof. Pedro Madeira 113 APOSTILA 01 DE FÍSICO-QUÍMICA – PROF. PEDRO MADEIRA (2022) 137. (ITA 2007 – Q14) Assinale a opção que indica a variação CORRETA de entalpia, em kJ/mol, da reação química a 298,15 K e 1 bar, representada pela seguinte equação: C4H10(g) à C4H8(g) + H2(g). Dados eventualmente necessários: ∆Hf o`C4H8(g)a= − 11,4; ∆Hf o`CO2(g)a= − 393,5; ∆Hf o`H2O(l)a= − 285,8; ∆Hc o`C4H10(g)a= − 2.877,6, em que ∆Hf o e ∆Hc o, em kJ/mol, representam as variações de entalpia de formação e de combustão a 298,15 K e 1 bar, respectivamente. A ( ) – 3.568,3 B ( ) – 2.186,9 C ( ) + 2.186,9 D ( ) + 125,4 E ( ) + 114,0 138. (ITA 2008 – Q19) Assinale a opção ERRADA que apresenta (em kJ/mol) a entalpia padrão de formação (ΔHf) da substância a 25oC. A ( ) ΔHf (H2(g)) = 0 B ( ) ΔHf (F2(g)) = 0 C ( ) ΔHf (N2(g)) = 0 D ( ) ΔHf (Br2(g)) = 0 E ( ) ΔHf (Cl2(g)) = 0 139. (ITA 2009 – Q26) São dadas as seguintes informações: I. O polietileno é estável até aproximadamente 340oC. Acima de 350oC ele entra em combustão. II. Para reduzir ou retardar a propagação de chama em casos de incêndio, são adicionados retardantes de chama à formulação dos polímeros. III. O Al(OH)3 pode ser usado como retardante de chama. A aproximadamente 220oC, ele se decompõe, segundo a reação 2 Al(OH)3(s) à Al2O3(s) + H2O(g), cuja variação de entalpia (ΔH) envolvida é igual a 1170 J g–1. IV. Os três requisitos de combustão de um polímero são: calor de combustão, combustível e oxigênio. Os retardantes de chama interferem no fornecimento de um ou mais desses requisitos. Se Al(OH)3 for adicionado a polietileno, cite um dos requisitos de combustão que será influenciado por cada um dos parâmetros abaixo quando a temperatura próxima ao polietileno atingir 350oC. Justifique resumidamente sua resposta. a) Formação de Al2O3(s) b) Formação de H2O(g) c) ΔH de decomposição do Al(OH)3 140. (ITA 2010 – Q16) Sabe-se que a 25°C as entalpias de combustão (em kJ mol−1) de grafita, gás hidrogênio e gás metano são, respectivamente: –393,5; –285,9 e –890,5. Assinale a alternativa que apresenta o valor CORRETO da entalpia da seguinte reação: C(grafita) + 2 H2(g) à CH4(g) A ( ) –211,1 kJ mol–1 B ( ) –74,8 kJ mol–1 C ( ) 74,8 kJ mol–1 D ( ) 136,3 kJ mol–1 E ( ) 211,1 kJ mol–1 141. (ITA 2011 – Q05) Considere a energia liberada em I. combustão completa (estequiométrica) do octano e em II. célula de combustível de hidrogênio e oxigênio. Assinale a opção que apresenta a razão CORRETA entre a quantidade de energia liberada por átomo de hidrogênio na combustão do octano e na célula de combustível. Dados: Energia de ligação, em kJ.mol–1: C–C 347 H–H 436 C–H 413 H–O 464 C=O 803 O=O 498 A ( ) 0,280 B ( ) 1,18 C ( ) 2,35 D ( ) 10,5 E ( ) 21,0 142. (ITA 2012 – Q03) A reação de sulfonação do naftaleno ocorre por substituição eletrofílica nas posições a e b do composto orgânico, de acordo com o diagrama de coordenada de reação a 50ºC. Com base neste diagrama, são feitas as seguintes afirmações: I. A reação de sulfonação do naftaleno é endotérmica. II. A posição a do naftaleno é mais reativa do que a de b. III. O isômero b é mais estável que o isômero a. Das afirmações acima, está(ão) CORRETA(S) apenas A ( ) I B ( ) I e II C ( ) II D ( ) II e III E ( ) III 143. (ITA 2012 – Q26) Considere a reação de combustão do composto X, de massa molar igual a 27,7 g.mol–1, representada pela seguinte equação química balanceada: X(g) + 3 O2(g) à Y(s) + 3 H2O(g); DHo c= –2035 kJ.mol–1 Calcule o valor numérico, em kJ, da quantidade de calor liberado na combustão de: a) 1,0 x 103 g de X b) 1,0 x 102 mol de X c) 2,6 x 1022 moléculas de X d) uma mistura de 10,0 g de X e 10,0 g de O2 144. (ITA 2013 – Q09) 100 gramas de água líquida foram aquecidos utilizando o calor liberado na combustão completa de 0,25 gramas de etanol. Sabendo que a variação da temperatura da água foi de 12,5ºC, assinale a alternativa que apresenta o valor CORRETO para a entalpia molar de combustão do etanol. Considere que a capacidade calorífica da água é igual a 4,18 kJ.kg–1.oC–1 e que a energia 114 APOSTILA 01 DE FÍSICO-QUÍMICA – PROF. PEDRO MADEIRA (2022) liberada na combustão do etanol foi utilizada exclusivamente no aquecimento da água. A ( ) – 961 kJ B ( ) – 5,2 kJ C ( ) + 4,2 kJ D ( ) + 5,2 kJ E ( ) + 961 kJ 145. (ITA 2015 – Q10) Para determinar a entalpia de vaporização do composto hipotético MX4(ℓ), o mesmo foi colocado num recipiente equipado com uma serpentina de aquecimento resistivo,a 80ºC e sob pressão de 1,0 bar. Para a manutenção da temperatura, foi utilizada uma fonte de 30 V com passagem de corrente de 900 mA durante 30s, tendo sido vaporizados 2,0g de MX4(ℓ). Sabendo que a massa molar desse composto é 200 g mol–1, assinale a opção que apresenta a entalpia molar de vaporização em kJ mol–1, a 80ºC. A ( ) 4,1 B ( ) 8,1 C ( ) 81 D ( ) 405 E ( ) 810 146. (ITA 2016 – Q13) Considere as entalpias padrão de formação dos seguintes compostos: CH4(g) O2(g) CO2(g) H2O(g) DHfo/kJ.mol–1 –74,81 zero –393,51 –285,83 Sabendo que a capacidade calorífica da água, à pressão constante, vale 75,9 J.mol–1 e que sua entalpia de vaporização é igual a 40,66 kJ.mol–1, assinale a alternativa que melhor corresponda ao número de mols de metano necessários para vaporizar 1 L de água pura, cuja temperatura inicial é 25ºC, ao nível do mar. A ( ) 1,0 B ( ) 2,0 C ( ) 2,9 D ( ) 3,8 E ( ) 4,7 Prof. Pedro Madeira 147. (ITA 2016 – Q24) Dadas as informações: I. O poder calorífico de um combustível representa a quantidade de calor gerada na combustão por unidade de massa. II. O poder calorífico do H2(g) é aproximadamente 3 vezes o da gasolina. III. O calor latente de ebulição do H2(ℓ) é desprezível frente ao poder calorífico do H2(g). IV. A massa específica do H2(ℓ) é de 0,071 g.cm–3 e a da gasolina é de 0,740 g.cm–3. Com base nestas informações, determine o valor numérico: a) da massa de 45 L de gasolina. b) do volume de H2(ℓ) que, ao sofrer combustão, fornece a mesma quantidade de calor liberada na combustão de 45 L de gasolina. c) do volume que o H2 ocuparia se estivesse na forma de gás, à pressão de 1 bar e a 25ºC. 148. (ITA 2017 – Q27) Considere que a radiação de comprimento de onda igual a 427 nm seja usada no processo de fotossíntese para a produção de glicose. Suponha que esta radiação seja a única fonte de energia para este processo. Considere também que o valor da variação de entalpia padrão da reação de produção de glicose, a 25ºC, seja igual a +2808 kJ·mol–1. a) Escreva a equação que representa a reação química de produção de um mol de glicose pelo processo de fotossíntese. b) Calcule a variação de entalpia envolvida na produção de uma molécula de glicose, via fotossíntese, a 25oC. c) Calcule a energia de um fóton de radiação com comprimento de onda de 427 nm. d) Quantos destes fótons (427 nm), no mínimo, são necessários para produzir uma molécula de glicose? 149. (ITA 2018 – Q19) O perclorato de amônio (PA) é um dos componentes mais utilizados em propelentes de foguetes. Para aperfeiçoar seu desempenho, hidrogênio pode ser utilizado como aditivo. Considere dadas as entalpias de combustão destas espécies: ΔHc,PA = – 189 kJ mol–1; ΔHc,H2 = – 286 kJ mol–1. Com base nessas informações, assinale a opção que apresenta a equação linear da variação de entalpia de combustão da mistura de PA com H2 em função da quantidade de H2. A ( ) y = –0,48x + 189 B ( ) y = –0,48x – 189 C ( ) y = –0,48x + 208 D ( ) y = –0,97x – 189 E ( ) y = –0,97x – 208 150. (ITA 2019 – Q01) Considere reações de combustão do etanol. a) Escreva a equação química balanceada para a reação com oxigênio puro. b) Escreva a equação química balanceada para a reação com ar atmosférico. c) Escreva a equação química balanceada para a reação com 50% da quantidade estequiométrica de ar atmosférico. d) Classifique as reações dos itens a), b) e c) em ordem crescente de variação de entalpia reacional. 151. (ITA 2019 – Q02) Uma determinada quantidade de um composto A foi misturada a uma quantidade molar três vezes maior de um composto B, ou seja, A + 3B. Essa mistura foi submetida a dois experimentos de combustão (I e II) separadamente, observando-se: I. A combustão dessa mistura A + 3B liberou 550 kJ de energia. II. A combustão dessa mistura A + 3B, adicionada de um composto C em quantidade correspondente a 25% em mol do total da nova mistura, liberou 814 kJ de energia. Considerando que os compostos A, B e C não reagem entre si, determine os valores numéricos a) da quantidade, em mol, de A, B e C. 115 APOSTILA 01 DE FÍSICO-QUÍMICA – PROF. PEDRO MADEIRA (2022) b) do calor de combustão, em kJ mol−1, do composto C, ΔHc(C). Dados: ΔHc(A) = −700 kJmol−1; ΔHc(B) = −500 kJ mol−1. 152. (ITA 2019 – Q08) Considere as variações de entalpia de processo abaixo tabeladas. Processo ΔH (kJ.mol–1) Ionização do Nao 495,8 Energia de ligação Cl–Cl 242,6 Entalpia de vaporização do Nao 97,4 Afinidade eletrônica do Cl -349 Entalpia de rede do NaCl -787 a) Esboce o diagrama de Born-Haber para a formação do NaCl(s) a partir de Na0(s) e Cl2(g) e calcule a variação de entalpia de formação do NaCl(s). b) Sabe-se que o valor absoluto (em módulo) da entalpia de rede do CaO(s) é maior do que a do NaCl(s). Explique por quê. 153. (ITA 2020 – Q02) Os biodigestores possibilitam o reaproveitamento de detritos convertendo material orgânico em metano, que é utilizado como combustível em sistemas de geração de energia. Um laticínio utiliza a queima do metano para aquecer 1 m3/h de água, de 25ºC a 100ºC em uma caldeira que opera a 1 atm. Sabendo-se que 25% do calor produzido no processo é perdido e que, nessas condições, a combustão completa do metano produz água líquida, determine a) a entalpia molar da combustão do metano; b) a taxa de calor necessária para aquecer a água; c) a vazão de metano, em kg/h, que deve alimentar a caldeira. Dados: DHf o(CH4(g)) = –17,9 kcal mol–1; DHf o(CO2(g)) = –94,1 kcal mol–1; DHf o(H2O(g)) = –57,9 kcal mol–1; DHeb o(H2O(ℓ)) = –10,5 kcal mol–1; CP o(H2O(ℓ)) = 1 cal g–1 oC–1; r(H2O(l)) = 1 g cm–3 154. (ITA 2021 – Q04) O poder calorífico é um indicativo do potencial energético dos combustíveis, sendo que a diferença entre o poder calorífico superior (PCS) e o poder calorífico inferior (PCI) equivale à energia necessária A vaporização da água formada numa reação de combustão completa. Sabe-se que o PCS do metano é 55 MJ.kg–1 e do etanol é 30 MJ.kg–1 e que a entalpia de vaporização da água é ∆Hvap,H2O = 44 kJ.mol–1. a) Calcule os valores do PCI do metano e do etanol, em kJ.mol–1. b) Sabendo que o gás natural é composto principalmente por metano e que os outros componentes possuem PCS muito inferiores ao deste gás, estime a porcentagem em massa de metano presente em um gás natural cujo PCS = 52 MJ.kg–1. c) Explique por que o PCS do metano é muito superior ao do etanol. 155. (ITA 2022 – Q57) Sobre a energia reticular, assinale alternativa ERRADA. A ( ) A energia reticular é a energia envolvida na formação do sólido cristalino quando o cristal sólido se forma a partir de íons separados no estado gasoso. B ( ) O ciclo de Born-Haber é um método usado para determinação do valor da energia reticular. C ( ) A energia reticular é função das cargas dos íons e da distância entre eles. D ( ) Um sólido que sofre mudança de estrutura cristalina mantém constante sua energia reticular. E ( ) A energia reticular representa a soma entre as forças de atração e repulsão eletrostática. 156. (ITA 2022 – Q62) Sistemas compostos por água e tensoativos em diferentes proporções, depois de homogeneizados, passam por um processo termodinâmico quando atingem temperaturas em torno de 0°C. A variação de entalpia (DH) desse processo foi determinada para cada mistura em função da composição do sistema, conforme apresentado no gráfico. Considere que o ponto de fusão do tensoativo puro é menor que -20ºC e o calor latente de fusão da água pura é 334 J.g–1. Sobre esses sistemas são feitas as seguintes afirmações: I. O DH refere-se à transição de fase do tensoativo. II. O calor latente de fusão do tensoativo puro é -180 J.g–1. III. Até 35% em massa de água pode se apresentar na forma associada à substância e não funde.IV. O DH é proporcional à quantidade de água não associada ao tensoativo. Com base no gráfico e nas informações do enunciado, assinale a opção que indica a(s) afirmação(ões) CORRETA(S). A ( ) Apenas I e II B ( ) Apenas I, II e IV C ( ) Apenas II D ( ) Apenas III e IV E ( ) Todas 116 APOSTILA 01 DE FÍSICO-QUÍMICA – PROF. PEDRO MADEIRA (2022) 157. (ITA 2022 – Q07) – RESOLVA AGORA Considere as seguintes informações: I. Primeira energia de ionização do cálcio: 590 kJ.mol–1 II. Segunda energia de ionização do cálcio: 1145 kJ.mol–1 III. Afinidade eletrônica do cloro: -340 kJ.mol–1 IV. Entalpia de solubilização do cloreto de cálcio: -81 kJ.mol–1 V. Entalpia de hidratação do íon de cálcio: -1579 kJ.mol–1 V. Entalpia de hidratação do íon de cloro: -378 kJ.mol–1 Com base nessas informações, responda os itens abaixo. a) Represente, na forma de equações químicas, as informações acima (I-VI). b) Equacione a reação de entalpia de rede do cloreto de cálcio a partir das equações I-VI, conforme a necessidade. c) Calcule o valor numérico da entalpia de rede do cloreto de cálcio (em kJ.mol–1). 158. (IME 1997 – Q03) Uma mistura de metano e etileno foi queimada em um recipiente, com volume constante de 3,0 litros, em presença de excesso de oxigênio, saturado em vapor d'água, de forma a que fosse obtida a combustão completa e para garantir que a água formada ficasse no estado líquido. A combustão foi realizada a 25oC, liberando 242,7 kcal, registrando-se uma redução na pressão de 16,3 atm. Determine o número de moles de metano e etileno presentes na mistura inicial. I) Entalpias de Formação (Ho f ) H2O (líquida) = - 68,3 kcal / mol CO2 (gasoso) = - 94,1 kcal / mol CH4 (gasoso) = - 17,9 kcal / mol C2H4 (gasoso) = + 12,5 kcal / mol 159. (IME 1998 – Q08) Metanol pode ser sintetizado diretamente a partir de monóxido de carbono e hidrogênio. Sabendo-se que os calores de combustão do monóxido de carbono e do metanol a 25oC, são respectivamente, –283,12 kJ/mol e –726,87 kJ/mol, calcule o calor de reação na formação de 2,0 g de metanol a 25oC, pela reação de hidrogenação direta do monóxido de carbono. DHf 0 (CO2) = – 393,70 kJ / mol DHf 0 (H2O) = – 281,79 kJ / mol 160. (IME 2000 – Q03) Determine a massa de água que, com uma variação de temperatura de 30oC, fornece energia equivalente ao calor de formação de um mol de sulfeto de carbono sólido. Dados: calor de combustão do sulfeto de carbono = -265 kcal/mol; calor de formação do gás sulfuroso = -71 kcal/mol; calor de formação do dióxido de carbono = -96 kcal/mol; capacidade calorífica da água líquida = 1,0 cal/g; peso molecular da água = 18. 161. (IME 2008 – Q03) Nas combustões completas de x gramas de acetileno e de y gramas de benzeno são liberadas, respectivamente, Q1 kcal e Q2 kcal. Determine o calor liberado, em kcal, na formação de z gramas de benzeno a partir do acetileno. 162. (IME 2010 – Q08) Uma dada massa de óxido ferroso é aquecida a 1273 K e, em seguida, exposta a uma mistura gasosa de monóxido de carbono e hidrogênio. Desta forma, o óxido é reduzido a metal sem qualquer fornecimento adicional de energia. Admita que ocorra uma perda de calor para as circunvizinhanças de 4,2 kJ/mol de óxido reduzido. Calcule a razão mínima entre as pressões parciais de monóxido de carbono e de hidrogênio (pCO/pH2) na mistura gasosa inicial, de modo que o processo seja auto-sustentável. Despreze a decomposição da água. Calores de reação a 1273 K (kJ/mol) redução do óxido ferroso 265 oxidação do hidrogênio – 250 oxidação do monóxido de carbono – 282 163. (IME 2012 – Q32) Sobre a diferença entre sólido amorfo e sólido cristalino, pode-se afirmar o seguinte: (A) os sólidos amorfos não têm uma entalpia de fusão definida, enquanto os sólidos cristalinos têm. (B) sólido amorfo é aquele que pode sofrer sublimação, enquanto sólido cristalino não. (C) embora ambos possuam estrutura microscópica ordenada, os sólidos amorfos não possuem forma macroscópica definida. (D) os sólidos cristalinos têm como unidade formadora átomos, enquanto para os amorfos a unidade formadora são moléculas. (E) os sólidos cristalinos são sempre puros, enquanto os amorfos são sempre impuros. 164. (IME 2012 – Q09) Em função do calor de formação do dióxido de carbono (ΔH°f,CO2); do calor de formação do vapor d’água (ΔH°f,H2O(g)); e do calor da combustão completa de uma mistura de metano e oxigênio, em proporção estequiométrica (ΔHr), deduza a expressão do calor de formação do metano (ΔH°f,CH4). Prof. Pedro Madeira 165. (IME 2014 – Q05) 1,00kg de carbonato de cálcio, na temperatura de 298K, é introduzido em um forno que opera a 101kPa. O forno é então aquecido até a temperatura Tc na qual ocorrerá a calcinação do carbonato de cálcio. Sabendo-se que o módulo da variação da energia livre de Gibbs da reação de calcinação à temperatura Tc é igual a 10,7kJ/mol, determine a temperatura de calcinação Tc e a quantidade de calor necessária à completa calcinação do carbonato. Despreze os efeitos de mistura e 117 APOSTILA 01 DE FÍSICO-QUÍMICA – PROF. PEDRO MADEIRA (2022) considere que, para o sistema reacional, aplicam-se as seguintes equações: Dados: Entalpias e entropias de formação a 298 K e capacidades caloríficas médias: Substância DHfo (kJ/mol) DSfo (J/mol.K) CP (J/mol.K) CO2(g) –394 213 54,0 CaO(s) –636 39,0 56,0 CaCO3(s) –1207 94,0 110 166. (IME 2015 – Q10) Monóxido de carbono a 473 K é queimado, sob pressão atmosférica, com 90% em excesso de ar seco, em base molar, a 773 K. Os produtos da combustão abandonam a câmara de reação a 1273 K. Admita combustão completa e considere que 1 mol de ar é constituído por 0,20 mol de oxigênio e 0,80 mol de nitrogênio. Calcule a quantidade de energia, em kJ, que é liberada no decorrer da reação, por mol de monóxido de carbono queimado. Considere que os gases apresentam comportamento ideal. DADOS: Calor de combustão do monóxido de carbono (a 298 K e 1 atm) = –283 kJ·mol–1 T(K) = t(oC) + 273 Substância CO CO2 O2 N2 Cp médio (kJ/mol.K) 0,03 0,04 0,03 0,03 167. (IME 2017 – Q01) O oxigênio e o hidrogênio combinam-se, em células de combustível, produzindo água líquida e gerando corrente elétrica. O máximo trabalho elétrico útil que essas células produzem é dado por DGo = –237 x 103 J.mol–1. Com base nos dados fornecidos, calcule o ponto de ebulição da água. Aproxime DH por DHo e DS por DSo. Dados termodinâmicos: O2(g) H2(g) H2O(l) H2O(g) So = 206 J/mol.K So = 131 J/mol.K So = 70,0 J/mol.K So = 189 J/mol.K DHo f = –242x103 J/mol 168. (IME 2018 – Q03) A reforma com vapor d`água, a temperaturas altas, é um método industrial para produção de hidrogênio a partir de metano . Calcule a entalpia de reação desse processo. Dados: i) Entalpias de combustão: C(grafite) ... ΔHo = -394 kJ/mol H2(g) ... ΔHo = -286 kJ/mol (forma água líquida) CH4(g) ... ΔHo = -890 kJ/mol (forma água líquida) ii) CO(g) + H2(g) → C(grafite) + H2O (g) ΔHo = -131 kJ/mol Entalpia de fusão do gelo: ΔHfus = 330 kJ·kg–1 Capacidade calorífica específica média da água: CV = 4,2 kJ·kg–1·K–1 169. (IME 2020 – Q37) Uma medida quantitativa da estabilidade de um composto sólido iônico é a sua energia de rede, definida como a energia requerida para decompor completamente 1 mol desse composto nos seus íons em fase gasosa. Considere os seguintes dados: I. a entalpia padrão de formação do CaCl2 é - 790 kJ.mol–1; II. a primeira energia de ionização do átomo de cálcio é 590 kJ.mol–1; III. a segunda energia de ionização do átomo de cálcio é 1146 kJ.mol–1; IV. a vaporização de um mol de Ca(s) consome 190 kJ; V. a energia de ligação do Cl2 é 242 kJ.mol–1; VI. a afinidade eletrônica do Cl é - 349 kJ.mol–1; Com base nessas informações, estima-se que a energia de rededo CaCl2, em kJ.mol–1, seja: (A) 790 (B) 1029 (C) 2070 (D) 2260 (E) 2609 170. (IME 2020 – Q01) Calcule a variação de entalpia (em J) no processo de decomposição de 600 mg de nitroglicerina (C3H5N3O9) que produz nitrogênio, dióxido de carbono e oxigênio gasosos, além de água líquida. Dados: ΔHo f (C3H5N3O9(ℓ)) = − 354 kJ/mol; ΔHo f (H2O(ℓ)) = − 286 kJ/mol; ΔHo f (CO2(g)) = − 394 kJ/mol. 171. (IME 2021 – Q07) O RDX (ciclo-1,3,5-Trimetileno-2,4,6 trinitroamina) e o TNT (2-metil-1,3,5-trinitrobenzeno), quando misturados na proporção percentual 60/40 em massa, formam o “Composto B”. Considerando que cada munição contém 2,5 kg de “Composto B”, inicialmente mantido a 25oC, determine a entalpia padrão teórica esperada na combustão completa de uma munição. DHo Formação H2O = - 286,2 kJ/mol DHo Formação CO2(g) = - 393,5 kJ/mol DHo Formação RDX = + 71 kJ/mol DHo Formação TNT = - 42 kJ/mol o f P o f P G H T S H H c T TS S c T • D = D - D • D = D + D D • D = D + 118 APOSTILA 01 DE FÍSICO-QUÍMICA – PROF. PEDRO MADEIRA (2022) 172. (IME 2022 – Q08) – RESOLVA AGORA Um motor de 6 cilindros e volume total de 5.700 cm3, utilizado em viaturas leves e blindadas, consome 0,5 g do combustível gasoso de composição média C8H18, em cada cilindro, por segundo de operação. Considerações: • o ciclo termodinâmico do motor compreende o funcionamento em 4 tempos: admissão, compra assim combustão e exaustão (escape); • o motor executa 10 ciclos por segundo, ou seja, a mistura de ar e combustível enche o cilindros e depois é comprimida 10 vezes por segundo; • 20,0% da quantidade de combustível sofre combustão incompleta, sendo convertida em CO(g); • 80,0% da quantidade de combustível sofre combustão completa, sendo convertida em CO2(g); • a mistura de ar e combustível comporta-se como gás ideal; • as capacidades caloríficas molares são independentes da temperatura; e • as entalpias de formação a 25ºC. Determine: a) a vazão da entrada de ar no motor, em m3/s; e b) a composição percentual molar dos produtos e a temperatura de combustão, em K. FOLHA DE DADOS • Composição percentual do ar atmosférico = 79,0% de N2(g) e 21,0% de O2(g) Entalpias-padrão de formação a 25ºC: Substância Química C8H18(g) H2O(g) H2O(l) CO2(g) CO(g) DHf o (kJ.mol–1) -208,45 -241,82 -285,83 -393,51 -110,53 Capacidade calorífica molar: Substância Química N2(g) O2(g) H2O(g) CO2(g) CO(g) "#! (J.K–1 .mol–1) 29,13 29,36 33,58 37,11 29,14 QUESTÕES EXTRAS: OLIMPÍADA 173. Uréia, CO(NH2)2, reage com água produzindo dióxido de carbono e amônia. Os dados termodinâmicos para os possíveis reagentes e produtos são dados abaixo (negligencie a solubilidade do dióxido de carbono e da amônia em água líquida). Composto DHf o (kJ/mol) So (J.K –1.mol –1) CO(NH2)2(s) – 333,51 104,60 H2O(l) – 285,83 69,91 H2O(g) – 241,82 188,83 CO2(g) – 393,51 213,74 NH3(g) – 46,11 192,45 a) Considere a hidrólise de uréia com H2O (l) (Reação A) e com H2O (g) (Reação B), respectivamente. Calcule DH°, DS° e DG°, a 25 °C, para cada reação e especifique se a reação é espontânea ou não. b) Considerando que ambos, DH° e DS°, são independentes da temperatura, encontre a temperatura na qual a Reação A ocorrerá espontaneamente. c) Calcule Kp a 25 °C para cada reação, expressando esse valor em unidades apropriadas. R = 8,314 J. K–1.mol–1. 174. As energias-livres de formação do etileno (C2H4) e do etano (C2H6), a 25 oC e 1 atm são, respectivamente, +68,06 kJ/mol e –32,85 kJ/mol. Suas entalpias-padrão de formação são +52,23 kJ/mol e – 84,58 kJ/mol. Pergunta-se: a) Para a redução do etileno a etano com H2, a 25oC e 1 atm, determine a variação da energia livre e a variação da entropia. b) Como se modificaram a energia e a desordem do sistema? c) A reação é espontânea? Justifique. d) Como a temperatura modificaria a espontaneidade da reação? 175. Alguns anos atrás, Texas City (Texas – USA) foi abalada por uma explosão de um depósito de nitrato de amônia, composto muito usado como fertilizante. Este composto, quando aquecido, pode decompor exotermicamente em N2O e água, conforme a equação: NH4NO3(s) à N2O(g) + 2 H2O(g) (equação 1) Se o calor liberado nesta reação ficar aprisionado, altas temperaturas serão atingidas, e assim, o NH4NO3 pode decompor explosivamente em N2, H2O e O2. 2 NH4NO3(s) à 2 N2(g) + 4 H2O(g) + O2(g) (equação 2) Usando as informações fornecidas abaixo, responda: a) Qual o calor liberado (à pressão constante de 1 atm e à temperatura de 25 ºC) na primeira reação ? b) Se 8,00 kg de nitrato de amônio explodem (segunda reação), qual a quantidade de calor liberada (à pressão constante de 1 atm e à temperatura de 25 ºC) ? c) Nitrato de amônia reage com alumínio em pó, produzindo Al2O3, segundo a reação: 2 Al(s) + 3 NH4NO3(s) à 3 N2(g) + 6 H2O(g) + Al2O3(g) Se 8,00 kg de nitrato de amônio são misturados com alumínio em pó, em excesso estequiométrico, qual a quantidade de calor produzida (à pressão constante)? Dados (valores aproximados): ar e combustível motor 6 cilindros produtos trabalho 119 APOSTILA 01 DE FÍSICO-QUÍMICA – PROF. PEDRO MADEIRA (2022) Entalpia de formação: DHfº , a 25 ºC (kJ/mol); Al2O3(g) = -1675,7; H2O(g) = -241,8 NH4NO3(s) = -365,6; N2O(g) = 82,0 176. PARTE I Para cada um dos seguintes processos, indique e justifique o sinal algébrico de DH0, DS0 e DG0: a) A "quebra" da molécula de água líquida em hidrogênio e oxigênio gasosos, processo que requer considerável quantidade de energia. b) A dissolução de pequena quantidade de NH4Cl em água. A solução torna-se bastante fria no processo. c) A explosão de dinamite, uma mistura de nitroglicerina (C3H5N3O9) e terra diatomácea. Essa decomposição explosiva origina produtos gasosos, tais como água, CO2 e outros. Muito calor é liberado neste processo d) A combustão de gasolina no motor de um carro, como exemplificado pela combustão do octano: 2 C8H18(g) + 25 O2(g) à 16 CO2(g) + 18 H2O(g) + calor PARTE II Hidrogênio gasoso pode ser produzido a partir da reação entre carvão e vapor d'água, como mostra a reação: C(s) + H2O(g) à CO(g) + H2(g) a) Calcule o DG0 para esta reação a 25° C, considerando que C(s) é grafite. b) Calcule o kp para esta reação a 25° C. c) Esta reação ocorre espontaneamente sob estas condições? Se não, a que temperatura ela se tornará espontânea? Dados (valores aproximados): H2O(g) à DH0 f = -241,8 kJ/mol, DS0 f = +188,8 J/K.mol e DG0 f = -228,6 kJ/mol (298,15K) CO(g) à DH0 f = -110,5 kJ/mol , DS0 f = +197,7 J/K.mol e DG0 f = -137,2 kJ/mol (298,15K) 177. (OBQ 2001) O álcool etílico líquido tem calor de vaporização igual a 39,3 kJ/mol e as energias das ligações: C–C, C–O, O–H e O–O, em kJ/mol, são, respectivamente: 348; 356; 463 e 493. a) Elabore um esquema (ciclo de Born-Haber) para indicar a obtenção de álcool etílico a partir de seus constituintes. b) Estes dados são suficientes para se calcular o DHf o do álcool etílico? Justifique 178. (OBQ 2005) Quando se queima uma amostra de 0,3212 g de glicose numa bomba calorimétrica, a volume constante, cuja constante calorimétrica é 641 JK–1, a elevação de temperatura é de 7,793 K. Pede-se: a) Calcule a energia interna padrão de combustão. b) Calcule a entalpia padrão de combustão c) Calcule a entalpia padrão de formação da glicose d) Que altura pode subir uma pessoa de 65 kg com a energia da combustão da amostra, admitindo que 25% possam ser convertidos em trabalho? Dados: MGlicose = 180,16 g.mol–1 ΔfHº (CO2(g)) = - 393,51 kJ mol–1 ΔfHº (H2O(l)) = - 285,83 kJ mol–1 Prof. Pedro Madeira 179. (OBQ 2010) Considere os seguintes dados termodinâmicos: Substância ΔfHo 298 (kJ.mol–1) So 298 (J.K–1.mol–1) Cp o 298 (J.K–1.mol– 1) Glicose(s) 1273,2 211,9 218,6 O2(g) 0 204,8 29,3 CO2(g)393,1 213,4 37,1 H2O(l) 285,6 69,8 75,2 H2O(g) 241,6 188,5 33,5 Responda as seguintes questões: a) Calcule o valor da variação de energia livre molar na temperatura de 298 K e 1 atm, para a reação de combustão completa para glicose. b) Calcule a variação de entropia molar para a reação na temperatura de 298 K e 1 atm e justifique com base na reação o valor positivo da mesma. c) Calcule o valor da variação de energia livre molar na temperatura de 400 K e 1 atm, para a reação de combustão completa para glicose. 180. (OBQ 2011) Um experimento calorimétrico foi realizado para a determinação do calor de combustão completa do antraceno. Esta substância é um hidrocarboneto aromático sólido, de massa molar 178,23 g.mol–1, incolor mas que apresenta uma fluorescência azul quando irradiado por uma fonte de radiação ultravioleta. O calorímetro utilizado, uma bomba calorimétrica adiabática, é tem como componentes principais um vaso de reação construído com aço inoxidável com suas paredes espessas e rígidas para suportar uma pressão de oxigênio até 20 atm e um sistema elétrico de ignição das amostras a serem investigadas. Todo experimento calorimétrico realizado no interior deste calorímetro é feito sob o regime de volume constante. A alta pressão do gás oxigênio no vaso de reação é necessária para que seja assegurada a combustão completa das amostras estudadas. A reação de combustão neste equipamento é iniciada ao passar uma corrente elétrica através de um pequeno fio fino de ferro que é mantido em contato com a amostra. O vaso de reação é imerso em um banho de água que é mantido isolado das vizinhanças. O calorímetro utilizado é do tipo adiabático, isto é, todo o calor liberado pela reação permanece no calorímetro; nada é perdido para as vizinhanças. O calor liberado na reação de combustão é absorvido pela massa do banho de água e outras partes do calorímetro. Este processo causa um aumento na temperatura interna no calorímetro. O aumento da temperatura no interior deste calorímetro é medida por um termômetro sensível previamente adaptado ao sistema. 120 APOSTILA 01 DE FÍSICO-QUÍMICA – PROF. PEDRO MADEIRA (2022) Utilizando um calorímetro adiabático como descrito acima, uma amostra de 1,228 g de antraceno foi queimada em um calorímetro com um banho de água de 2 L. a pressão de O2 no início da reação marcava 20 atm. Um fio de ferro de 10,00 cm foi utilizado inicialmente na a ignição do sistema e, após o experimento, 0,65 cm intacto restante deste fio foi coletado. Neste experimento a temperatura medida no termômetro imerso no banho da água utilizada registrou uma variação do ser valor inicial de 23,74ºC até o valor final constante de 30,28ºC. Considerando os valores aproximados para a capacidade térmica específica e densidade da água como c(H2O,l) = 1,0 cal.g–1 grau–1 e ρ(H2O,l) = 1,0 g.ml– 1. A capacidade térmica do calorímetro adiabático utilizado vale Ccal = 881 J grau–1 e o calor de combustão linear por comprimento de fio de ferro vale Qcomb linear (ε) = 0,1 J cm–1. Sabe-se também que 1 cal = 4,184 J e que a constante dos gases ideais tem o valor de R = 8,314 JK–1 mol–1. Utilizando os dados acima, determine a entalpia de combustão completa ΔHcomb do antraceno. 181. (IChO) a) A quantity of 0.10 mol of an ideal gas A initially at 22.2oC is expanded from 0.200 dm3 to 2.42 dm3. Calculate the values of work (w), heat (q), internal energy change (ΔU), entropy change of the system (ΔSsys), entropy change of the surroundings (ΔSsurr), and total entropy change (ΔSuniv) if the process is carried out isothermally and irreversibly against an external pressure of 1.00 atm. b) If 3.00 mol of A is condensed into liquid state and is mixed with 5.00 mol of liquid B, calculate the changes in entropy and Gibbs free energy upon such mixing at 25.0oC. This mixture can be assumed to be ideal. 182. (IChO) Students have made a device capable to operate in a mode that is close to the ideal Brayton cycle. This thermodynamic cycle has once been proposed for development of internal combustion engines. The device consists of a cylinder with 1 mole of helium fitted with a computer-controlled movable piston. A Peltier element which can heat or cool the gas is mounted in the cylinder wall. The device can operate in the following modes: 1) reversible adiabatic expansion or compression, 2) reversible isobaric cooling or heating. Through a number of cooling and compression steps, helium is going from the initial state with the pressure of 1 bar and the temperature of 298 K into the final state with the pressure of 8 bar and the temperature of 298 K. (The total number of cooling and compression stages can be from two up to infinity). 1. What is the minimum work that should be done on the gas for this? Compare this value to the work during a reversible isothermal compression. 2. What is the maximum work that can be done on the gas in this process? 3. Let the process be accomplished in three steps. At each step helium is first cooled and then compressed. At the end of each step the pressure increases twice and the temperature returns to the value of 298 K. What is the total heat removed from the gas by a Peltier element? Once the gas is compressed, it is returned to the initial state (1 bar and 298 K) in two stages (heating and expansion). 4. What is the range of possible values of the formal efficiency η for the resulting cycle? η is the ratio of the useful work done by the gas to the amount of heat given to the gas during the heating stage. 5. In one of the experiments, the gas has been compressed from 1 bar and 298 K to 8 bar and 298 K in several steps (like in question 3). At the end of each step the pressure is increased by x times and the temperature returns to 298 K. Then helium has been returned to the initial state in two stages – heating and expansion. Theoretical value of η for this cycle is 0.379. How many steps were used? In fact, Peltier elements also consume electric energy during the cooling stage. Assume that they consume as much energy as is removed from the gas. 6. What is the maximum possible efficiency of the considered cycle, taking into account energy consumption during cooling? Hint: in reversible adiabatic process for helium . Isochoric molar heat capacity of helium is 3/2R. 183. (IChO) Two rigid containers in thermal equilibrium at 298 K connected by a valve are isolated from the surroundings. In one of the containers, 1.00 mol of He(g) and 0.50 mol of A(g) are present at 1.00 atm. In the other container, 2.00 mol of Ar(g) and 0.50 mol of B2(g) are present at 1.00 atm. a) Predict whether the entropy will increase or decrease when the valve separating the two containers is opened assuming that no chemical reaction takes place. b) Predict whether the entropy will increase or decrease, stating all factors that will have contribution, if a chemical reaction takes place according to the following equation when the valve separating the two containers is opened. A(g) + ½B2(g) → BA(g) ΔH°298 = - 99.0 kJ c) Assuming that all the gases present are ideal, calculate the final pressure at the end of the reaction. The total heat capacity of two containers is 547.0 J/°C. 184. (IChO) Given 10 liters of an ideal gas at 0oC and 10 atm, calculate the final volume and work done, under the 5 3/pV const= 121 APOSTILA 01 DE FÍSICO-QUÍMICA – PROF. PEDRO MADEIRA (2022) following three sets of condition, to a final pressure of 1 atm 1. Isothermal reversible expansion 2. Adiabatic reversible expansion 3. Irreversible adiabatic expansion carried out as follows: Assume the pressure is suddenly released to 1 atm and the gas expands adiabatically at constant pressure. [Note that the molar heat capacity at constant volume is given by the relation: CV = 3R/2, whereR is the gas constant.] 185. (IChO) Determine the work done by 1 mol of N2 gas when it expands reversibly and isothermally at 300 K from 1.00 L to 10.0 L, treating it as a van der Waals gas. 122 APOSTILA 01 DE FÍSICO-QUÍMICA – PROF. PEDRO MADEIRA (2022) CAPÍTULO 02 – TERMODINÂMICA QUÍMICA GABARITO TÓPICO 01 SEÇÃO VESTIBULARES 1. a) eutética b) 15130 g 2. OPÇÃO E 3. OPÇÃO C 4. OPÇÃO D 5. t = 32 min 6. SOMA 07 7. a) C = 13,5 J / oC b) θ = 432ºC 8. OPÇÃO B 9. OPÇÃO C 10. a) b) libera pois há aquecimento da água após a reação. c) 1,0 mol/L 11. a) N = 1 mol de glicose b) E = 2,8x10 – 19 J c) n = 1025 fótons d) V = 134,4 L 12. a) b) Xbase = 2/3 c) ácido oxálico. Proporção de 2:1. d) è1400J: a base é limitante è 1500J: o ácido é limitante. PROF. PEDRO MADEIRA SEÇÃO ITA / IME 13. Reação: Ba(OH)2.8H2O(s) + 2 NH4NO3(s) à 2 NH4OH(aq) + Ba(NO3)2(aq) + 8 H2O(l) Como a reação é endotérmica, retira calor do ambiente (incluindo da água que está na madeira), deste modo, como a reação é fortemente endotérmica, retira calor da água suficiente para causar sua solidificação, assim, o gelo formado une a madeira ao frasco de vidro. 14. OPÇÃO E 15. OPÇÃO C 16. OPÇÃO B 17. OPÇÃO C 18. OPÇÃO B PROF. PEDRO MADEIRA 123 APOSTILA 01 DE FÍSICO-QUÍMICA – PROF. PEDRO MADEIRA (2022) TÓPICO 02 SEÇÃO VESTIBULAR 19. OPÇÃO E 20. SOMA 06 21. VVFFV 22. OPÇÃO C 23. OPÇÃO C 24. OPÇÃO A PROF. PEDRO MADEIRA SEÇÃO ITA / IME 25. OPÇÃO A 26. OPÇÃO A Em B. Pois há o dobro de produto formado e uma massa de solução 1,5x maior. 27. OPÇÃO A 28. OPÇÃO C 29. OPÇÃO E 30. OPÇÃO D 31. OPÇÃO B 32. OPÇÃO B 33. OPÇÃO D 34. OPÇÃO D 35. OPÇÃO C 36. OPÇÃO E 37. A) | w | = PV; | DU | = 0; | Q | = PV B) | w | = PV; | DU | = PV; | Q | = 0 38. OPÇÃO A 39. OPÇÃO C 40. OPÇÃO E 41. OPÇÃO B 42. OPÇÃO E 43. OPÇÃO A 44. AgCl(s): DU = – 125,8 kJ/mol CaCO3(s): DU = – 1203,3 kJ/mol H2O(l): DU = – 282,3 kJ/mol H2S(g): DU = – 20 kJ/mol NO2(g): DU = – 35,2 kJ/mol 45. OPÇÃO C 46. a) 4,88x1015 J b) A molécula de água é a única que faz ponte de hidrogênio entre os calcogenetos, por esse motivo possui maior ponto de ebulição. c) Para que as pontes de hidrogênio sejam rompidas, uma grande quantidade de energia se faz necessária. A elevada Teb está relacionada com estas ligações de hidrogênio. Assim, como a precipitação pluviométrica é representada pela formação das ligações de hidrogênio, uma grande quantidade de energia é liberada. 47. OPÇÃO B 48. OPÇÃO C 49. Como DU = DH – DngasesRT, o DU será igual ao DH quando o número de mol de gases nos reagentes for igual ao número de mol de gases nos produtos. 50. OPÇÃO C 51. a) P = 4 atm; T = 1200 K b) m = 0,10 g 52. a) SRR: 2HOCℓ(aq) + 2e– + 2H+(aq) → Cℓ2(g) + 2H2O(ℓ) SRO: PbSO4(s) + 2H2O(ℓ) → PbO2(s) + 2e– + HSO4–(aq) + 3H+(aq) PbSO4(s) + 2HOCℓ(aq) → Cℓ2(g) + PbO2(s) + HSO4 – (aq) + H+ (aq) DEo = 1,61 – 1,63 = – 0,02 V b) ∆Y<= = + 17,4 kJ c) uma vez que há a produção de gases na reação, ocorre trabalho de expansão volumétrica, consumindo parte da energia para tal processo. Logo, o saldo energético global (DU) é menor do que apenas o calor envolvido a pressão constante (DH). 53. 80% de enxofre 54. V = 6,64 m3 55. T = 1760 K; [H2O] = 8,7x10–4 mol.L–1 56. DfHo (C10H8) = + 20,6 kcal / mol 57. A) DcHo = – 780,89 kcal / mol B) DH = – 752 kcal / mol Se ocorre compressão qv > qp Se ocorre expansão qv < qp 58. V = 1,22 m3. 59. OPÇÃO D 60. P =7,5xPo è aumento de 650% 61. a) DU = – 134.812 cal b) DU = – 115.004 cal c) DU = 9.904 cal/mol 124 APOSTILA 01 DE FÍSICO-QUÍMICA – PROF. PEDRO MADEIRA (2022) 62. TF = 32,8ºC 63. OPÇÃO A PROF. PEDRO MADEIRA TÓPICO 03 SEÇÃO VESTIBULARES 64. OPÇÃO D 65. OPÇÃO A 66. A) DG = + 21,106 kcal B) Não; | TDS | < | DH | C) quanto maior a temperatura, mais a posição do equilíbrio está deslocada no sentido dos produtos. 67. A) DGo = +27,6 kcal.mol–1, não espontânea. B) DSo = +38,6 cal.mol–1K–1. C) T > 1013 K; T > 740oC. D) Altas pressões. 68. T > 397,5 K 69. A) DG = 0; DS > 0. B) DH > 0; DS > 0; DG > 0. C) Espontânea acima de 100oC (1 atm) , pois DG < 0. PROF. PEDRO MADEIRA SEÇÃO ITA / IME 70. OPÇÃO D 71. A) DS > 0, mesma Ec, menor pressão. B) DS < 0, forças coesivas mais fortes no diamante. C) DS < 0, se houver formação de precipitado. DS < 0, se o sistema for apenas a solução (águas de solvatação diminuem). D) DS < 0, sólido mais organizado. E) DS < 0, diminuição do movimento molecular. 72. OPÇÃO C 73. OPÇÃO A 74. A à B: Expansão isotérmica B à C: Expansão adiabática C à D: Compressão isotérmica D à A: Compressão adiabática 75. OPÇÃO D 76. OPÇÃO D 77. OPÇÃO E 78. OPÇÃO C 79. OPÇÃO A 80. DG = – 153,1 kJ/mol 81. DU = – 966,6 kJ 82. A) ortorrômbica B) DH > 0 83. A reação é termodinamicamente favorável quando T > 1111 K 84. OPÇÃO E 85. OPÇÃO E 86. OPÇÃO A 87. OPÇÃO C 88. OPÇÃO D 89. OPÇÃO C 90. OPÇÃO D 91. DS = 8,10 J/K 92. OPÇÃO D 93. OPÇÃO C 94. è Calorimetria: mCp(T + ΔT – TF) = mCp (TF – T) TF = T + KL ! è ΔSA = mCpQ( &MN,O∆& & è ΔSB = mCp Q( &MN,O∆& &MQ& 125 APOSTILA 01 DE FÍSICO-QUÍMICA – PROF. PEDRO MADEIRA (2022) è ΔSTOT = ΔSA + ΔSB ΔSTOT = mCp Q( (&MN,O∆&)" &∙(&MQ&) ΔSTOT = mCp Q( &"M&∙∆&MN,!O∆&" &"M&∙∆& è Perceba que o numerador é maior do que o denominador. Logo, o ΔSTOT > 0. 95. OPÇÃO D 96. OPÇÃO B 97. a) DSSISTEMA = -102,3 J.K–1 b) DSVIZINHANÇA = +106,8 J.K–1 c) DSUNIVERSO = +4,5 J.K–1 PROF. PEDRO MADEIRA TÓPICO 04 SEÇÃO VESTIBULARES 98. OPÇÃO C 99. OPÇÃO E 100. A) DHo = – 32,70 kcal.mol–1 B) 101. A) DHo = 131,3 kJ (Endotérmica) B) O elemento C muda de estado de oxidação de zero para +2 na substância composta CO, portanto sofre uma oxidação, sendo classificado como agente redutor. Já o H muda de +1 na substância composta H2O para zero na substância simples H2, sofrendo uma redução, sendo a molécula H2O classificada como agente oxidante. 102. A) O calor liberado por grama de substância será: H2 : q = – 143,0 kJ g–1 CH3OH: q = – 22,69 kJ g–1 C2H5OH: q = – 29,72 kJ g–1 C8H18: q = – 47,98 kJ g–1 A substância que libera a maior quantidade de energia por grama é o H2, apresentando, portanto, a maior eficiência nesta situação. B) A massa de 1 mL de cada substância será: H2 : 0,07 g; CH3OH: 0,79 g; C2H5OH: 0,80 g e C8H18: 0,70 g. Portanto, a energia liberada por mL de substância será: H2 : DH = – 10,01 kJ mL–1 CH3OH: DH = – 17,93 kJ mL–1 C2H5OH: DH = – 23,78 kJ mL–1 C8H18: DH = – 33,59 kJ mL–1 A substância que libera a maior quantidade de energia por mL é o C8H18 (octano), apresentando, portanto, a maior eficiência nesta situação. 103. a) b) 2 NCℓ3 → N2 + 3 Cℓ2 Reagente = +3; Produto = 0 c) DH = – 230 kJ/mol; exotérmcia 104. A) DHI = – 184 kJ e DHII = – 78 kJ B) Reação I 105. A) ainda restarão 974,4 g de H2O na fase líquida. B) q = 2261 kJ. 106. OPÇÃO E 107. OPÇÃO C 108. DHo = – 152 kcal.mol–1 109. a) –585 kJ/mol; b) + 101 kJ/mol 110. a) C8H18(v) + 25/2 O2(g) à 8CO2(g) + 9H2O(l) b) DH = – 3419,5 kJ/mol c) 30776 kJ 111. a) ligação de hidrogênio b) 170 kJ/mol 112. a) DH = – 1970 kJ/mol; b) 4,83x104 m 113. a) b) C5H10 + 15/2 O2 à 5 CO2 + 5 H2O C C H H H H + H2 Pt H C C H H H HH Cl N Cl Cl 126 APOSTILA 01 DE FÍSICO-QUÍMICA – PROF. PEDRO MADEIRA (2022) c) DH’s diferentes pois as entalpias iniciais são diferentes e as finais são as mesmas em cada reação. 114. a) 2NH4ClO4 à N2 + 2 O2 + Cl2 + 4 H2O b) 6NH4ClO4 + 8 Al à 3 N2 + 3Cl2 + 12H2O + 4Al2O3c) 2,24x103 kJ (calor liberado a mais) 115. a) C5H8N4O12 à 2CO +3CO2 + 2N2 + 4H2O b) B.Ox = –10,12% c) DH = – 1832 kJ/mol d) quanto mais próximo de zero for o balanço de oxigênio, mais eficiente será a liberação de calor. PROF. PEDRO MADEIRA 116. OPÇÃO A SEÇÃO ITA / IME 117. OPÇÃO D Calor envolvido na transformação de 1 mol de C(grafite) em 1 mol de C(diamante) a pressão constante 118. OPÇÃO D 119. OPÇÃO D 120. OPÇÃO C 121. E = hc / l; Quanto maior a energia, menor o comprimento de onda. 122. OPÇÃO A 123. OPÇÃO A 124. OPÇÃO D 125. OPÇÃO C 126. OPÇÃO E 127. OPÇÃO D 128. OPÇÃO C 129. A) Combustão do n-hexano: 1C6H14(l) + 19/2 O2(g) à 6 CO2(g) + 7 H2O(g) DH = – 3883 kJ/mol Combustão do n-heptano: 1C7H16(l) + 11 O2(g) à 7 CO2(g) + 8 H2O(g) DH = – 4498 kJ/mol B) As moléculas de um hidrocarboneto linear podem ser entendidas como sendo CH3-(CH2)n-CH3. Portanto, a partir dos dados da questão, pode-se concluir que a adição de 1 grupo metileno (–CH2–) no n-hexano promove um aumento de 615 kJ/mol no calor liberado na combustão do n-heptano. Logo, do n-hexano para o n-decano há 4 grupos metileno a mais, o que acarretará um acréscimo do calor de combustão liberado em 4x615 kJ. Assim, temos: DcombHo (n-C10H22) = – 6343 kJ/mol C) O calor liberado seria maior, pois a água líquida tem uma entalpia menor do que a água gasosa. Uma vez que ela é produto da reação, a variação de entalpia (Hprodutos – Hreagentes) será mais negativa do que no caso da água gasosa. 130. OPÇÃO E 131. OPÇÃO B 132. A) n = 0,06 mol B) q = 8,25 x 10–3 kJ 133. OPÇÃO E 134. No processo de fusão, deve haver grande absorção de energia a fim de que haja a ruptura das ligações de hidrogênio no gelo. Contudo, a formação de pontes de hidrogênio na fase líquida é acompanhada pela liberação de certa quantidade de calor. O saldo final de calor liberado é uma medida desses dois processos de absorção e liberação de calor. Já no estado vapor, as ligações de hidrogênio não são importantes (por exemplo, para um gás ideal não há interações intermoleculares), frente ao grande número e intensidade das mesmas na fase líquida. Portanto, o calor absorvido na ruptura das ligações de hidrogênio no líquido não tem uma contrapartida de calor liberado na fase vapor. Desta forma, DHvap > DHfus. 135. OPÇÃO C 136. 137. OPÇÃO E 138. OPÇÃO D 139. A) O Al2O3 forma uma camada protetora sobre o polímero dificultando seu contato com o gás oxigênio (comburente). B) Da mesma forma, a H2O(g) também irá atuar dificultando tal contato. C) DH > 0 implica em absorção de calor. Portanto, parte do calo disponível para ativar a combustão será empregada na decomposição do hidróxido de alumínio. 127 APOSTILA 01 DE FÍSICO-QUÍMICA – PROF. PEDRO MADEIRA (2022) 140. OPÇÃO B 141. OPÇÃO C 142. OPÇÃO D 143. a) q = – 73.465 kJ b) q = – 203.500 kJ c) q = – 87,5 kJ d) q = – 211,64 kJ 144. OPÇÃO A 145. OPÇÃO C 146. OPÇÃO C 147. a) 33,3 kg b) 156,34 L c) 1,37x105 L 148. a) 6 CO2(g) + 6 H2O(l) à C6H12O6(s) + 6 O2(g) b) Q = 4,664x10–21 kJ/molécula c) EFÓTON = 4,655x10–19 J d) n > 10; assim, no mínimo são 11 fótons. 149. OPÇÃO D 150. a) 1C2H6O(ℓ) + 3O2(g) → 2CO2(g) + 3H2O(ℓ) b) 1C2H6O(ℓ)+3O2(g)+ 12N2(g) → 2CO2(g) + 3H2O(ℓ) + 12N2(g) c) 1C2H6O(ℓ)+3/2O2(g)+6N2(g)→3H2O(ℓ)+1CO(g)+1C(s)+6N2(g) d) as duas primeiras reações são combustões completas (são mais exotérmicas). A última é uma combustão incompleta (menos exotérmica). Logo, ΔH(a) = ΔH(b) < ΔH(c) 151. a) nA = 0,25 mol; nB = 0,75 mol; nc = 1/3 mol b) ΔH = – 792 kJ.mol–1. 152. a) ΔHf(NaCl) = - 421,5 kJ.mol–1 b) O CaO possui íons de maiores cargas, o que contribui para que a sua entalpia de rede (em módulo) seja maior do que a do NaCl. 153. Usando o DHeb o = +10,5 kcal mol–1 (valor correto) a) ΔHo = − 213 kcal/mol b) 1,0x105 kcal/h c) 7,51 kg/h Usando o DHeb o = –10,5 kcal mol–1 (valor dado) a) ΔHo = − 171 kcal/mol b) 1,0x105 kcal/h c) 9,36 kg/h 154. a) PCI(metano) = 792 MJ.kg–1; PCI(etanol) = 1248 MJ.kg–1 b) 94,5% de CH4 c) Primeiramente, deve-se salientar que a oxidação que o carbono do metano sofre na combustão completa a CO2 é mais intensa do que a do etanol. Isto contribui para que o calor liberado por unidade de massa seja maior no caso do metano do que no caso do etanol. A entalpia molar de combustão do etanol é maior do que a do metano, o que se explica pela maior massa molar. Contudo, o acréscimo de calor não é proporcional ao acréscimo de massa pelo exposto anteriormente. Assim, ao se dividir pela massa molar, a PCS do metano se torna maior do que a PCS do etanol. 155. OPÇÃO D 156. OPÇÃO D 157. a) I. Ca(g) → Ca+(g) + e– DH = 590 kJ.mol–1 II. Ca+(g) → Ca2+(g) + e– DH = 1145 kJ.mol–1 III. Cℓ(g) + e– → Cℓ–(g) DH = -340 kJ.mol–1 IV. CaCℓ2(s) → Ca2+(aq) + 2Cℓ–(aq) DH = -81 kJ.mol–1 V. Ca2+(g) → Ca2+(aq) DH = -1579 kJ.mol–1 VI. Cℓ–(g) → Cℓ–(aq) DH = -378 kJ.mol–1 b) CaCℓ2(s) → Ca2+(aq) + 2Cℓ–(aq) DH = -81 kJ.mol–1 Ca2+(aq) → Ca2+(g) DH = +1579 kJ.mol–1 2Cℓ–(aq) → 2Cℓ–(g) DH = 2x378 kJ.mol–1 CaCℓ2(s) → Ca2+(g) + 2Cℓ–(g) DH = 2254 kJ.mol–1 c) DH = 2254 kJ.mol–1 158. n(CH4) = 0,76 mol; n(C2H4) = 0,24 mol 159. 1 CO + 2 H2 à 1 CH3OH DH = – 119,83 kJ/mol 128 APOSTILA 01 DE FÍSICO-QUÍMICA – PROF. PEDRO MADEIRA (2022) Q = - 7,49 kJ / 2,0 g de metanol 160. m = 900g 161. c = de ∙ .S" T − S& U /d kcal 162. PCO / PH2 = 1,5 163. OPÇÃO A 164. ΔH°f,CH4 = 2x ΔH°f,H2O(g) + ΔH°f,CO2 – ΔHr 165. T = 1188 K; Q = 2749 kJ 166. Calor liberado para a vizinhança = 192,6 kJ/mol 167. T = 369 K 168. DH = + 207 kJ/mol 169. OPÇÃO D 170. DH = - 4,08x103 J/mol 171. DH = - 29359 kJ/mol 172. a) 0,0562 m3/s b) 73,4% N2; 12,0% H2O; 8,5% CO2; 4% O2; 2,1% CO TF = 2418 K PROF. PEDRO MADEIRA “Quem pergunta é bobo por cinco minutos; quem não pergunta é bobo para sempre.” Provérbio Chinês 129 APOSTILA 01 DE FÍSICO-QUÍMICA – PROF. PEDRO MADEIRA (2022) LISTA EXTRA – GASES 01. Um estudante ligou um bulbo de vidro contendo gás neônio a um manômetro de tubo aberto e verificou que a pressão do gás é 0,890 atm. (a) Se a pressão atmosférica é 762 Torr, que diferença de altura entre os dois lados do mercúrio o estudante encontrou? (b) Que lado está mais alto, o lado do manômetro ligado ao bulbo ou o lado aberto à atmosfera? (c) Se o estudante se enganou e trocou as leituras dos lados do manômetro ao registrar os dados no caderno de laboratório, qual seria a pressão do bulbo que ele registrou? 02. O gás de um composto fluorado de metano tem densidade 8,0 g.L–1, em 2,81 atm e em 300 K. (a) Qual é a massa molar do composto? (b) Qual é a fórmula do composto sabendo-se que ele é formado somente por C, H e F? (c) Qual é a densidade do gás em 1,00 atm e 298 K? 03. Um composto usado na fabricação de saran é 24,7% C, 2,1% H e 73,2% Cl em massa. O armazenamento de 3,557g do composto em um recipiente de 755 mL, em 0oC, eleva a pressão até 1,10 atm. Qual é a fórmula molecular do composto? 04. O processo Haber de síntese da amônia é um dos processos industriais mais importantes para o bem-estar da humanidade. Ele é muito usado na produção de fertilizantes, polímeros e outros produtos. (a) Que volume de hidrogênio em 1,00 atm e 350oC deve ser usado para produzir 1,0 tonelada de NH3? (b) Que volume de hidrogênio seria necessário em (a) se o gás fosse fornecido em 3,76x102 atm e 250ºC? 05. O dióxido de carbono e a água produzem, através de uma série de etapas enzimáticas do processo de fotossíntese, glicose e oxigênio, de acordo com a equação: 6 CO2(g) + 6 H2O(l) à C6H12O6(s) + 6 O2(g). Sabendo-se que a pressão parcial do dióxido de carbono na troposfera é 0,26 Torr e que a temperaturaé 25ºC, calcule o volume de ar necessário para produzir 10,0g de glicose. 06. A pressão total de uma mistura dos gases dióxido de enxofre e nitrogênio, em 25ºC, em um recipiente de 500 mL, é 1,09 atm. A mistura passa sobre óxido de cálcio em pó quente, que remove o dióxido de enxofre pela reação CaO(s) + SO2(g) à CaSO3(s), e é transferida para um recipiente de 150 mL, no qual a pressão é 1,09 atm em 50ºC. (a) Qual era a pressão parcial de SO2 na mistura inicial? (b) Qual era a massa de SO2 na mistura inicial? 07. Durante um experimento de eletrólise de água, o gás hidrogênio foi coletado em um dos eletrodos sob água em 20ºC e pressão externa igual a 756,7 Torr. A pressão de vapor da água em 20ºC é 17,54 Torr. O volume do gás era 0,220 L. (a) Qual é a pressão parcial do hidrogênio? (b) O outro produto da eletrólise da água é o gás oxigênio. Escreva uma equação balanceada para a eletrólise da água para dar H2 e O2. (c) Que massa de oxigênio foi produzida na reação? 08. Um hidrocarboneto de fórmula empírica C2H3 levou 349s para efundir por uma rolha porosa. Nas mesmas condições de temperatura e pressão, são necessários 210s para que ocorra a efusão da mesma quantidade de átomos de argônio. Determine a massa molar e a fórmula molecular do hidrocarboneto. 09. Calcule a energia cinética molar (em joules) de uma amostra de gás criptônio em (a) 55,85ºC e (b) 54,85ºC (c) A diferença de energia molar entre as respostas de (a) e (b) é a energia por mol necessária para elevar a temperatura do gás criptônio de 1,00oC. Essa quantidade é conhecida como capacidade calorífica molar. Quanto ela vale? 10. Considere a distribuição de velocidade de Maxwell. (a) A partir do gráfico, encontre o ponto que representa a velocidade mais provável das moléculas. (b) O que acontece com a percentagem de moléculas que têm a velocidade mais provável quando a temperatura aumenta? 11. A pressão de uma amostra de fluoreto de hidrogênio é mais baixa do que a esperada e, com o aumento da temperatura, sobe mais depressa do que o predito pela lei dos gases ideais. Forneça uma explicação. 12. Use a equação do gás ideal para calcula a pressão, em 298 K, exercida por 1,00 mol de CO2(g) quando limitado ao volume de (a) 15,0 L; (b) 0,500 L; (c) 50,0 mL. Repita estes cálculos usando a equação de van der Waals. O que estes cálculos indicam sobre a precisão da dependência da pressão na lei dos gases ideais? DADOS: a = 3,592 atm.L2/mol2; b = 0,04267 L/mol 13. (a) Calcule a pressão de CO2(g) confinado em um vaso de 1,00 L em temperatura constante (27ºC). Use a lei dos gases ideais e a equação de van der Waals no intervalo de 0,100 mol CO2 a 0,500 mol CO2, em incrementos de 0,100 mol. 130 APOSTILA 01 DE FÍSICO-QUÍMICA – PROF. PEDRO MADEIRA (2022) (b) Calcule o desvio percentual entre o valor ideal e o valo “real” (calculado pela equação de van der Waals) em cada ponto. (c) Nestas condições, que termo tem o maior efeito na pressão real de CO2, as atrações intermoleculares ou o volume molar? (d) Se considerarmos como ideais os gases para os quais a pressão observada difere menos de 5% do valor ideal, a que pressão o CO2 torna-se um gás “real”? 14. A fumaça poluente fotoquímica é formada, em parte, pela ação da luz sobre o dióxido de nitrogênio e reação posterior com O2 para produzir O3. O comprimento da radiação absorvida pelo NO2 nesta reação é 197 nm. NO2 + hn à NO + O O + O2 à O3 + M ( M = N2 ou O2) (a) Desenhe a estrutura de Lewis de NO2 e faça um esquema de seus orbitais p. (b) Quando 1,07 mJ de energia são absorvido por 2,5 L de ar em 20ºC e 0,85 atm, todas as moléculas NO2 da amostra se dissociam segundo a reação acima. Imagine que cada fóton absorvido resulte na dissociação (em NO e O) de uma molécula NO2. Qual é a proporção, em partes por milhão, de moléculas NO2 na amostra? Imagine que a amostra tem comportamento ideal. 15. Suponha que 200 mL de cloreto de hidrogênio, em 690 Torr e 20ºC, foram dissolvidos em 100 mL de água. A solução foi titulada até o ponto estequiométrico com 15,7 mL de uma solução de hidróxido de sódio. Qual é a concentração molar do hidróxido de sódio em solução? 16. Um frasco de volume 5,00 L foi evacuado e 43,78g de tetróxido de dinitrogênio, N2O4, foram admitidos. Em - 196ºC, este composto é um sólido incolor. A amostra foi aquecida até 25ºC e no processo, N2O4 se vaporiza e se dissocia parcialmente para formar o gás NO2. A pressão cresce lentamente e se estabiliza em 2,96 atm. (a) Escreva uma equação para a reação. (b) Se o gás que está no frasco fosse exclusivamente N2O4, qual seria a pressão? (c) Se todo o gás que está no frasco fosse NO2, qual seria a pressão? (d) Quais são as frações molares de N2O4 e NO2 quando da pressão se estabiliza em 2,96 atm? 17. Determine a razão entre o número de moléculas de um gás que tem velocidade dez vezes maior do que a da velocidade quadrática média e o número de moléculas que tem velocidade igual à da velocidade quadrática média. Será que esta razão é independente da temperatura? Por quê? DADO: #(H) = IJ1 ; CJ*'7 3/! H! ∙ 0#45"/'& 18. Um estudante de pós-graduação tem de preparar gás fosgênio marcado isotopicamente, 13COCl2(g), para uso na síntese de um composto orgânico (todos os átomos C são carbono-13). O estudante colocou 3,59 atm de 13CO(g) e 2,75 atm Cl2(g) em um cilindro de aço, em 25ºC, selou o cilindro e o aqueceu até 227ºC. O cilindro foi mantido nesta temperatura elevada por uma semana em 227ºC, chegou a 9,75 atm. (a) Quais são as frações molares de 13CO, Cl2 e 13COCl2 presentes no cilindro no fim da semana? (b) Qual é a densidade da mistura? 19. O espalhamento de odores pelo ar é devido à difusão de moléculas de gás. Alguém abriu um frasco contendo octanoato de etila na extremidade norte de uma sala de 5 m de comprimento e, simultaneamente, outra pessoa abriu um frasco contendo p-anisaldeído na extremidade sul da sala (C10H29O2) tem odor semelhante ao de frutas e o p-anisaldeído, odor semelhante ao da hortelã. A que distância (em metros) da extremidade norte deve estar uma pessoa para sentir primeiro o cheiro de hortelã? 20. Uma amostra sólida, finamente pulverizada, de um óxido de ósmio (que funde em 40ºC e ferve em 130ºC), cuja massa é 1,509g, foi colocada em um cilindro dotado de um pistão móvel que pode se expandir contra a pressão atmosférica de 745 Torr. Imagine que a quantidade de ar residual inicialmente presente no cilindro é desprezível. Quando a amostra é aquecida até 200ºC, ocorre vaporização completa e o volume do cilindro se expande até 235 mL. Qual é a massa molar do óxido? Imaginando que a fórmula do óxido é OsOx, qual é o valor de x? 21. (a) Os parâmetros de van der Waals do hélio são a = 3,412x10–2 L2.atm.mol–2 e b = 2,370x10–2 L.mol–1. Calcule, a partir dos parâmetros de van der Waals, o volume aparente (em pm3) e o raio (em pm) de um átomo de hélio. (b) Estime o volume de um átomo de hélio na base do raio atômico. (c) Como estas quantidades se comparam? Será que elas deveriam ser iguais? Discuta. 22. Um litro de gás cloro, em 1 atm e 298K, reage completamente com 1,00 L de gás nitrogênio e 2,00 L de gás oxigênio na mesma temperatura e mesma pressão. Forma-se um único produto gasoso, que enche um frasco de 2,00 L, em 1,00 atm e 298 K. Use estas informações para determinar as seguintes características do produto: (a) sua fórmula empírica; (b) sua fórmula molecular; (c) a fórmula de Lewis mais favorável com base em argumentos de carga formal (o átomo central é N); a forma da molécula. 131 APOSTILA 01 DE FÍSICO-QUÍMICA – PROF. PEDRO MADEIRA (2022) 23. A equação de van der Waals pode ser rearranjada em uma equação cúbica: %3 + (1 *' + ,! ! 7%! + f (!- ! g% − (3-, ! = B (a) Use esta equação para calcular o volume ocupado por 0,505 mol de NH3(g) em 25ºC e95,0 atm. Os parâmetros de van der Waals do NH3 são a = 4,225x10–2 L2.atm.mol–2 e b = 3,707x10–2 L.mol–1. (b) Que forças predominam, nesta temperatura e pressão, as atrativas ou as repulsivas? 24. Um manômetro conectando dois frascos (rotulados A e B) contém um óleo com uma densidade de 0,847 g/cm3. O óleo no braço conectado ao frasco A está 74 cm mais alto que o óleo no braço conectado ao frasco B. O gás no frasco A tem uma pressão de 97,7 kPa (836 Torr). Qual é a pressão do gás no frasco B? 25. Uma bomba de bicicleta possui um pistom de 75,0 cm de comprimento. Assumindo que, ao se puxar o êmbolo, o ar é introduzido a uma pressão de 101 kPa, de quanto deve ser esta êmbolo comprimido (em centímetros) para que a pressão do ar atinja 555 kPa, se a temperatura do ar permanecer constante? (Essa pressão corresponde, aproximadamente, à pressão de um pneu numa bicicleta de 10 marchas). 26. Um gás é coletado sobre água até a pressão total interna de um frasco de 100 cm3 ser de 93,3 kPa a 25ºC. Calcule o volume do gás seco nas CNTP. 27. Uma amostra de 0,2000g de um líquido com odor de peixe, contendo somente carbono, hidrogênio e nitrogênio, foi queimada e produziu 0,482g de CO2 e 0,271g de H2O. Uma segunda amostra, pesando 0,2500g, foi tratada de tal forma que todo o nitrogênio contido na substância foi convertido em N2. Este gás foi coletado, ocupando um volume de 42,3 cm3, a 26,5ºC e 100,6 kPa. (a) Qual a porcentagem, em massa, de carbono, hidrogênio e nitrogênio no composto? (b) Qual a fórmula empírica do composto? 28. O oxigênio gasoso, gerado na reação KClO3 à KCl + O2 (não-balanceada), foi coletado sobre água a 30ºC em um vaso de 150 cm3 até a pressão total ser de 80,0 kPa. (a) Quantos gramas de o2 seco foram produzidos? (b) Quantos gramas do KClO3 foram consumidos na reação? 29. Use a equação de van der Waals para calcular a pressão de 1,000 mol de C2H6 a 0oC em um volume de 22,400 dm3. Compare com a pressão de um gás ideal sob estas mesmas condições. 30. São coletados 280 cm3 de gás sobre água a 20ºC. O nível da água dentro do frasco coletor está 28,4 mm mais alto que o nível da água do lado de fora. A pressão atmosférica é de 102 kPa. Qual seria o volume de gás seco nas CNTP? 31. O ozônio, O3, é uma espécie importante na cadeia de reações que conduzem à produção do nevoeiro. Em uma análise de ozônio, 2,0x104 dm3 de ar, nas CNTP, foram passados através de uma solução de NaI, onde o O3 sofre a reação O3 + 2 I– + H2O à O2 + I2 + 2 OH– O I2 formado foi titulado com 0,0100 M de Na2S2O3, com o qual reagiu: I2 + 2 S2O3 2– à 2 I– + S4O6 2– Na análise, foram necessários 0,042 cm3 da solução de Na2S2O3 para reagir completamente com todo o I2. (a) Calcule o número de moles de I2 que reagiram com a solução de S2O3 2– (b) Quantos moles de I2 foram produzidos na primeira reação? (c) Quantos moles de O3 estavam contidos nos 20000 dm3 de ar? (d) Que volume o O3 ocuparia nas CNTP? (e) Qual é a concentração de O3, em partes por milhão em volume, na amostra de ar? GABARITO 01. (a) 86 mmHg; (b) O lado direito do bulbo está mais alto; (c) 848 Torr. 02. (a) 70 g.mol–1 ; (b) CHF3; (c) 2,9 g.L–1 03. C2H2Cl2 04. (a) 3,0x105 L; (b) 1,0x104 L 05. 2,4x104 L 06. (a) 0,787 atm; (b) 1,03 g 07. (a) 739,2 Torr; (b) H2O(l) à H2(g) + ½ O2(g); (c) 0,142g 08. 110g/mol; C8H12 09. (a) 4103,2 J/mol; (b) 4090,7 J/mol; 132 APOSTILA 01 DE FÍSICO-QUÍMICA – PROF. PEDRO MADEIRA (2022) (c) 12,5 J/mol 10. (a) A velocidade mais provável é a que corresponde ao máximo da curva de distribuição. (b) A percentagem decresce. 11. Em temperaturas baixas, as ligações hidrogênio fazem com que as moléculas de HF se atraiam mais fortemente e a pressão cai. Quando a temperatura aumenta, as ligações hidrogênio se quebram e a pressão sobe mais depressa do que para um gás ideal. 12. (a) 1,63; 1,62 atm. (b) 48,9; 38,9 atm. (c) 489; 1,88x103 atm. Em pressões baixas, a lei do gás ideal dá essencialmente o mesmo valor da equação de van der Waals, mas em pressões elevadas existem diferenças importantes. 13. (a) e (b) n Pideal Pvan der Waals % de desvio 0,100 2,46 2,44 0,8 0,200 4,92 4,82 2,1 0,300 7,38 7,15 3,2 0,400 9,85 9,44 4,3 0,500 12,31 11,67 5,5 (c) Pvan der Waals < Pideal, o que sugere que as atrações intermoleculares são mais importantes. (d) em pressões superiores a cerca de 10 atm. 14. (a) (b) 0,020 ppm 15. 0,481 M 16. (a) N2O4(g) à 2 NO2(g) (b) 2,33 atm; (c) 4,65 atm; (d) X(NO2) = 0,426; X(N2O4) = 0,574 17. #(2BHh) #(Hh) = 2BB ∙ 0#WW45X" !'&⁄ Não, a distribuição varia conforma a temperatura. 18. (a) X(COCl2) = 0,090; X(CO) = 0,527; X(Cl2) = 0,383; (b) 7,20 g/L 19. distância > 2,35 m 20. 254 g/mol; x = 4 21. (a) V = 3,936x107 pm3; r = 211 pm. (b) r = 128 pm; V = 8,78x106 pm3. (c) A diferença entre esses valores mostra que não existe definição simples para os limites de um átomo. O valor de van der Waals obtido da correção para o volume molar é consideravelmente maior do que o raio atômico, devido, talvez ao maior alcance e às interações fracas entre átomos. Lembre-se de que o valor da constante b de van der Waals é um parâmetro usado para obter um bom ajuste a uma curva e sua interpretação inclui outros fatores além do volume molar. 22. (a) ClNO2; (b) ClNO2; (c) (d) trigonal planar. 1 23. (a) V = 0,0979 L. (b) Videal = 0,130 L < VvdW; portanto, as forças atrativas dominam. 24. 117,6 kPa 25. 61,4 cm 26. 81,5 cm3 27. (a) 65,8% C; 15,2% H; 19,2% N (b) C4H11N 28. (a) 0,144g (b) 0,368g 29. 100,6 kPa (gás ideal, P = 101,3 kPa) 30. 256 cm3 O N O O N O .. .. .. .. . .. .. .. .. ..... O N O Cl O N O Cl.. .. .. .. .. .. .. .. .... .. .... .. .... 133 APOSTILA 01 DE FÍSICO-QUÍMICA – PROF. PEDRO MADEIRA (2022) LISTA EXTRA – TERMODINÂMICA QUÍMICA 01. Um gás em um cilindro foi colocado em um aquecedor e ganhou 5500 kJ de calor. Se o volume do cilindro aumentou de 345 mL para 1846 mL contra uma pressão atmosférica de 750 Torr durante o processo, qual é a variação de energia interna do gás no cilindro? 02. Calcule o trabalho em cada um dos seguintes processos, começando com uma amostra de gás em um sistema com pistão com T = 305 K, P = 1,79 atm e V = 4,29 L. (a) expansão irreversível contra a pressão externa constante de 1,00 atm, até o volume final 6,52 L; (b) expansão reversível isotérmica até o volume final 6,52 L. 03. Que gás tem maior capacidade calorífica molar, NO ou NO2? Por quê? 04. Calcule o calor liberado por 5,025g de Kr(g), em 0,400 atm, que esfria de 97,6ºC até 25,0oC, (a) em pressão constante e (b) em volume constante. Imagine que o criptônio comporta-se como um gás ideal. 05. A contribuição dos modos vibracionais de alta temperatura para a capacidade calorífica molar de um sólido em volume constante é R para cada modo de vibração. Portanto, para um sólido atômico, a capacidade calorífica molar em volume constante é aproximadamente 3R. (a) A capacidade calorífica específica de um determinado sólido atômico é 0,392 J.K–1.g–1. O cloreto deste elemento (XCl2) é 52,7% cloro em massa. Identifique o elemento. (b) Esse elemento cristaliza em uma célula unitária cúbica de face centrada e seu raio atômico é 128 pm. Qual é a densidade desse sólido atômico? 06. A entalpia de formação do trinitro tolueno (TNT) é -67 kJ/mol e, a densidade, 1,65 g/cm3. Em princípio, ele poderia ser usado como combustível de foguetes, com os gases formados na decomposição saindo do foguete para dar o impulso necessário. Na prática, é claro, ele seria extremamente perigoso como combustível, porque é sensível ao choque. Explore seu potencial como combustível de foguete, calculando a densidade de entalpia (a entalpia liberada por litro) na reação 4C7H5N3O6(s)+21O2(g)à 28CO2(g)+10H2O(g)+6N2(g) 07. Dois estágios sucessivos da preparação industrial do ácido sulfúrico são a combustão do enxofre e a oxidação do dióxido de enxofre. A partir das entalpias padrão de reação S(s) + O2(g) à SO2(g) DHo = – 296,83 kJ 2 S(s) + 3 O2(g) à 2 SO3(g) DHo = – 791,44 kJ Calcule a entalpia de reação da oxidação do dióxido de enxofre a Trióxido de enxofre na reação 2 SO2(g) + O2(g) à 2 SO3(g) 08. Determine a entalpia da reação de hidrogenação do etino a etano C2H2(g) + 2 H2(g) à C2H6(g), a partir dos seguintes dados: DHc o(C2H2(g))=–1300 kJ/mol; DHc o(C2H6(g)) = –1560 kJ/mol; DHc o(H2(g)) = –286 kJ/mol. 09. Complete a seguinte tabela (valores em kJ/mol) MX DHfo M(g) EI M DHfo X(g) AE X DHLo MX DHfo MX(s) NaCl 108 494 122 +349 787 ? KBr 89 418 97 +325 ? -394 RbF ? 402 79 +328 774 -558 10. Derive a lei de Kirchhoff para uma reação da forma A + 2B à 3C + D, levando em conta a variação de entalpia molar de cada substância quando a temperatura aumenta de T1 até T2. 11. A luz solar forte bombardeia a Terra com cerca de 1 kJ/m2 em 1 s. Calcule a massa máxima de etanol puro que poderia ser vaporizada, em 10 min, em um bécher deixado sob luz solar forte, imaginando que a área superficial do etanol é 50 cm2 e que todo o calor é usado para a vaporização e não para aumentar a temperatura. DHvap = 38,6 kJ/mol 12. Anilina, C6H5NH2(l) é um derivado do benzeno em que um átomo de hidrogênio foi substituído por um grupo NH2. (a) Escreva a equação balanceada da combustão da anilina. (b) Qual é a massa de cada produto quando 0,1754g de anilina são queimados em excesso de oxigênio? (c) Se o calorímetro de bomba no qual essa reação foi feia tinha um volume de 355 mL, qual foi a pressão mínima de oxigênio, em 23ºC, usado para garantir a combustão completa? Considere desprezível o volume da anilina. 13. Será que a produção de gás de água (um combustível industrial barato de baixo grau calorífico) é um processo exotérmico ou endotérmico? A reação é: C(s) + H2O(g) à CO(g) + H2(g) 14. Imagine que 50,0 mL de 0,500 M NaOH(aq) e 50,0 mL de 0,500 M HNO3(aq), inicialmente em 18,6ºC, são misturados e agitados em um calorímetro cuja 134 APOSTILA 01 DE FÍSICO-QUÍMICA – PROF. PEDRO MADEIRA (2022) capacidade calorífica é 525,0 J/oC quando vazio. A temperatura da mistura subiu para 20,0oC. (a) Qual é a variação de entalpia da reação de neutralização? (b) Qual é a variação de entalpia da neutralização em kJ/mol HNO3? Considere a capacidade específica de cada solução como sendo 4,184 J/g.oC 15. Qual seria mais exotérmica, a oxidação do 1,3,5- trinitrobenzeno, C6H3(NO2)3, ou a oxidação do 1,3,5- triaminobenzeno, C6H3(NH2)3? Justifique sua resposta. 16. Durante exercícios físicos, as gorduras reagem com água para produzir ácidos graxos. Estes são, então, convertidos em água e dióxido de carbono, uma reação que libera energia. O organismo usa essa energia para suas atividades. Um ácido graxo típico é o ácido láurico, CH3(CH2)10COOH, que tem o mesmo número de átomos de carbono da sacarose C12H22O11. DHf o(CH3(CH2)10COOH) = – 774,6 kJ/mol e DHf o(C12H22O11) = – 2222 kJ/mol (a) Que massa de sacarose é necessária para produzir a mesma energia que 15,0 g de ácido láurico? (b) Por que é mais eficiente armazenar energia na forma de gordura do que na forma de carboidratos? 17. Os ácidos maléico e fumárico são dois isômeros cis e trans, respectivamente, cuja fórmula química é C4H4O4. Eles são insumos importantes para a indústria química. (a) Calcule a entalpia da isomerização cis-trans. (b) Que isômero tem a entalpia padrão de formação menor? (c) Na reação de combustão desses compostos, o mais negativo é DUc ou DHc? DHc(ácido maléico) = – 1355,2 kJ/mol DHc(ácido fumárico) = – 1334,7 kJ/mol 18. Um técnico conduz a reação 2SO2(g) + O2(g) à 2SO3(g), em 25ºC e 1,00 atm, em um cilindro dotado de um pistão, em pressão constante. No início, estão no cilindro 0,030 mol SO2 e 0,030 mol O2. O técnico adiciona, então, um catalisador para iniciar a reação. (a) Calcule o volume do cilindro que contém os gases antes do começo da reação. (b) Qual é o reagente limitante? (c) Imagine que a reação se completa e que a temperatura e a pressão permanecem constantes. Qual é o volume final do cilindro? (d) Qual é o trabalho executado? Ele é feito contra ou a favor do sistema? (e) Qual é a entalpia trocada? Ela deixa ou entra no sistema? (f) Leve em conta as respostas das partes (d) e (e) para calcular a variação de energia da reação. GABARITO 01. +5500 kJ 02.a) -226 J b) -326 J 03. NO2. Moléculas mais complexas têm mais vibrações, nas quais é possível armazenar energia. 04.a) – 90,6 J; b) – 54,4 J 05. a) Cu; b) 8,90 g/cm3 06. +23,9x103 kJ/L 07. –197,78 kJ 08. –312 kJ/mol 09. (a) –412 kJ/mol; (b) 673 kJ/mol; (c) +63 kJ/mol 10. Se aumentarmos a temperatura de uma substância de T1 até T2, a entalpia da substância cresce de H1 até H2, em que H2 – H1 = Cp(T2 – T1). Portanto, a entalpia de qualquer reagente ou produto na temperatura final está relacionada à entalpia na temperatura inicial por H2 = H1 + Cp(T2 – T1). Essa expressão se aplica a todas as substâncias que tomam parte na reação; logo, deduz-se a lei de Kirchhoff a partir daí. 11. 3g 12. (a)C6H5NH2(l)+31/4O2(g)à6CO2(g)+7/2H2O(l)+1/2N2(g) (b) m(CO2) = 0,4873g, m(H2O) = 0,1188g, m(N2) = 0,02639g; (c) 0,999 atm 13. endotérmico 14. (a) – 1,33 kJ; (b) – 53,2 kJ/mol 15. As equações de combustão mostram que as diferenças são (1) o consumo de 18/4 mols de O2(g) e (2) a produção de mais 6 mols de H2O(l) na combustão da anilina. Como DHf o de O2(g) é zero, a diferença total é a produção de mais 3 mols de H2O(l), – 857,5 kJ. 16. (a) 33,5g (b) A massa molar da sacarose é 71% maior do que a do ácido láurico, mas a sacarose produz 23% menos energia na conversão em dióxido de carbono do que o ácido láurico. 17. (a) –20,5 kJ/mol; (b) ácido fumárico; (c) DU. 18. (a) 1,5 L; (b) SO2; (c) 1,1 L; (d) + 40 J, contra o sistema; (e) – 2970 J, calor deixa o sistema; (f) – 2930 J 1 18 1 1 H 1,01 –259,3 –252,9 2 13 14 15 16 17 2 He 4,00 –268,9 2 3 Li 6,94 180,5 1342 4 Be 9,01 1287 2471 Número atômico, Símbolo Massa Atômica Média (u) Ponto de Fusão (oC) Ponto de Ebulição (oC) 5 B 10,81 2075 4000 6 C 12,01 4440 7 N 14,01 –210,0 –195,79 8 O 16,00 –218,8 –182,9 9 F 19,00 –219,7 –188,1 10 Ne 20,18 –248,6 –246,1 3 11 Na 22,99 97,80 883 12 Mg 24,30 650 1090 3 4 5 6 7 8 9 10 11 12 13 Al 26,98 660,32 2519 14 Si 28,09 1414 3265 15 P 30,97 44,15 280,5 16 S 32,01 115,2 444,6 17 Cl 35,45 –101,5 –34,04 18 Ar 39,95 –189,3 –185,8 4 19 K 39,10 63,38 759 20 Ca 40,08 842 1484 21 Sc 44,96 1541 2836 22 Ti 47,87 1668 3287 23 V 50,94 1910 3407 24 Cr 52,00 1907 2671 25 Mn 54,94 1246 2061 26 Fe 55,85 1538 2861 27 Co 58,93 1495 2927 28 Ni 58,69 1455 2913 29 Cu 63,46 1085 2562 30 Zn 65,39 419,5 907 31 Ga 69,72 29,76 2204 32 Ge 72,61 938,2 2833 33 As 74,92 614 34 Se 78,96 221 685 35 Br 79,90 –7,2 58,8 36 Kr 83,80 –157,4 –153,2 5 37 Rb 85,47 39,31 688 38 Sr 87,62 777 1382 39 Y 88,91 1522 3345 40 Zr 91,22 1855 4409 41 Nb 92,91 2477 4744 42 Mo 95,94 2623 4639 43 Tc (98) 2157 4265 44 Ru 101,07 2334 4150 45 Rh 102,91 1964 3695 46 Pd 106,42 1555 2963 47 Ag 107,87 961,8 2162 48 Cd 112,4 321,1 767 49 In 114,82 156,6 2072 50 Sn 118,71 231,9 2602 51 Sb 121,76 630,6 1587 52 Te 127,60 449,5 988 53 I 126,90 113,7 184,4 54 Xe 131,3 –111,7–108,0 6 55 Cs 132,91 28,44 671 56 Ba 137,33 727 1897 57 La 138,91 918 3464 72 Hf 178,49 2233 4603 73 Ta 180,95 3017 5458 74 W 183,84 3422 5555 75 Re 186,21 3186 5596 76 Os 190,23 3033 5012 77 Ir 192,22 2446 4428 78 Pt 195,08 1768 3825 79 Au 196,97 1064 2856 80 Hg 200,59 –38,83 356,7 81 Tl 204,38 302 1473 82 Pb 207,2 327,5 1749 83 Bi 208,98 271,4 1564 84 Po (209) 254 962 85 At (210) 302 86 Rn (222) –71 –61,7 7 87 Fr (223) 27 88 Ra (226) 7000 89 Ac (227) 1051 3198 104 Rf (267) 105 Db (268) 106 Sg (269) 107 Bh (270) 108 Hs (269) 109 Mt (278) 110 Ds (281) 111 Rg (281) 112 Cn (285) 113 Nh (286) 114 Fl (289) 115 Mc (288) 116 Lv (293) 117 Ts (294) 118 Og (294) PROF. PEDRO MADEIRA Série dos LANTANÍDEOS 58 Ce 140,12 798 3443 59 Pr 140,91 931 3443 60 Nd 144,24 1021 3074 61 Pm (145) 1042 3000 62 Sm 150,36 1074 1794 63 Eu 151,96 822 1529 64 Gd 157,25 1313 3273 65 Tb 158,93 1356 3230 66 Dy 162,50 1412 2567 67 Ho 164,93 1474 2700 68 Er 167,26 1529 2868 69 Tm 168,93 1545 1950 70 Yb 173,04 819 1196 71 Lu 174,97 1663 3402 Série dos ACTNÍDEOS 90 Th 232,04 1750 4788 91 Pa 231,04 1572 92 U 238,03 1135 4131 93 Np (237) 644 94 Pu (244) 640 3228 95 Am (243) 1176 2011 96 Cm (247) 1345 3100 97 Bk (247) 1050 98 Cf (251) 900 99 Es (252) 860 100 Fm (257) 1527 101 Md (258) 827 102No (259) 827 103 Lr (262) 1627